AMC

136
1989 AHSME January 25, 2004 1. (-1) 5 2 +1 2 5 = (A) -7 (B) -2 (C) 0 (D) 1 (E) 57 2. 1 9 + 1 16 = (A) 1 5 (B) 1 4 (C) 2 7 (D) 5 12 (E) 7 12 3. A square is cut into three rectangles along two lines parallel to a side. If the perimeter of each of the three rectangles is 24, then the area of the original square is (A) 24 (B) 36 (C) 64 (D) 81 (E) 96 4. ABCD is an isosceles trapezoid with side lengths AD = BC = 5, AB = 4, and DC = 10. DB is extended to a point E so that B is the midpoint of DE. DC is extended to a point F so that DF EF . Find CF . (A) 3.25 (B) 3.5 (C) 3.75 (D) 4 (E) 4.25 5. How many toothpicks are needed to construct a rectangular grid that is 20 toothpicks high and 10 toothpicks wide? (A) 30 (B) 200 (C) 410 (D) 420 (E) 430 6. If a, b > 0 and the triangle in the first quadrant bounded by the coordinate axes and the graph of ax + by = 6 has area 6, then ab = (A) 3 (B) 6 (C) 12 (D) 108 (E) 432 1

Transcript of AMC

Page 1: AMC

1989 AHSME

January 25, 2004

1. (!1)52+ 125

=(A) !7 (B) !2 (C) 0 (D) 1 (E) 57

2.!

19 + 1

16 =(A) 1

5 (B) 14 (C) 2

7 (D) 512 (E) 7

12

3. A square is cut into three rectangles along two lines parallel to a side. Ifthe perimeter of each of the three rectangles is 24, then the area of theoriginal square is(A) 24 (B) 36 (C) 64 (D) 81 (E) 96

4. ABCD is an isosceles trapezoid with side lengths AD = BC = 5,AB = 4, and DC = 10. DB is extended to a point E so that B is themidpoint of DE. DC is extended to a point F so that DF"EF . FindCF .(A) 3.25 (B) 3.5 (C) 3.75 (D) 4 (E) 4.25

5. How many toothpicks are needed to construct a rectangular grid that is20 toothpicks high and 10 toothpicks wide?(A) 30 (B) 200 (C) 410 (D) 420 (E) 430

6. If a, b > 0 and the triangle in the first quadrant bounded by thecoordinate axes and the graph of ax + by = 6 has area 6, then ab =(A) 3 (B) 6 (C) 12 (D) 108 (E) 432

1

Page 2: AMC

7. In # ABC, ! A = 100", ! B = 50", ! C = 30". AH is the altitude to BCand BM is a median to AC. Then ! MHC =(A) 15" (B) 22.5" (C) 30" (D) 40" (E) 45"

8. For how many integers n between 1 and 100 does x2 + x! n factor intothe product of two linear factors with integer coe!cients?(A) 0 (B) 1 (C) 2 (D) 9 (E) 10

9. Mr. and Mrs. Zeta want to name their baby Zeta so that its monogram(first, middle, and last initials) will be in alphabetical order with noletter repeated. How many such monograms are possible?(A) 276 (B) 300 (C) 552 (D) 600 (E) 15600

10. Consider the sequence defined recursively by u1 = a (any positivenumber), and un+1 = #1

un+1 , n = 1, 2, 3, . . .. For which of the followingvalues of n must un = a?(A) 14 (B) 15 (C) 16 (D) 17 (E) 18

11. Let a, b, c, and d be integers with a < 2b, b < 3c, and c < 4d. If d < 100,the largest possible value for a is(A) 2367 (B) 2375 (C) 2391 (D) 2399 (E) 2400

12. The tra!c on a certain east-west highway moves at a constant speed of60 miles per hour in both directions. An eastbound driver passes 20westbound vehicles in a five-minute interval. Assume vehicles in thewetbound lane are equally spaced. Which of the following is closest tothe number of westbound vehicles present in a 100-mile section ofhighway?(A) 100 (B) 120 (C) 200 (D) 240 (E) 400

13. Two strips of width 1 overlap at an angle of !. The area of the overlap is(A) sin! (B) 1

sin ! (C) 11#cos ! (D) 1

sin !2 (E) 11#cos !2

2

Page 3: AMC

14. cot 10 + tan 5 =(A) csc 5 (B) csc 10 (C) sec 5 (D) sec 10 (E) sin 15

15. In #ABC, AB = 5, BC = 7, AC = 9 and D is on BD = 5. Find theration AD : DC.(A) 4 : 3 (B) 7 : 5 (C) 11 : 6 (D) 13 : 5 (E) 19 : 8

16. A lattice point is a point in the plane with integer coordinates. Howmany lattice points are on the line segment whose endpoints are (3, 17)and (48, 281)? (Include both endpoints of the segment in your count.)(A) 2 (B) 4 (C) 6 (D) 16 (E) 46

17. The perimeter of an equilateral triangle exceeds the perimeter of asquare by 1989 cm. The length of each side of the triangle exceeds thelength of each side by d cm. The square has perimeter greater than 0.How many positive integers are not possible values for d?(A) 0 (B) 9 (C) 221 (D) 663 (E) infinitely many

18. The set of all real numbers x for which

x +"

x2 + 1! 1x +

$x2 + 1

is a rational number is the set of all(A) integers x (B) rational x (C) real x (D) x for which$

x2 + 1 is rational (E) x for which x +$

x2 + 1 is rational

19. A triangle is inscribed in a circle. The vertices of the triangle divide thecircle into three arcs of lengths 3, 4, and 5. What is the area of thetriangle?(A) 6 (B) 18

"2 (C) 9"2 ($

3! 1) (D) 9"2 ($

3 + 1) (E)9

"2 ($

3 + 3)

20. Let x be a real number selected uniformly at random between 100 and200. If [x] = 12, find the probability that [

$100x] = 120. ([x] means the

greatest integer less than or equal to x.)(A) 2

25 (B) 2412500 (C) 1

10 (D) 96625 (E) 1

3

Page 4: AMC

21. A square flag consists of a white background, two red strips of uniformwidth stretching from opposite corners, and a blue square in the areawhere the two red strips intersect. (The cross is symmetric with respectto each of the diagonals of the square.) If the entire cross (both the redarms and the blue center) takes up 36% of the area of the flag, whatpercent of the area of the flag is blue?(A) .5 (B) 1 (C) 2 (D) 3 (E) 6

22. A child has a set of 96 distinct blocks. Each block is one of 2 materials(plastic, wood), 3 sizes (small, medium, large), 4 colors (blue, green, red,yellow), and 4 shapes (circle, hexagon, square, triangle). How manyblocks in the set are di"erent from the “plastic medium red circle” inexactly two ways? (The “wood medium red square” is such a block.)(A) 29 (B) 39 (C) 48 (D) 56 (E) 62

23. A particle moves through the first quadrant as follows: during the firstminute, it moves from the origin to (1, 0). It them moves up 1 unit, left 1unit, up 1 unit, right 2 units, down 2 units, right 1 unit, up 3 units, left 3units, up 1 unit, right 4 units, down 4 units, right 1 unit, up 5, and so onas suggested by the diagram, at a rate of 1 unit per minute. At whichpoint will the particle be after exactly 1989 minutes?(A) (35, 44) (B) (36, 45) (C) (37, 45) (D) (44, 35) (E)(45, 36)

24. Five people are sitting at a round table. Let f % 0 be the number ofpeople sitting next to at least one female and m % 0 be the number ofpeople sitting next to at least one male. The number of possible orderedpairs (f,m) is(A) 7 (B) 8 (C) 9 (D) 10 (E) 11

25. In a certain cross-country meet between two teams of five runners each,a runner who finishes in the nth position contributes n to his team’sscore. The team with the lower score wins. If there are no ties amongthe runners, how many di"erent winning scores are possible?(A) 10 (B) 13 (C) 27 (D) 120 (E) 126

26. A regular octahedron is formed by joining the centers of adjoining facesof a cube. The ratio of teh volume of the octahedron to the volume ofthe cube is

4

Page 5: AMC

(A)$

312 (B)

$6

16 (C) 16 (D)

$2

8 (E) 14

27. Let n be a positive integer. If the euqation 2x + 2y + z = n has 28solutions in positive integers x, y, and z, then n must be either(A) 14 or 15 (B) 15 or 16 (C) 16 or 17 (D) 17 or 18(E) 18 or 19

28. Find the sum of the roots of tan2 x! 9 tanx + 1 = 0 that are betweenx = 0 and x = 2" radians.(A) "

2 (B) " (C) 3"2 (D) 3" (E) 4"

29. Find49#

k=0

(!1)k(992k)

where (nj ) = n!

j!(n#j)! .(A) !250 (B) !249 (C) 0 (D) 249 (E) 250

30. Suppose that 7 boys and 13 girls line up in a row. Let S be the numberof places in the row where a boy and a girl are standing next to eachother. For exmaple, for the row GBBGGGBGBGGGBGBGGBGG wehave S = 12. The average value of S (if all possible orders of these 20people are considered) is closest to(A) 9 (B) 10 (C) 11 (D) 12 (E) 13

5

Page 6: AMC

48th AHSME 1997 2

1. If a and b are digits for which

2 a! b 3

6 99 29 8 9

then a + b =

(A) 3 (B) 4 (C) 7 (D) 9 (E) 12

2. The adjacent sides of the decagon shown meet atright angles. What is its perimeter?

(A) 22 (B) 32 (C) 34 (D) 44 (E) 50•• •

••

••

••••

.......................................................................................................................................................................................................................................................................................................................................

........

........

........

........

........

........

........

........

........

........

..............................................................................................................................................................................................................................................................................................................................................................................................

8

12

2

3. If x, y, and z are real numbers such that

(x" 3)2 + (y " 4)2 + (z " 5)2 = 0,

then x + y + z =

(A) "12 (B) 0 (C) 8 (D) 12 (E) 50

4. If a is 50% larger than c, and b is 25% larger than c, then a is what percentlarger than b?

(A) 20% (B) 25% (C) 50% (D) 100% (E) 200%

5. A rectangle with perimeter 176 is divided into fivecongruent rectangles as shown in the diagram. Whatis the perimeter of one of the five congruent rectangles?

(A) 35.2 (B) 76 (C) 80 (D) 84 (E) 86

6. Consider the sequence1,"2, 3,"4, 5,"6, . . . ,

whose nth term is ("1)n+1·n. What is the average of the first 200 terms of thesequence?

(A) "1 (B) "0.5 (C) 0 (D) 0.5 (E) 1

7. The sum of seven integers is "1. What is the maximum number of the sevenintegers that can be larger than 13?

(A) 1 (B) 4 (C) 5 (D) 6 (E) 7

Page 7: AMC

48th AHSME 1997 3

8. Mientka Publishing Company prices its best seller Where’s Walter? as follows:

C(n) =

!"#

"$

12n, if 1 # n # 24,11n, if 25 # n # 48,10n, if 49 # n,

where n is the number of books ordered, and C(n) is the cost in dollars of nbooks. Notice that 25 books cost less than 24 books. For how many values ofn is it cheaper to buy more than n books than to buy exactly n books?

(A) 3 (B) 4 (C) 5 (D) 6 (E) 8

9. In the figure, ABCD is a 2 ! 2 square, E is themidpoint of AD, and F is on BE. If CF is per-pendicular to BE, then the area of quadrilateralCDEF is

(A) 2 (B) 3"$

3

2(C)

11

5(D)

$5

(E)9

4

• •

• ••

B C

A

F

DE

..........................................................................................................................................................................................................................................................................................................................................................................................................................................................................................................................................................

...................................

10. Two six-sided dice are fair in the sense that each face is equally likely to turnup. However, one of the dice has the 4 replaced by 3 and the other die has the3 replaced by 4. When these dice are rolled, what is the probability that thesum is an odd number?

(A)1

3(B)

4

9(C)

1

2(D)

5

9(E)

11

18

11. In the sixth, seventh, eighth, and ninth basketball games of the season, a playerscored 23, 14, 11, and 20 points, respectively. Her points-per-game average washigher after nine games than it was after the first five games. If her averageafter ten games was greater than 18, what is the least number of points shecould have scored in the tenth game?

(A) 26 (B) 27 (C) 28 (D) 29 (E) 30

12. If m and b are real numbers and mb > 0, then the line whose equation isy = mx + b cannot contain the point

(A) (0, 1997) (B) (0,"1997) (C) (19, 97) (D) (19,"97) (E) (1997, 0)

13. How many two-digit positive integers N have the property that the sum of Nand the number obtained by reversing the order of the digits of N is a perfectsquare?

(A) 4 (B) 5 (C) 6 (D) 7 (E) 8

Page 8: AMC

48th AHSME 1997 4

14. The number of geese in a flock increases so that the di!erence between thepopulations in year n+2 and year n is directly proportional to the populationin year n + 1. If the populations in the years 1994, 1995, and 1997 were 39,60, and 123, respectively, then the population in 1996 was

(A) 81 (B) 84 (C) 87 (D) 90 (E) 102

15. Medians BD and CE of triangle ABC are per-pendicular, BD = 8, and CE = 12. The area oftriangle ABC is

(A) 24 (B) 32 (C) 48 (D) 64 (E) 96• •

A

B

G

CD

E

............................................................................................................................................................................................................................................................................................................................................................................................................................

..............................................

..............................................

..............................................

................................................................................................................................................................................................................................................................................................

....................

16. The three row sums and the three column sums of the array

%

&'4 9 28 1 63 5 7

(

)*

are the same. What is the least number of entries that must be altered tomake all six sums di!erent from one another?

(A) 1 (B) 2 (C) 3 (D) 4 (E) 5

17. A line x = k intersects the graph of y = log5 x and the graph of y = log5(x+4).The distance between the points of intersection is 0.5. Given that k = a+

$b,

where a and b are integers, what is a + b?

(A) 6 (B) 7 (C) 8 (D) 9 (E) 10

18. A list of integers has mode 32 and mean 22. The smallest number in the listis 10. The median m of the list is a member of the list. If the list member mwere replaced by m + 10, the mean and median of the new list would be 24and m + 10, respectively. If m were instead replaced by m" 8, the median ofthe new list would be m" 4. What is m?

(A) 16 (B) 17 (C) 18 (D) 19 (E) 20

19. A circle with center O is tangent to the coordinateaxes and to the hypotenuse of the 30!-60!-90! trian-gle ABC as shown, where AB = 1. To the nearesthundredth, what is the radius of the circle?

(A) 2.18 (B) 2.24 (C) 2.31

(D) 2.37 (E) 2.41•

A

O

B

C

60!1

.....................................................................................................................................................................................................................................................................................................................................................................................................................................................................................................................................................................................................................................................................................

............................................................................................................................................................................................................................................................................................................................................................................................................................

Page 9: AMC

48th AHSME 1997 5

20. Which one of the following integers can be expressed as the sum of 100 con-secutive positive integers?

(A) 1,627,384,950 (B) 2,345,678,910 (C) 3,579,111,300

(D) 4,692,581,470 (E) 5,815,937,260

21. For any positive integer n, let

f(n) =

+log8 n, if log8 n is rational,0, otherwise.

What is1997,

n=1

f(n)?

(A) log8 2047 (B) 6 (C)55

3(D)

58

3(E) 585

22. Ashley, Betty, Carlos, Dick, and Elgin went shopping. Each had a wholenumber of dollars to spend, and together they had $56. The absolute di!erencebetween the amounts Ashley and Betty had to spend was $19. The absolutedi!erence between the amounts Betty and Carlos had was $7, between Carlosand Dick was $5, between Dick and Elgin was $4, and between Elgin andAshley was $11. How much did Elgin have?

(A) $6 (B) $7 (C) $8 (D) $9 (E) $10

23. In the figure, polygons A, E, and F are isosceles righttriangles; B, C, and D are squares with sides of length 1;and G is an equilateral triangle. The figure can be foldedalong its edges to form a polyhedron having the polygonsas faces. The volume of this polyhedron is

(A) 1/2 (B) 2/3 (C) 3/4 (D) 5/6 (E) 4/3

A

B

C D E

FG

.....................................................................................................................................................................................................................

...........................................................................................................

......................................................................................................................................................................................................................

24. A rising number, such as 34689, is a positive integer each digit of which islarger than each of the digits to its left. There are

-95

.= 126 five-digit rising

numbers. When these numbers are arranged from smallest to largest, the 97th

number in the list does not contain the digit

(A) 4 (B) 5 (C) 6 (D) 7 (E) 8

Page 10: AMC

48th AHSME 1997 6

25. Let ABCD be a parallelogram and let"#AA$,

"#BB$,

"#CC $, and

"#DD$ be parallel

rays in space on the same side of the plane determined by ABCD. If AA$ = 10,BB$ = 8, CC $ = 18, DD$ = 22, and M and N are the midpoints of A$C $ andB$D$, respectively, then MN =

(A) 0 (B) 1 (C) 2 (D) 3 (E) 4

26. Triangle ABC and point P in the same plane are given.Point P is equidistant from A and B, angle APB is twiceangle ACB, and AC intersects BP at point D. If PB = 3and PD = 2, then AD · CD =

(A) 5 (B) 6 (C) 7 (D) 8 (E) 9 • •

••

A B

P

CD

.................................................................................................................................................................................................................................................................................................................................................................................................................................................................................................................................

....................................................

....................................................

....................................................

....................................................

...............................................................................................................

27. Consider those functions f that satisfy f(x + 4) + f(x" 4) = f(x) for all realx. Any such function is periodic, and there is a least common positive periodp for all of them. Find p.

(A) 8 (B) 12 (C) 16 (D) 24 (E) 32

28. How many ordered triples of integers (a, b, c) satisfy

|a + b| + c = 19 and ab + |c| = 97?

(A) 0 (B) 4 (C) 6 (D) 10 (E) 12

29. Call a positive real number special if it has a decimal representation thatconsists entirely of digits 0 and 7. For example, 700

99 = 7.07 = 7.070707 . . . and77.007 are special numbers. What is the smallest n such that 1 can be writtenas a sum of n special numbers?

(A) 7 (B) 8 (C) 9 (D) 10

(E) 1 cannot be represented as a sum of finitely many special numbers

30. For positive integers n, denote by D(n) the number of pairs of di!erentadjacent digits in the binary (base two) representation of n. For example,D(3) = D(112) = 0, D(21) = D(101012) = 4, and D(97) = D(11000012) = 2.For how many positive integers n less than or equal to 97 does D(n) = 2?

(A) 16 (B) 20 (C) 26 (D) 30 (E) 35

Page 11: AMC

47th AHSME 1996 2

1. The addition below is incorrect. What is the largest digit that can be changedto make the addition correct?

6 4 18 5 2

+ 9 7 32 4 5 6

(A) 4 (B) 5 (C) 6 (D) 7 (E) 8

2. Each day Walter gets $3 for doing his chores or $5 for doing them exceptionallywell. After 10 days of doing his chores daily, Walter has received a total of$36. On how many days did Walter do them exceptionally well?

(A) 3 (B) 4 (C) 5 (D) 6 (E) 7

3.(3!)!

3!=

(A) 1 (B) 2 (C) 6 (D) 40 (E) 120

4. Six numbers from a list of nine integers are 7, 8, 3, 5, 9, and 5. The largestpossible value of the median of all nine numbers in this list is

(A) 5 (B) 6 (C) 7 (D) 8 (E) 9

5. Given that 0 < a < b < c < d, which of the following is the largest?

(A)a + b

c + d(B)

a + d

b + c(C)

b + c

a + d(D)

b + d

a + c(E)

c + d

a + b

6. If f(x) = x(x+1)(x + 2)(x+3) then f(0) + f(!1) + f(!2) + f(!3) =

(A) !8/9 (B) 0 (C) 8/9 (D) 1 (E) 10/9

7. A father takes his twins and a younger child out to dinner on the twins’birthday. The restaurant charges $4.95 for the father and $0.45 for each yearof a child’s age, where age is defined as the age at the most recent birthday. Ifthe bill is $9.45, which of the following could be the age of the youngest child?

(A) 1 (B) 2 (C) 3 (D) 4 (E) 5

Page 12: AMC

47th AHSME 1996 3

8. If 3 = k · 2r and 15 = k · 4r, then r =

(A) ! log2 5 (B) log5 2 (C) log10 5 (D) log2 5 (E)5

2

9. Triangle PAB and square ABCD are in perpendicularplanes. Given that PA = 3, PB = 4, and AB = 5,what is PD?

(A) 5 (B)"

34 (C)"

41 (D) 2"

13 (E) 8......................................................................................... ............

........................................................................................................................................

...........................

•P

•A•B

•D•C

10. How many line segments have both their endpoints located at the vertices ofa given cube?

(A) 12 (B) 15 (C) 24 (D) 28 (E) 56

11. Given a circle of radius 2, there are many line segments of length 2 that aretangent to the circle at their midpoints. Find the area of the region consistingof all such line segments.

(A) !/4 (B) 4! ! (C) !/2 (D) ! (E) 2!

12. A function f from the integers to the integers is defined as follows:

f(n) =!

n + 3 if n is oddn/2 if n is even

Suppose k is odd and f(f(f(k))) = 27. What is the sum of the digits of k?

(A) 3 (B) 6 (C) 9 (D) 12 (E) 15

13. Sunny runs at a steady rate, and Moonbeam runs m times as fast, where m isa number greater than 1. If Moonbeam gives Sunny a head start of h meters,how many meters must Moonbeam run to overtake Sunny?

(A) hm (B)h

h + m(C)

h

m! 1(D)

hm

m! 1(E)

h + m

m! 1

14. Let E(n) denote the sum of the even digits of n. For example, E(5681) =6 + 8 = 14. Find E(1) + E(2) + E(3) + · · · + E(100).

(A) 200 (B) 360 (C) 400 (D) 900 (E) 2250

Page 13: AMC

47th AHSME 1996 4

15. Two opposite sides of a rectangle are each divided into n congruentsegments, and the endpoints of one segment arejoined to the center to form triangle A. The othersides are each divided into m congruent segments,and the endpoints of one of these segments arejoined to the center to form triangle B. [See figurefor n = 5, m = 7.] What is the ratio of the areaof triangle A to the area of triangle B?

• •

• •

• •

• •

• •

B

A....................................................................................................................................................................................................................

.................................................................................................................................................................................................................

(A) 1 (B) m/n (C) n/m (D) 2m/n (E) 2n/m

16. A fair standard six-sided die is tossed three times. Given that the sum of thefirst two tosses equals the third, what is the probability that at least one “2”is tossed?

(A)1

6(B)

91

216(C)

1

2(D)

8

15(E)

7

12

17. In rectangle ABCD, angle C is trisected by CFand CE, where E is on AB, F is on AD, BE = 6,and AF = 2. Which of the following is closest tothe area of the rectangle ABCD?

(A) 110 (B) 120 (C) 130 (D) 140

(E) 150•

A•B

•C•D

•E

•F62

.........................................................................................................................................................................................................................................................................................................................................................................................................................................

18. A circle of radius 2 has center at (2, 0). A circle of radius 1 has center at (5, 0).A line is tangent to the two circles at points in the first quadrant. Which ofthe following is closest to the y-intercept of the line?

(A)"

2/4 (B) 8/3 (C) 1 +"

3 (D) 2"

2 (E) 3

19. The midpoints of the sides of a regular hexagonABCDEF are joined to form a smaller hexagon.What fraction of the area of ABCDEF is enclosedby the smaller hexagon?

(A)1

2(B)

"3

3(C)

2

3(D)

3

4(E)

"3

2

•A •B

•C

•D

•E

•F

.................................................................................................................................................................................................................................................................................................................................................................................................................... .......................................................................................

.......................................................................................................................

................................

.......................

................................

................................

.......................

Page 14: AMC

47th AHSME 1996 5

20. In the xy-plane, what is the length of the shortest path from (0, 0) to (12, 16)that does not go inside the circle (x! 6)2 + (y ! 8)2 = 25?

(A) 10"

3 (B) 10"

5 (C) 10"

3 +5!

3(D) 40

"3

3(E) 10 + 5!

21. Triangles ABC and ABD are isosceles with AB =AC = BD, and BD intersects AC at E. If BD # AC,then ! C + ! D is

(A) 115" (B) 120" (C) 130" (D) 135"

(E) not uniquely determined

•A

•B

•C

•D•E

.....................................................................................................................................................................................................................................................................................................................................................................................................................................................................................................................................................................................................................................

.................................

22. Four distinct points, A, B, C, and D, are to be selected from 1996 pointsevenly spaced around a circle. All quadruples are equally likely to be chosen.What is the probability that the chord AB intersects the chord CD?

(A)1

4(B)

1

3(C)

1

2(D)

2

3(E)

3

4

23. The sum of the lengths of the twelve edges of a rectangular box is 140, andthe distance from one corner of the box to the farthest corner is 21. The totalsurface area of the box is

(A) 776 (B) 784 (C) 798 (D) 800 (E) 812

24. The sequence

1, 2, 1, 2, 2, 1, 2, 2, 2, 1, 2, 2, 2, 2, 1, 2, 2, 2, 2, 2, 1, 2, . . .

consists of 1’s separated by blocks of 2’s with n 2’s in the nth block. The sumof the first 1234 terms of this sequence is

(A) 1996 (B) 2419 (C) 2429 (D) 2439 (E) 2449

25. Given that x2 + y2 = 14x + 6y + 6, what is the largest possible value that3x + 4y can have?

(A) 72 (B) 73 (C) 74 (D) 75 (E) 76

Page 15: AMC

47th AHSME 1996 6

26. An urn contains marbles of four colors: red, white, blue, and green. Whenfour marbles are drawn without replacement, the following events are equallylikely:

(a) the selection of four red marbles;

(b) the selection of one white and three red marbles;

(c) the selection of one white, one blue, and two red marbles; and

(d) the selection of one marble of each color.

What is the smallest number of marbles satisfying the given condition?

(A) 19 (B) 21 (C) 46 (D) 69 (E) more than 69

27. Consider two solid spherical balls, one centered at (0, 0, 212 ) with radius 6, and

the other centered at (0, 0, 1) with radius 92 . How many points (x, y, z) with

only integer coordinates (lattice points) are there in the intersection of theballs?

(A) 7 (B) 9 (C) 11 (D) 13 (E) 15

28. On a 4$ 4$ 3 rectangular parallelepiped, verticesA, B, and C are adjacent to vertex D. The per-pendicular distance from D to the plane containingA, B, and C is closest to(A) 1.6 (B) 1.9 (C) 2.1 (D) 2.7 (E) 2.9 .................

..................................

..................................

..................................

.

..................................

..................................

..................................

..................

..................................

..................................

..................................

..................

..................................

..................................

..................................

..................•A

•B

•C

•D

.....................................................................................................................................................................................................................................................

3

44

29. If n is a positive integer such that 2n has 28 positive divisors and 3n has 30positive divisors, then how many positive divisors does 6n have?

(A) 32 (B) 34 (C) 35 (D) 36 (E) 38

30. A hexagon inscribed in a circle has three consecutive sides each of length 3 andthree consecutive sides each of length 5. The chord of the circle that dividesthe hexagon into two trapezoids, one with three sides each of length 3 and theother with three sides each of length 5, has length equal to m/n, where mand n are relatively prime positive integers. Find m + n.

(A) 309 (B) 349 (C) 369 (D) 389 (E) 409

Page 16: AMC

SOLUTIONS 1997 AHSME 2

Note: Throughout this pamphlet, [P1P2 . . . Pn] denotes the areaenclosed by polygon P1P2 . . . Pn.

1. (C) Since a!3 has units digit 9, a must be 3. Hence b!3 has units digit 2, sob must be 4. Thus, a+b = 7.

2. (D) Each polygon in the sequence below has the same perimeter, which is 44.

•• •••

••

••••

............................................................................................................................................................................................................................................................

........

........

........

........

........

........

........

........

........

........

................................................................................................................................................................................................................................................................................................................................................

8

12

2

•• •••

••

••••............................................................................................................................................................................................................................................................................................................................

........

.......................................................................................................................................................................

........

........

........

........

............................................................................

.................................

10

12•• •

••

•••••....................................................................................................................................................................................................................................................................................................................................................

........

........................................................................................................................................

..........................................................................................................

.........................................................................................................................

10

12•• •

••••••............................................................................................................................................................................................................................................................................................................................

........

.

...............................................................................................................................................................................................................................................................................................

10

12

3. (D) Since each summand is nonnegative, the sum is zero only when each termis zero. Hence the only solution is x = 3, y = 4, and z = 5, so the desiredsum is 12.

4. (A) If a is 50% larger than c, then a = 1.5c. If b is 25% larger than c, then

b = 1.25c. So ab = 1.5c

1.25c = 65 = 1.20, and a = 1.20b. Therefore, a is 20%

larger than b.

5. (C) Let x and y denote the width and height of one of the five rectangles,with x < y. Then 5x + 4y = 176 and 3x = 2y. Solve simultaneously to getx = 16 and y = 24. The perimeter in question is 2·16 + 2·24 = 80.

6. (B) The 200 terms can be grouped into 100 odd-even pairs, each with a sumof "1. Thus the sum of the first 200 terms is "1 ·100 = "100, and the averageof the first 200 terms is "100/200 = "0.5.

7. (D) Not all seven integers can be larger than 13. If six of them were each 14,then the seventh could be "(6! 14)" 1, so that the sum would be "1.

8. (D) The cost of 25 books is C(25) = 25! $11 = $275. The cost of 24 books isC(24) = 24!$12 = $288, while 23 and 22 books cost C(23) = 23!$12 = $276and C(22) = 22!$12 = $264, respectively. Thus it is cheaper to buy 25 booksthan 23 or 24 books. Similarly, 49 books cost less than 45, 46, 47, or 48 books.In these six cases the total cost is reduced by ordering more books. There areno other cases.

Page 17: AMC

SOLUTIONS 1997 AHSME 3

OR

A discount of $1 per book is given on orders of at least 25 books. This discountis larger than 2!$12, the cost of two books at the regular price. Thus, n = 23and n = 24 are two values of n for which it is cheaper to order more books.Similarly, we receive an additional $1 discount per book when we buy at least49 books. This discount would enable us to buy 4 more books at $10 per book,so there are 4 more values of n: 45, 46, 47, and 48, for a total of 6 values.

9. (C) In right triangle BAE, BE =#

22 + 12 =#

5. Since $CFB % $BAE,

it follows that [CFB] =!

CBBE

"2·[BAE] =

!2!5

"2· 12(2·1) = 4

5 . Then [CDEF ] =

[ABCD]" [BAE]" [CFB] = 4" 1" 45 = 11

5 .

OR

Draw the figure in the plane as shown with B atthe origin. An equation of the line BE is y = 2x,and, since the lines are perpendicular, an equationof the line CF is y = "1

2(x" 2). Solve these twoequations simultaneously to get F = (2/5, 4/5)and

• •

• ••

B(0, 0) C(2, 0)

A(0, 2)

F

DE(1, 2)

........................................................................................................................................................................................................................................................................................................................................

....................................................................................................................................................................................................................................................................................................................................................................

....................................

....................................

....................................

...............................

...............................

[CDEF ] = [DEF ] + [CDF ] =1

2(1)

#2" 4

5

$+

1

2(2)

#2" 2

5

$=

11

5.

10. (D) There are 36 equally likely outcomes as shown in the following table.

(1, 1) (1,2) (1,4) (1,4) (1, 5) (1,6)

(2,1) (2, 2) (2, 4) (2, 4) (2,5) (2, 6)

(3, 1) (3,2) (3,4) (3,4) (3, 5) (3,6)

(3, 1) (3,2) (3,4) (3,4) (3, 5) (3,6)

(5, 1) (5,2) (5,4) (5,4) (5, 5) (5,6)

(6,1) (6, 2) (6, 4) (6, 4) (6,5) (6, 6)

Exactly 20 of the outcomes have an odd sum. Therefore, the probability is2036 = 5

9 .OR

The sum is odd if and only if one number is even and the other is odd. Theprobability that the first number is even and the second is odd is 1

3 · 13 , and

the probability that the first is odd and the second is even is 23 · 2

3 . Therefore,the required probability is (1

3)2 + (2

3)2 = 5

9 .

Page 18: AMC

SOLUTIONS 1997 AHSME 4

11. (D) The average for games six through nine was (23 + 14 + 11 + 20)/4 = 17,which exceeded her average for the first five games. Therefore, she scored atmost 5 · 17 " 1 = 84 points in the first five games. Because her average afterten games was more than 18, she scored at least 181 points in the ten games,implying that she scored at least 181"84"68 = 29 points in the tenth game.

12. (E) Since mb > 0, the slope and the y-intercept of the line are either bothpositive or both negative. In either case, the line slopes away from the positivex-axis and does not intersect it. The answer is therefore (1997, 0). Note thatthe other four points lie on lines for which mb > 0. For example, (0, 1997) lieson y = x + 1997; (0,"1997) lies on y = "x" 1997; (19, 97) lies on y = 5x + 2;and (19,"97) lies on y = "5x" 2.

13. (E) Let N = 10x + y. Then 10x + y + 10y + x = 11(x + y) must be a perfectsquare. Since 1 & x + y & 18, it follows that x + y = 11. There are eight suchnumbers: 29, 38, 47, 56, 65, 74, 83, and 92.

14. (B) Let x be the number of geese in 1996, and let k be the constant ofproportionality. Then x " 39 = 60k and 123 " 60 = kx. Solve the secondequation for k, and use that value to solve for x in the first equation, obtainingx"39 = 60· 63

x . Thus x2"39x"3780 = 0. Factoring yields (x"84)(x+45) = 0.Since x is positive, it follows that x = 84.

15. (D) Let the medians meet at G. Then CG = (2/3)CE = 8and the area of triangle BCD is (1/2)BD · CG =(1/2) · 8 · 8 = 32. Since BD is a median, trianglesABD and DBC have the same area. Hence the areaof the triangle is 64. • •

••

A

B

G

CD

E

..............................

..........................................................................................................................................................................................................................................................................................................................

..............................................

..............................................

........................................................................................................................................................................................................................................

...................................................

OR

Since the medians are perpendicular, the area of the quadrilateral BCDE ishalf the product of the diagonals 1

2(12)(8) = 48. (Why?) However, D and Eare midpoints, which makes the area of triangle AED one fourth of the area oftriangle ABC. Thus the area of BCDE is three fourths of the area of triangleABC. It follows that the area of triangle ABC is 64.

Page 19: AMC

SOLUTIONS 1997 AHSME 5

16. (D) If only three entries are altered, then either two lines are not changed atall, or some entry is the only entry in its row and the only entry in its columnthat is changed. In either case, at least two of the six sums remain the same.However, four alterations are enough. For example, replacing 4 by 5, 1 by 3,2 by 7, and 6 by 9 results in the array

%

&'5 9 78 3 93 5 7

(

)*

for which the six sums are all di!erent.

17. (A) The line x = k intersects y = log5(x+4) and y = log5 x at (k, log5(k+4))and (k, log5 k), respectively. Since the length of the vertical segment is 0.5,

0.5 = log5(k + 4)" log5 k = log5

k + 4

k,

so k + 4k =

#5. Solving for k yields k = 4#

5" 1= 1 +

#5, so a + b = 6.

18. (E) When 10 is added to a number in the list, the mean increases by 2, sothere must be five numbers in the original list whose sum is 5 ·22 = 110. Since10 is the smallest number in the list and m is the median, we may assume

10 & a & m & b & c,

denoting the other members of the list by a, b, and c. Since the mode is 32, wemust have b = c = 32; otherwise, 10 + m + a + b + c would be larger than 110.So a + m = 36. Since decreasing m by 8 decreases the median by 4, a mustbe 4 less than m. Solving a + m = 36 and m" a = 4 for m gives m = 20.

19. (D) Let D and E denote the points of tangencyon the y- and x-axes, respectively, and let BCbe tangent to the circle at F . Tangents to acircle from a point are equal, so BE = BF andCD = CF . Let x = BF and y = CF . Becausex + y = BC = 2, the radius of the circle is

(1 + x) + (#

3 + y)

2=

3 +#

3

2' 2.37. • •

• •

E

DO

B

C

F

A•

60"1

y

y

x

x1

......................................................................................................................................................................................................................................................................................................................................................................................................................................................................................................

..................................................................................................................................................................................................................................................................................................................................................................................................................

..........................

..................................................................................................................................................................................................

.......................................................................................................................................................................................................................................................................................................................................................................................................

Page 20: AMC

SOLUTIONS 1997 AHSME 6

OR

Let r be the radius of the circle. The area of square AEOD, r2, may also beexpressed as the sum of the areas of quadrilaterals OFBE and ODCF andtriangle ABC. This is given by rx + ry +

!3

2 , where x + y = 2. Thus

r2 = 2r +

#3

2.

Solving for r using the quadratic formula yields the positive solution

r = 1 +

+

1 +

#3

2' 2.37.

Note. The circle in question is called an escribed circle of the triangle ABC.

20. (A) Since1 + 2 + 3 + · · · + 100 = (100)(101)/2 = 5050,

it follows that the sum of any sequence of 100 consecutive positive integersstarting with a + 1 is of the form

(a+1) + (a+2) + (a+3) + · · · + (a+100) = 100a + (1 + 2 + 3 + · · · + 100)

= 100a + 5050.

Consequently, such a sum has 50 as its rightmost two digits. Choice A is thesum of the 100 integers beginning with 16,273,800.

21. (C) Since log8 n = 13(log2 n), it follows that log8 n is rational if and only

if log2 n is rational. The nonzero numbers in the sum will therefore be allnumbers of the form log8 n, where n is an integral power of 2. The highestpower of 2 that does not exceed 1997 is 210, so the sum is:

log8 1 + log8 2 + log8 22 + log8 23 + · · · + log8 210 =

0 +1

3+

2

3+

3

3+ · · · + 10

3=

55

3.

Challenge. Prove that log2 3 is irrational. Prove that, for every integer n,log2 n is rational if and only if n is an integral power of 2.

Page 21: AMC

SOLUTIONS 1997 AHSME 7

22. (E) Let A, B, C,D, and E denote the amounts Ashley, Betty, Carlos, Dick,and Elgin had for shopping, respectively. Then A " B = ±19, B " C =±7, C"D = ±5, D"E = ±4, and E"A = ±11. The sum of the left sides iszero, so the sum of the right sides must also be zero. In other words, we mustchoose some subset S of {4, 5, 7, 11, 19} which has the same element-sum asits complement. Since 4 + 5 + 7 + 11 + 19 = 46, the sum of the members ofS is 23. Hence S is either the set {4, 19} or its complement {5, 7, 11}. Thuseither A " B and D " E are the only positive di!erences or B " C, C " D,and E " A are. In the former case, expressing A, B, C, and D in terms of E,we get 5E +6 = 56, which yields E = 10. In the latter case, the same strategyyields 5E " 6 = 56, which leads to non-integer values. Hence E = 10.

23. (D) The polyhedron is a unit cube with acorner cut o!. The missing corner may beviewed as a pyramid whose altitude is 1and whose base is an isosceles right triangle(shaded in the figure). The area of the baseis 1/2. The pyramid’s volume is therefore(1/3)(1/2)(1) = 1/6, so the polyhedron’svolume is 1" 1/6 = 5/6.

...................................................................................................................................................................................................................................................................................................................................................................................................................................................................................

........................

........................

........................

........................

........................

................

..........................................................

..........................................................

.........................................................................................................................................................................................................................................................................................................................................................

............................................................................................................................................................................................................................................................................

........................

........................

........................

........................

........................

......................................

..............................................

..............................................

..............................................

.....

.....................................................................................

..........................

.......................... ............. ............. ............. ................................................................

.........................

..

..

.

..

..

..

..

..

..

..

..

..

.

..

..

..

.

..

..

..

..

..

..

..

..

..

..

..

..

.

..

..

..

..

.

..

..

..

..

..

..

..

..

..

..

..

..

..

..

..

.

..

..

..

..

..

.

..

..

..

..

..

..

..

..

..

..

..

..

..

..

..

..

..

..

.

..

..

..

..

..

..

.

..

..

..

..

..

..

..

..

..

..

..

..

..

..

..

..

..

..

..

..

..

.

..

..

..

..

..

..

..

.

..

..

..

..

..

..

..

..

..

..

..

..

..

..

..

..

..

..

..

..

..

..

..

..

.

..

..

..

..

..

..

..

..

.

..

..

..

..

..

..

..

..

..

..

..

..

..

..

..

..

..

..

..

..

..

..

..

..

..

..

..

.

24. (B) The number of five-digit rising numbers that begin with 1 is!

84

"= 70,

since the rightmost four digits must be chosen from the eight-member set{2, 3, 4, 5, 6, 7, 8, 9}, and, once they are chosen, they can be arranged in in-

creasing order in just one way. Similarly, the next!

74

"= 35 integers in the

list begin with 2. So the 97th integer in the list is the 27th among those thatbegin with 2. Among those that begin with 2, there are

!63

"= 20 that be-

gin with 23 and!

53

"= 10 that begin with 24. Therefore, the 97th is the 7th

of those that begin with 24. The first six of those beginning with 24 are24567, 24568, 24569, 24578, 24579, 24589, and the seventh is 24678. The digit5 is not used in the representation.

OR

As above, note that there are 105 integers in the list starting with either 1 or 2,so the 97th one is ninth from the end. Count backwards: 26789, 25789, 25689,25679, 25678, 24789, 24689, 24679, 24678 . Thus 5 is a missing digit.

Page 22: AMC

SOLUTIONS 1997 AHSME 8

25. (B) Let O be the intersection of AC and BD. Then O is the midpoint of ACand BD, so OM and ON are the midlines in trapezoids ACC #A# and BDD#B#,respectively. Hence OM = (10+18)/2 = 14 and ON = (8+22)/2 = 15. SinceOM(AA#, ON(BB#, and AA#(BB#, it follows that O,M, and N are collinear.Therefore,

MN = |OM "ON | = |14" 15| = 1.

• •

••

••

A B

CD

A# B#

C #

D#

O

M

N

10 8

18

22

........................................................................................

........................................................................................

........................................................................................

.................................................................................................................................................................................

..............................................................................................................................................................................................................................................................................

...............................................................................................................................................................................................................................................................................................................................................................................................

.....................................................................................................................................................................................................................................................................................................................................................................................................................

Note. In general, if AA# = a, BB# = b, CC # = c, and DD# = d, thenMN = |a" b + c" d|/2.

26. (A) Construct a circle with center P and radius PA.Then C lies on the circle, since the angle ACB ishalf angle APB. Extend BP through P to get adiameter BE. Since A, B, C, and E are concyclic,

AD · CD = ED · BD

= (PE + PD)(PB " PD)

= (3 + 2)(3" 2)

= 5. • •

••

A B

P

CD

E

................................................................................................................................................................................................................................................................................................................................................................................................................................................

....................................................

....................................................

....................................................

.........................................................................................................

.............

.............

.............

.............

.............

.............

.....................................................................................................................................

........................................................................................................................................................................................................................................................

.......................

............................

.................................................

..............................................................................................................................................................................................................................................................................................................................................................................................................

Page 23: AMC

SOLUTIONS 1997 AHSME 9

OR

Let E denote the point where AC intersects theangle bisector of angle APB. Note that$PED %$CBD. Hence DE/2 = 1/DC so DE · DC = 2.Apply the Angle Bisector Theorem to $APD toobtain

EA

DE=

PA

PD=

3

2. • •

••

A B

P

CD

E..................................................................................................................................................................................................................................................................................................................................................................................................................................................................................................................................................................................................................

....................................................

....................................................

....................................................

....................................................

.......................................................................................................................................................................

................................................................................................................................................................................

Thus DA ·DC = (DE +EA) ·DC = (DE +1.5DE) ·DC = 2.5DE ·DC = 5.

27. (D) We may replace x with x + 4 in

f(x + 4) + f(x" 4) = f(x) (1)

to get

f(x + 8) + f(x) = f(x + 4). (2)

From (1) and (2), we deduce that f(x + 8) = "f(x " 4). Replacing x withx + 4, the latter equation yields f(x + 12) = "f(x). Now replacing x inthis last equation with x + 12 yields f(x + 24) = "f(x + 12). Consequently,f(x + 24) = f(x) for all x, so that a least period p exists and is at most 24.

On the other hand, the function f(x) = sin!

!x12

"has fundamental period 24,

and satisfies (1), so p ) 24. Hence p = 24.

OR

Let x0 be arbitrary, and let yk = f(x0+4k) for k = 0, 1, 2, . . . . Then f(x+4) =f(x) " f(x " 4) for all x implies yk+1 = yk " yk$1, so if y0 = a and y1 = b,then y2 = b" a, y3 = "a, y4 = "b, y5 = a" b, y6 = a, and y7 = b. It followsthat the sequence (yk) is periodic with period 6 and, since x0 was arbitrary,f is periodic with period 24. Since f(x) = sin(!x

12 ) has fundamental period 24and satisfies f(x + 4) + f(x" 4) = f(x), it follows that p ) 24. Hence p = 24.

Page 24: AMC

SOLUTIONS 1997 AHSME 10

28. (E) If c ) 0, then ab" |a+b| = 78, so (a"1)(b"1) = 79 or (a+1)(b+1) = 79.Since 79 is prime, {a, b} is {2, 80}, {"78, 0}, {0, 78}, or {"80,"2}. Hence|a + b| = 78 or |a + b| = 82, and, from the first equation in the problemstatement, it follows that c < 0, a contradiction.

On the other hand, if c < 0, then ab + |a + b| = 116, so (a + 1)(b + 1) = 117in the case that a + b > 0 and (a" 1)(b" 1) = 117 in the case that a + b < 0.Since 117 = 32 · 13, we distinguish the following cases:

{a, b} = {0, 116} yields c = "97;

{a, b} = {2, 38} yields c = "21;

{a, b} = {8, 12} yields c = "1;

{a, b} = {"116, 0} yields c = "97;

{a, b} = {"38,"2} yields c = "21;

{a, b} = {"12,"8} yields c = "1.

Since a and b are interchangeable, each of these cases leads to two solutions,for a total of 12.

29. (B) Suppose 1 = x1 +x2 + · · ·+xn where x1, x2, . . . , xn are special and n & 9.For k = 1, 2, 3, . . . , let ak be the number of elements of {x1, x2, . . . , xn} whosekth decimal digit is 7. Then

1 =7a1

10+

7a2

102+

7a3

103+ · · · ,

which yields1

7= 0.142857 =

a1

10+

a2

102+

a3

103+ · · · .

Hence a1 = 1, a2 = 4, a3 = 2, a4 = 8, etc. In particular, this implies thatn ) 8. On the other hand,

x1 = 0.700, x2 = x3 = 0.07, x4 = x5 = 0.077777, and x6 = x7 = x8 = 0.000777

are 8 special numbers whose sum is

700700 + 2(70707) + 2(77777) + 3(777)

999999= 1.

Thus the smallest n is 8.

Page 25: AMC

SOLUTIONS 1997 AHSME 11

30. (C) In order that D(n) = 2, the binary representation of n must consist of ablock of 1’s followed by a block of 0’s followed by a block of 1’s. Among theintegers n with d-digit binary representations, how many are there for whichD(n) = 2? If the 0’s block consists of just one 0, there are d " 2 possible

locations for the 0. If the block consists of multiple 0’s, then there are!

d$22

"

such blocks, since only the first and last places for the 0’s need to be identified.Thus there are (d" 2) + 1

2(d" 2)(d" 3) = 12(d " 2)(d " 1) values of n with

d binary digits such that D(n) = 2. The binary representation of 97 hasseven digits, so all the 3-, 4-, 5-, and 6-digit binary integers are less than 97.(We need not consider the 1- and 2-digit binary integers.) The sum of thevalues of 1

2(d " 2)(d " 1) for d = 3, 4, 5, and 6 is 20. We must also considerthe 7-digit binary integers less than or equal to 11000012 = 97. If the initialblock of 1’s contains three or more 1’s, then the number would be greaterthan 97; by inspection, if there are one or two 1’s in the initial 1’s block,there are respectively five or one acceptable configurations of the 0’s block. Itfollows that the number of solutions of D(n) = 2 within the required range is20 + 5 + 1 = 26.

OR

Note that D(n) = 2 holds exactly when the binary representation of n consistsof an initial block of 1’s, followed by a block of 0’s, and then a final block of1’s. The number of nonnegative integers n & 27" 1 = 127 for which D(n) = 2

is thus!

73

"= 35, since for each n, the corresponding binary representation is

given by selecting the position of the leftmost bit in each of the three blocks. If98 & n & 127, the binary representation of n is either (a) 110XXXX2 or (b)111XXXX2. Consider those n’s for which D(n) = 2. By the same argumentas above, there are three of type (a), namely 11011112 = 111, 11001112 =

103, and 11000112 = 99. There are!

42

"= 6 of type (b). It follows that the

number of solutions of D(n) = 2 for which 1 & n & 97 is 35" (3 + 6) = 26.

Page 26: AMC

SOLUTIONS 1998 AHSME 2

1. (E) Only the rectangle that goes in position II must match on both verticalsides. Since rectangle D is the only one for which these matches exist, it mustbe the one that goes in position II. Hence the rectangle that goes in positionI must be E.

CB

ADE 70

9

2

48

7

5

52

3

8

36

1

0

69

4

1

2. (E) We need to make the numerator large while making the denominatorsmall. The smallest the denominator can be is 0 + 1 = 1. The largest thenumerator can be is 9+8 = 17. The fraction 17/1 is an integer, so A+B = 17.

3. (D) The subtraction problem posed is equivalent to the addition problem

4 8 b+ c 7 3

7 a 2

which is easier to solve. Since b+3 = 12, b must be 9. Since 1 + 8 + 7 hasunits digit a, a must be 6. Because 1 + 4+ c= 7, c= 2. Hence a+ b+c= 6 + 9 + 2 = 17.

4. (E) Notice that the operation has the property that, for any r, a, b, and c,

[ra, rb, rc] =ra + rb

rc= [a, b, c].

Thus all three of the expressions [60, 30, 90], [2, 1, 3], and [10, 5, 15] have thesame value, which is 1. So [[60, 30, 90], [2, 1, 3], [10, 5, 15]] = [1, 1, 1] = 2.

5. (C) Factor the left side of the given equation:

21998 ! 21997 ! 21996 + 21995 = (23 ! 22 ! 2 + 1)21995 = 3 · 21995 = k · 21995,

so k = 3.

6. (C) The number 1998 has prime factorization 2 · 33 · 37. It has eight factor-pairs: 1"1998 = 2"999 = 3"666 = 6"333 = 9"222 = 18"111 = 27"74 =37" 54 = 1998. Among these, the smallest di!erence is 54! 37 = 17.

Page 27: AMC

SOLUTIONS 1998 AHSME 3

7. (D)3

!

N3"

N 3#

N =3

!

N3"

N · N13 =

3

!

N3"

N43 =

3"

N · N49 =

3"

N139 = N

1327 .

OR

3

!

N3"

N 3#

N =#N

$N(N)

13

% 13

& 13

=$N

$N

13 · N

19

%% 13 = N

13 · N 1

9 · N 127 = N

1327 .

8. (D) The area of each trapezoid is 1/3, so 12 · 1

2(x + 12) = 1

3 . Simplifying yieldsx + 1

2 = 43 , and it follows that x = 5/6.

OR

...................................................................................................................................................................

.......................................................................................................................................................................................................................................................................................................................................................................................................................................................................................................................................................................................................................................................................................................................................................................................................................................................................................................................................................................................................................................................................................................................................................................................................................................................................................................................................................

.......................................................................................................................................................................................................................................................................................................................................................................................................................................................................................................................................................................................................................................................................................................................................................................................................................................................................................................................................................................................................................................................................................................................................................................................................................................................................................................................................................

R

R R

R R

S

S S 2S + 2R = S + 3R... S = R... b = 2a

...

a + b + b + a = 1

2b + a = 5/6•

••

•••

• •

a

ba b

9. (D) Let N be the number of people in the audience. Then 0.2N people heard60 minutes, 0.1N heard 0 minutes, 0.35N heard 20 minutes, and 0.35N heard40 minutes. In total, the N people heard

60(0.2N) + 0(0.1N) + 20(0.35N) + 40(0.35N) = 12N + 0 + 7N + 14N = 33N

minutes, so they heard an average of 33 minutes each.

10. (A) Let x and y denote the dimensions of the four congruent rectangles. Then2x+2y = 14, so x+ y = 7. The area of the large square is (x+ y)2 = 72 = 49.

11. (D) The four vertices determine six possible diameters, namely, the four sidesand two diagonals. However, the two diagonals are diameters of the samecircle. Thus there are five circles.

12. (A) Note that N = 753211

, which has only 7 as a prime factor.

Page 28: AMC

SOLUTIONS 1998 AHSME 4

13. (E) Factor 144 into primes, 144 = 24 · 32, and notice that there are at mosttwo 6’s and no 5’s among the numbers rolled. If there are no 6’s, then theremust be two 3’s since these are the only values that can contribute 3 to theprime factorization. In this case the four 2’s in the factorization must be theresult of two 4’s in the roll. Hence the sum 3 + 3 + 4 + 4 = 14 is a possiblevalue for the sum. Next consider the case with just one 6. Then there mustbe one 3, and the three remaining 2’s must be the result of a 4 and a 2. Thus,the sum 6 + 3 + 4 + 2 = 15 is also possible. Finally, if there are two 6’s, thenthere must also be two 2’s or a 4 and a 1, with sums of 6 + 6 + 2 + 2 = 16 and6 + 6 + 4 + 1 = 17. Hence 18 is the only sum not possible.

OR

Since 5 does not divide 144 and 63 > 144, there can be no 5’s and at most two6’s. Thus the only ways the four dice can have a sum of 18 are: 4, 4, 4, 6; 2, 4, 6, 6;and 3, 3, 6, 6. Since none of these products is 144, the answer is (E).

14. (A) Because the parabola has x-intercepts of opposite sign and the y-coordinateof the vertex is negative, a must be positive, and c, which is the y-intercept,must be negative. The vertex has x-coordinate !b/2a = 4 > 0, so b must benegative.

15. (C) The regular hexagon can be partitioned into six equilateral triangles,each with area one-sixth of the original triangle. Since the original equilateraltriangle is similar to each of these, and the ratio of the areas is 6, it followsthat the ratio of the sides is

#6.

16. (B) The area of the shaded region is

!

2

'

()

a + b

2

*2

+#

a

2

&2

!)

b

2

*2+

, =!

2

a + b

2

)a + b

2+

a! b

2

*

=!(a + b)a

4

and the area of the unshaded region is

!

2

'

()

a + b

2

*2

!#

a

2

&2

+

)b

2

*2+

, =!

2

a + b

2

)a + b

2+

b! a

2

*

=!(a + b)b

4.

Their ratio is a/b.

17. (E) Note that f(x) = f(x + 0) = x + f(0) = x + 2 for any real numberx. Hence f(1998) = 2000. The function defined by f(x) = x + 2 has bothproperties: f(0) = 2 and f(x + y) = x + y + 2 = x + (y + 2) = x + f(y).

Page 29: AMC

SOLUTIONS 1998 AHSME 5

OR

Note that

2 = f(0) = f(!1998 + 1998) = !1998 + f(1998).

Hence f(1998) = 2000.

18. (A) Suppose the sphere has radius r. We can write the volumes of the threesolids as functions of r as follows:

Volume of cone = A =1

3!r2(2r) =

2

3!r3,

Volume of cylinder = M = !r2(2r) = 2!r3, and

Volume of sphere = C =4

3!r3.

Thus, A!M + C = 0.

Note. The AMC logo is designed to show this classical result of Archimedes.

19. (C) The area of the triangle is 12(base)(height) = 1

2 · (5 ! (!5)) · |5 sin "| =25| sin "|. There are four values of " between 0 and 2! for which | sin "| = 0.4,and each value corresponds to a distinct triangle with area 10.

OR

The vertex (5 cos ", 5 sin ") lies on a circle of diameter 10 centered at the origin.In order that the triangle have area 10, the altitude from that vertex must be2. There are four points on the circle that are 2 units from the x-axis.

20. (C) There are eight ordered triples of numbers satisfying the conditions:(1, 2, 10), (1, 3, 9), (1, 4, 8), (1, 5, 7), (2, 3, 8), (2, 4, 7), (2, 5, 6), and (3, 4, 6). Be-cause Casey’s card gives Casey insu"cient information, Casey must have seena 1 or a 2. Next, Tracy must not have seen a 6, 9, or 10, since each of thesewould enable Tracy to determine the other two cards. Finally, if Stacy hadseen a 3 or a 5 on the middle card, Stacy would have been able to determinethe other two cards. The only number left is 4, which leaves open the twopossible triples (1, 4, 8) and (2, 4, 7).

Page 30: AMC

SOLUTIONS 1998 AHSME 6

21. (C) Let r be Sunny’s rate. Thush

rand

h + d

rare the times it takes Sunny

to cover h meters and h + d meters, respectively. Because Windy covers onlyh! d meters while Sunny is covering h meters, it follows that Windy’s rate is(h! d)r

h. While Sunny runs h + d meters, the number of meters Windy runs

is(h! d)r

h· h + d

r= h! d2

h. Sunny’s victory margin over Windy is

d2

h.

22. (C) Express each term using a base-10 logarithm, and note that the sumequals

log 2/ log 100! + log 3/ log 100! + · · · + log 100/ log 100! = log 100!/ log 100! = 1.

OR

Since 1/logk 100! equals log100! k for all positive integers k, the expressionequals log100!(2 · 3 · · · · · 100) = log100! 100! = 1.

23. (D) Complete the squares in the two equations to bring them to the form

(x! 6)2 + (y ! 3)2 = 72 and (x! 2)2 + (y ! 6)2 = k + 40.

The graphs of these equations are circles. The first circle has radius 7, andthe distance between the centers of the circles is 5. In order for the circles tohave a point in common, therefore, the radius of the second circle must be atleast 2 and at most 12. It follows that 22 $ k + 40 $ 122, or !36 $ k $ 104.Thus b! a = 140.

24. (C) There are 10,000 ways to write the last four digits d4d5d6d7, and amongthese there are 10000 ! 10 = 9990 for which not all the digits are the same.For each of these, there are exactly two ways to adjoin the three digits d1d2d3

to obtain a memorable number. There are ten memorable numbers for whichthe last four digits are the same, for a total of 2 · 9990 + 10 = 19990.

OR

Let A denote the set of telephone numbers for which d1d2d3 and d4d5d6 areidentical and B the set for which d1d2d3 is the same as d5d6d7. A numberd1d2d3-d4d5d6d7 belongs to A%B if and only if d1 = d4 = d5 = d2 = d6 = d3 =d7. Hence, n(A %B) = 10. Thus, by the Inclusion-Exclusion Principle,

n(A &B) = n(A) + n(B)! n(A %B) = 103 · 1 · 10 + 103 · 10 · 1! 10 = 19990.

Page 31: AMC

SOLUTIONS 1998 AHSME 7

25. (B) The crease in the paper is the perpendicular bisector of the segment thatjoins (0,2) to (4,0). Thus the crease contains the midpoint (2, 1) and has slope2, so the equation y = 2x!3 describes it. The segment joining (7,3) and (m,n)

must have slope !1

2, and its midpoint

#7 + m

2,3 + n

2

&must also satisfy the

equation y = 2x! 3. It follows that

!1

2=

n! 3

m! 7and

3 + n

2= 2 · 7 + m

2! 3, so

2n + m = 13 and n! 2m = 5.

Solve these equations simultaneously to find that m = 3/5 and n = 31/5, sothat m + n = 34/5 = 6.8.

OR

As shown above, the crease is described by the equation y = 2x!3. Therefore,the slope of the line through (m, n) and (7, 3) is !1/2, so the points on the linecan be described parametrically by (x, y) = (7! 2t, 3 + t). The intersection ofthis line with the crease y = 2x! 3 is found by solving 3 + t = 2(7! 2t)! 3.This yields the parameter value t = 8/5. Since t = 8/5 determines the pointon the crease, use t = 2(8/5) to find the coordinates m = 7 ! 2(16/5) = 3/5and n = 3 + (16/5) = 31/5.

26. (B) Extend DA through A and CB through B and denote the intersection byE. Triangle ABE is a 30!-60!-90! triangle withAB = 13, so AE = 26. Triangle CDE is alsoa 30!-60!-90! triangle, from which it follows thatCD = (46 + 26)/

#3 = 24

#3. Now apply the

Pythagorean Theorem to triangle CDA to find

that AC ="

462 + (24#

3)2 = 62.

..........................

..........................

..........................

..........................

..........................

.

............. ............. ............. .....

.............................

.. ............................

............................

............................

............................

............................

............................

............................

............................

.................

........................................................

46 26

24#

3

13•

D•E

•A

•C

•B

OR

Since the opposite angles sum to a straight angle, the quadrilateral is cyclic,and AC is the diameter of the circumscribed circle. Thus AC is the diameterof the circumcircle of triangle ABD. By the Extended Law of Sines,

AC =BD

sin 120!=

BD#3/2

.

We determine BD by the Law of Cosines:

BD2 = 132 + 462 + 2 · 13 · 46 · 1

2= 2883 = 3 · 312, so BD = 31

#3.

Hence AC = 62.

Page 32: AMC

SOLUTIONS 1998 AHSME 8

27. (E) After step one, twenty 3" 3" 3 cubes remain, eight of which are cornercubes and twelve of which are edge cubes. At this stage each 3" 3" 3 cornercube contributes 27 units of area and each 3" 3" 3 edge cube contributes 36units of area. The second stage of the tunneling process takes away 3 units ofarea from each of the eight 3"3"3 corner cubes (1 for each exposed surface),but adds 24 units to the area (4 units for each of the six 1 " 1 center facialcubes removed). The twelve 3 " 3 " 3 edge cubes each lose 4 units but gain24 units. Therefore, the total surface area of the figure is

8 · (27! 3 + 24) + 12 · (36! 4 + 24) = 384 + 672 = 1056.

28. (B) Let E denote the point on BC for which AE bisects " CAD. Becausethe answer is not changed by a similarity transformation, we may assume thatAC = 2

#5 and AD = 3

#5. Apply the Pythagorean Theorem to triangle ACD

to obtain CD = 5, then apply the Angle Bisector Theorem to triangle CAD toobtain CE = 2 and ED = 3. Let x = DB. Apply the Pythagorean Theoremto triangle ACE to obtain AE =

#24, then apply the Angle Bisector Theorem

to triangle EAB to obtain AB = (x/3)#

24. Now apply the PythagoreanTheorem to triangle ABC to get

(2#

5)2 + (x + 5)2 =#

x

3

#24

&2

,

from which it follows that x = 9. Hence BD/DC = 9/5, and m + n = 14.

Page 33: AMC

SOLUTIONS 1998 AHSME 9

OR

Denote by a the measure of angle CAE. Let AC = 2u, and AD = 3u. Itfollows that CD =

#5u. We may assume BD =#

5. (Otherwise, we could simply modify the tri-angle with a similarity transformation.) Hence,the ratio CD/BD we seek is just u. Since cos 2a =2/3, we have sin a = 1/

#6. Applying the Law of

Sines in triangle ABD yields

sin D

AB=

sin a#5

=2/3

"(2u)2 + (

#5(1 + u))2

=1/#

6#5

.

Solve this for u to get

2#

5#

6 = 3"

4u2 + 5(1 + 2u + u2)

120 = 9(9u2 + 10u + 5)

0 = 27u2 + 30u! 25

0 = (9u! 5)(3u + 5)

so u = 5/9 and m + n = 14.

........................................

........................................

........................................

........................................

.....................

......................................................................................................................................................................................................................................................

............................................................................................................................................................................................................................................................................................................................................................................................................................................................. .................................................................

..........................

.................

.........

.......................................

.

.........

.........a

2a3a

•A

•B

•C

•D

•E

OR

Again, let a = " CAE. We are given that cos 2a = 2/3 and we wish to compute

CD

BD=

AC tan 2a

AC(tan 3a! tan 2a)=

#tan 3a

tan 2a! 1

&#1

.

Let y = tan a. Trigonometric identities yield (upon simplification)

#tan 3a

tan 2a! 1

&#1

=2(1! 3y2)

(1 + y2)2and

2

3= cos 2a =

1! y2

1 + y2.

Thus y2 = 1/5 andCD

BD=

2(1! 3/5)

(6/5)2=

5

9.

Alternatively, starting with a = cos#1(2/3)/2, electronic calculation yieldstan(3a)/ tan(2a) = 2.8 = 14/5, so CD/BD = 5/9.

Page 34: AMC

SOLUTIONS 1998 AHSME 10

29. (D) If a square encloses three collinear lattice points, then it is not hard tosee that the square must also enclose at least one additional lattice point. Ittherefore su"ces to consider squares that enclose only the lattice points (0,0),(0,1), and (1,0). If a square had two adjacent sides, neither of which containeda lattice point, then the square could be enlarged slightly by moving thosesides parallel to themselves. To be largest, therefore, a squaremust contain a lattice point on at least two non-adjacent sides. The desired square will thus haveparallel sides that contain (1,1) and at least one of(!1, 0) and (0,!1). The size of the square is de-termined by the separation between two parallelsides. Because the distance between parallel linesthrough (1,1) and (0,!1) can be no larger than#

5, the largest conceivable area for the square is5. To see that this is in fact possible, draw thelines of slope 2 through (!1, 0) and (1,!1), andthe lines of slope !1/2 through (1, 1) and (0,!1).These four lines can be described by the equations

............................................................................................................................................................................................................................................................................................................................................................................

............................................................................................................................................................................................................................................................................................................................................................................

............................................................................................................................................................................................................................................................................................................................................................................

............................................................................................................................................................................................................................................................................................................................................................................

••

••

y = 2x + 2, y = 2x ! 3, 2y + x = 3, and 2y + x = !2, respectively. Theyintersect to form a square whose area is 5, and whose vertices are (!1/5, 8/5),(9/5, 3/5), (4/5,!7/5), and (!6/5,!2/5). There are only three lattice pointsinside this square.

Page 35: AMC

SOLUTIONS 1998 AHSME 11

30. (E) Factor an as a product of prime powers:

an = n(n + 1)(n + 2) · · · (n + 9) = 2e13e25e3 · · · .

Among the ten factors n, n+1, . . . , n+9, five are even and their product can bewritten 25m(m+1)(m+2)(m+3)(m+4). If m is even then m(m+2)(m+4)is divisible by 16 and thus e1 ' 9. If m is odd, then e1 ' 8. If e1 > e3,then the rightmost nonzero digit of an is even. If e1 $ e3, then the rightmostnonzero digit of an is odd. Hence we seek the smallest n for which e3 ' e1.Among the ten numbers n, n + 1, . . . , n + 9, two are divisible by 5 and atmost one of these is divisible by 25. Hence e3 ' 8 if and only if one ofn, n + 1, . . . , n + 9 is divisible by 57. The smallest n for which an satisfiese3 ' 8 is thus n = 57 ! 9, but in this case the product of the five evennumbers among n, n + 1, . . . , n + 9 is 25m(m + 1)(m + 2)(m + 3)(m + 4)where m is even, namely (57 ! 9)/2 = 39058. As noted earlier, this givese1 ' 9. For n = 57 ! 8 = 78117, the product of the five even numbers amongn, n + 1, . . . , n + 9 is 25m(m + 1)(m + 2)(m + 3)(m + 4) with m = 39059.Note that in this case e1 = 8. Indeed, 39059 + 1 is divisible by 4 but notby 8, and 39059 + 3 is divisible by 2 but not by 4. Compute the rightmostnonzero digit as follows. The odd numbers among n, n + 1, . . . , n + 9 are78117, 78119, 78121, 78123, 78125 = 57 and the product of the even numbers78118, 78120, 78122, 78124, 78126 is 25 · 39059 · 39060 · 39061 · 39062 · 39063 =25 · 39059 · (22 · 5 · 1953) · 39061 · (2 · 19531) · 39063. (For convenience, wehave underlined the needed unit digits.) Having written n(n + 1) · · · (n + 9)as 2858 times a product of odd factors not divisible by 5, we determine therightmost nonzero digit by multiplying the units digits of these factors. Itfollows that, for n = 57!8, the rightmost nonzero digit of an is the units digitof 7 · 9 · 1 · 3 · 9 · 3 · 1 · 1 · 3 = (9 · 9) · (7 · 3) · (3 · 3), namely 9.

Page 36: AMC

50th AHSME 1999 2

1. 1! 2 + 3! 4 + · · ·! 98 + 99 =

(A) !50 (B) !49 (C) 0 (D) 49 (E) 50

2. Which one of the following statements is false?

(A) All equilateral triangles are congruent to each other.

(B) All equilateral triangles are convex.

(C) All equilateral triangles are equiangular.

(D) All equilateral triangles are regular polygons.

(E) All equilateral triangles are similar to each other.

3. The number halfway between 1/8 and 1/10 is

(A)1

80(B)

1

40(C)

1

18(D)

1

9(E)

9

80

4. Find the sum of all prime numbers between 1 and 100 that are simultaneously1 greater than a multiple of 4 and 1 less than a multiple of 5.

(A) 118 (B) 137 (C) 158 (D) 187 (E) 245

5. The marked price of a book was 30% less than the suggested retail price. Alicepurchased the book for half the marked price at a Fiftieth Anniversary sale.What percent of the suggested retail price did Alice pay?

(A) 25% (B) 30% (C) 35% (D) 60% (E) 65%

6. What is the sum of the digits of the decimal form of the product 21999 · 52001?

(A) 2 (B) 4 (C) 5 (D) 7 (E) 10

7. What is the largest number of acute angles that a convex hexagon can have?

(A) 2 (B) 3 (C) 4 (D) 5 (E) 6

Page 37: AMC

50th AHSME 1999 3

8. At the end of 1994 Walter was half as old as his grandmother. The sum of theyears in which they were born is 3838. How old will Walter be at the end of1999?

(A) 48 (B) 49 (C) 53 (D) 55 (E) 101

9. Before Ashley started a three-hour drive, her car’s odometer reading was29792, a palindrome. (A palindrome is a number that reads the same wayfrom left to right as it does from right to left.) At her destination, the odome-ter reading was another palindrome. If Ashley never exceeded the speed limitof 75 miles per hour, which of the following was her greatest possible averagespeed?

(A) 331

3(B) 53

1

3(C) 66

2

3(D) 70

1

3(E) 74

1

3

10. A sealed envelope contains a card with a single digit on it. Three of thefollowing statements are true, and the other is false.

I. The digit is 1.

II. The digit is not 2.

III. The digit is 3.

IV. The digit is not 4.

Which one of the following must necessarily be correct?

(A) I is true. (B) I is false. (C) II is true. (D) III is true.

(E) IV is false.

11. The student lockers at Olympic High are numbered consecutively beginningwith locker number 1. The plastic digits used to number the lockers cost twocents apiece. Thus, it costs two cents to label locker number 9 and four centsto label locker number 10. If it costs $137.94 to label all the lockers, how manylockers are there at the school?

(A) 2001 (B) 2010 (C) 2100 (D) 2726 (E) 6897

12. What is the maximum number of points of intersection of the graphs of twodi!erent fourth degree polynomial functions y = p(x) and y = q(x), each withleading coe"cient 1?

(A) 1 (B) 2 (C) 3 (D) 4 (E) 8

Page 38: AMC

50th AHSME 1999 4

13. Define a sequence of real numbers a1, a2, a3, . . . by a1 = 1 and a3n+1 = 99a3

n forall n " 1. Then a100 equals

(A) 3333 (B) 3399 (C) 9933 (D) 9999 (E) none of these

14. Four girls — Mary, Alina, Tina, and Hanna — sang songs in a concert as trios,with one girl sitting out each time. Hanna sang 7 songs, which was more thanany other girl, and Mary sang 4 songs, which was fewer than any other girl.How many songs did these trios sing?

(A) 7 (B) 8 (C) 9 (D) 10 (E) 11

15. Let x be a real number such that secx! tan x = 2. Then sec x + tanx =

(A) 0.1 (B) 0.2 (C) 0.3 (D) 0.4 (E) 0.5

16. What is the radius of a circle inscribed in a rhombus with diagonals of length10 and 24?

(A) 4 (B) 58/13 (C) 60/13 (D) 5 (E) 6

17. Let P (x) be a polynomial such that when P (x) is divided by x ! 19, theremainder is 99, and when P (x) is divided by x ! 99, the remainder is 19.What is the remainder when P (x) is divided by (x! 19)(x! 99)?

(A) !x + 80 (B) x + 80 (C) !x + 118 (D) x + 118 (E) 0

18. How many zeros does f(x) = cos(log(x)) have on the interval 0 < x < 1?

(A) 0 (B) 1 (C) 2 (D) 10 (E) infinitely many

19. Consider all triangles ABC satisfying the followingconditions: AB = AC, D is a point on AC for whichBD # AC, AD and CD are integers, and BD2 = 57.Among all such triangles, the smallest possible valueof AC is .......

....................................................................................................................................................................................................................................................................

......................................

.........................

•B

•C

•A

•D

(A) 9 (B) 10 (C) 11 (D) 12 (E) 13

Page 39: AMC

50th AHSME 1999 5

20. The sequence a1, a2, a3, . . . satisfies a1 = 19, a9 = 99, and, for all n " 3, an isthe arithmetic mean of the first n! 1 terms. Find a2.

(A) 29 (B) 59 (C) 79 (D) 99 (E) 179

21. A circle is circumscribed about a triangle with sides 20, 21, and 29, thus divid-ing the interior of the circle into four regions. Let A, B, and C be the areasof the non-triangular regions, with C being the largest. Then

(A) A + B = C (B) A + B + 210 = C (C) A2 + B2 = C2

(D) 20A + 21B = 29C (E)1

A2+

1

B2=

1

C2

22. The graphs of y = !|x ! a| + b and y = |x ! c| + d intersect at points (2, 5)and (8, 3). Find a + c.

(A) 7 (B) 8 (C) 10 (D) 13 (E) 18

23. The equiangular convex hexagon ABCDEF has AB = 1, BC = 4, CD = 2,and DE = 4. The area of the hexagon is

(A)15

2

$3 (B) 9

$3 (C) 16 (D)

39

4

$3 (E)

43

4

$3

24. Six points on a circle are given. Four of the chords joining pairs of the sixpoints are selected at random. What is the probability that the four chordsare the sides of a convex quadrilateral?

(A)1

15(B)

1

91(C)

1

273(D)

1

455(E)

1

1365

25. There are unique integers a2, a3, a4, a5, a6, a7 such that

5

7=

a2

2!+

a3

3!+

a4

4!+

a5

5!+

a6

6!+

a7

7!,

where 0 % ai < i for i = 2, 3, . . . , 7. Find a2 + a3 + a4 + a5 + a6 + a7.

(A) 8 (B) 9 (C) 10 (D) 11 (E) 12

Page 40: AMC

50th AHSME 1999 6

26. Three non-overlapping regular plane polygons, at least two of which are con-gruent, all have sides of length 1. The polygons meet at a point A in sucha way that the sum of the three interior angles at A is 360!. Thus the threepolygons form a new polygon with A as an interior point. What is the largestpossible perimeter that this polygon can have?

(A) 12 (B) 14 (C) 18 (D) 21 (E) 24

27. In triangle ABC, 3 sin A + 4 cosB = 6 and 4 sinB + 3 cosA = 1. Then " Cin degrees is

(A) 30 (B) 60 (C) 90 (D) 120 (E) 150

28. Let x1, x2, . . . , xn be a sequence of integers such that(i) !1 % xi % 2, for i = 1, 2, 3, . . . , n;(ii) x1 + x2 + · · · + xn = 19; and(iii) x2

1 + x22 + · · · + x2

n = 99.Let m and M be the minimal and maximal possible values of x3

1+x32+ · · ·+x3

n,respectively. Then M/m =

(A) 3 (B) 4 (C) 5 (D) 6 (E) 7

29. A tetrahedron with four equilateral triangular faces has a sphere inscribedwithin it and a sphere circumscribed about it. For each of the four faces, thereis a sphere tangent externally to the face at its center and to the circumscribedsphere. A point P is selected at random inside the circumscribed sphere. Theprobability that P lies inside one of the five small spheres is closest to

(A) 0 (B) 0.1 (C) 0.2 (D) 0.3 (E) 0.4

30. The number of ordered pairs of integers (m, n) for which mn " 0 and

m3 + n3 + 99mn = 333

is equal to

(A) 2 (B) 3 (C) 33 (D) 35 (E) 99

Page 41: AMC

SOLUTIONS 1996 AHSME 2

1. (D) The mistake occurs in the tens column where any of the digits of theaddends can be decreased by 1 or the 5 in the sum changed to 6 to make theaddition correct. The largest of these digits is 7.

2. (A) Walter gets an extra $2 per day for doing chores exceptionally well. Ifhe never did them exceptionally well, he would get $30 for 10 days of chores.The extra $6 must be for 3 days of exceptional work.

3. (E)(3!)!

3!=

6!

3!= 6 · 5 · 4 = 120.

OR

(3!)!

3!=

6!

6= 5! = 5 · 4 · 3 · 2 = 120.

4. (D) The largest possible median will occur when the three numbers not givenare larger than those given. Let a, b, and c denote the three missing numbers,where 9 ! a ! b ! c. Ranked from smallest to largest, the list is

3, 5, 5, 7, 8, 9, a, b, c,

so the median value is 8.

5. (E) The largest fraction is the one with largest numerator and smallest de-nominator. Choice (E) has both.

6. (E) Since 0z = 0 for any z > 0, f(0) = f("2) = 0. Since ("1)0 = 1,

f(0) + f("1)+ f("2)+ f("3) = ("1)0(1)2 +("3)!2("1)0 = 1+1

("3)2=

10

9.

7. (B) The sum of the children’s ages is 10 because $9.45 " $4.95 = $4.50 =10#$0.45. If the twins were 3 years old or younger, then the third child wouldnot be the youngest. If the twins are 4, the youngest child must be 2.

8. (D) Since 3 = k · 2r and 15 = k · 4r, we have

5 =15

3=

k · 4r

k · 2r=

22r

2r= 2r.

Thus, by definition, r = log2 5.

Page 42: AMC

SOLUTIONS 1996 AHSME 3

9. (B) Since line segment AD is perpendicular to the planeof PAB, angle PAD is a right angle. In right tri-angle PAD, PA = 3 and AD = AB = 5. By thePythagorean Theorem, PD =

$32 + 52 =

$34. The

fact that PB = 4 was not needed......................................................................................................................................................

.....................................................................................................................................................

............................

.........................................................................................................................

.........................................

..............................................................

......................................

.................................................................................................................................................................................................................................................................................................................................................................................................................................................................................................................................................................

..............................

.........................

•A•

•B

•D

•C•P3

5

10. (D) There are 12 edges, 12 face diagonals, and 4 space diagonals for a totalof 12 + 12 + 4 = 28.

OR

Each pair of vertices of the cube determines a line segment. There are!

82

"=

8!(8!2)!2! = 8·7

2 = 28 such pairs.

11. (D) The endpoints of each of these line segments are atdistance

$22 + 12 =

$5 from the center of the circle. The

region is therefore an annulus with inner radius 2 and outerradius

$5. The area covered is !(

$5)2 " !(2)2 = !.

........

................................................

...........................

..................................................................................................................................................................................................................................................................................................................

................................................................................... .................................................................

.............................

...................................................................................................................................................................................................................................................................................................................................................

..............................................................................................

..

.!

5 2

....................................................................................

..

..

..

..

.

..

..

..

..

..

..

.................................................

..............................................................

..

..

..

..

.

..

..

..

..

..

..

..

..

..

..

..

.

..

..

.

..

....................

........................................................

...............................

Note. The area of the annular region covered by the segments of length 2does not depend on the radius of the circle.

12. (B) Since k is odd, f(k) = k + 3. Since k + 3 is even,

f(f(k)) = f(k+3) = (k + 3)/2.

If (k + 3)/2 is odd, then

27 = f(f(f(k))) = f((k+3)/2) = (k + 3)/2 + 3,

which implies that k = 45. This is not possible because f(f(f(45))) =f(f(48)) = f(24) = 12. Hence (k + 3)/2 must be even, and

27 = f(f(f(k))) = f((k+3)/2) = (k + 3)/4,

which implies that k = 105. Checking, we find that

f(f(f(105))) = f(f(108)) = f(54) = 27.

Hence the sum of the digits of k is 1 + 0 + 5 = 6.

Page 43: AMC

SOLUTIONS 1996 AHSME 4

13. (D) Let x be the number of meters that Moonbeam runs to overtake Sunny,and let r and mr be the rates of Sunny and Moonbeam, respectively. BecauseSunny runs x" h meters in the same time that Moonbeam runs x meters, it

follows thatx" h

r=

x

mr. Solving for x, we get x =

hm

m" 1.

14. (C) Since E(100) = E(00), the result is the same as E(00)+E(01)+E(02)+E(03) + · · · + E(99), which is the same as

E(00010203 . . . 99).

There are 200 digits, and each digit occurs 20 times, so the sum of the evendigits is 20(0 + 2 + 4 + 6 + 8) = 20(20) = 400.

15. (B) Let the base of the rectangle be b and the height a. Triangle A has analtitude of length b/2 to a base of length a/n, andtriangle B has an altitude of length a/2 to a baseof length b/m. Thus the required ratio of areas is

1

2· a

n· b

21

2· b

m· a

2

=m

n.

................................................................................................................................................................................

............................................................................................................................................................................................................................

bm

.........................

............................................................................

........................................................................................................

b

..................................................

an

.......................................................................................................................................................................................

a

16. (D) There are 15 ways in which the third outcome is the sum of the first twooutcomes.

(1,1,2) (2,1,3) (3, 1, 4) (4, 1, 5) (5, 1, 6)

(1,2,3) (2,2,4) (3,2,5) (4,2,6)

(1, 3, 4) (2,3,5) (3, 3, 6)

(1, 4, 5) (2,4,6)

(1, 5, 6)

Since the three tosses are independent, all of the 15 possible outcomes areequally likely. At least one “2” appears in exactly eight of these outcomes, sothe required probability is 8/15.

17. (E) In the 30"-60"-90" triangle CEB, BC = 6$

3. Therefore, FD = AD "AF = 6

$3" 2. In the 30"-60"-90" triangle CFD, CD = FD

$3 = 18" 2

$3.

The area of rectangle ABCD is

(BC)(CD) =!6$

3" !

18" 2$

3"

= 108$

3" 36 % 151.

Page 44: AMC

SOLUTIONS 1996 AHSME 5

18. (D) Let D and F denote the centers of the circles. Let C and B be the pointswhere the x-axis and y-axis intersect the tangent line, respectively. Let E andG denote the points of tangency as shown. We know that AD = DE = 2,DF = 3, and FG = 1. Let FC = u andAB = y. Triangles FGC and DEC aresimilar, so

u

1=

u + 3

2,

which yields u = 3. Hence, GC =$

8.Also, triangles BAC and FGC are similar,which yields

y

1=

BA

FG=

AC

GC=

8$8

=$

8 = 2$

2.

.......................................................................................................................................................................................................................................................................................................................................................................................

...........................

......................................................................................................................................................... ..........................................

........................................................................................................................................................

......................................

..........................

..........................................................•A

•D

•F

•C

•B•E

•G

19. (D) Let R and S be the vertices of the smaller hexagon adjacent to vertex Eof the larger hexagon, and let O be the center of the hexagons. Then, since# ROS = 60", quadrilateral ORES encloses 1/6 of the area of ABCDEF ,&ORS encloses 1/6 of the area of the smaller hexagon, and&ORS is equilat-eral. Let T be the center of &ORS. Then triangles TOR, TRS, and TSO arecongruent isosceles triangles with largest angle120". Triangle ERS is an isosceles triangle withlargest angle 120" and a side in common with&TRS, so ORES is partitioned into four congru-ent triangles, exactly three of which form &ORS.Since the ratio of the area enclosed by the smallregular hexagon to the area of ABCDEF is thesame as the ratio of the area enclosed by &ORSto the area enclosed by ORES, the ratio is 3/4.

•O

•R

•F

•D

•E

•S •T

.................................................................................................

................................

................................

................................

..................................................................................................................................................................................................

..................................................................................

................................

................................

................................

................................

.............

OR

Let M and N denote the midpoints of AB andAF , respectively. Then MN = AM

$3 since

&AMO is a 30"-60"-90" triangle and MN = MO.It follows that the hexagons are similar, with sim-ilarity ratio 1

2

$3. Thus the desired quotient is

(12

$3)2 = 3

4 .

•O

•M•A •B

•C

•D

•E

•F

•N..................................................................................................................................................................................................

.................................................................................................................................................................................................

...............................................................................

...............................................................................................................

................................

...............

................................

................................

...............

..

..

..

..

..

..

..

..

................

Page 45: AMC

SOLUTIONS 1996 AHSME 6

20. (C) Let O = (0, 0), P = (6, 8), and Q = (12, 16). As shown in the figure,the shortest route consists of tangent OT , minor arc TR,and tangent RQ. Since OP = 10, PT = 5, and # OTPis a right angle, it follows that # OPT = 60" and OT =5$

3. By similar reasoning, # QPR = 60" and QR = 5$

3.Because O, P, and Q are collinear (why?), # RPT = 60",so arc TR is of length 5!/3. Hence the length of the

shortest route is 2!5$

3"

+5!

3. .

.

.. ...........................

.....................

...............................

...........................................................................................................................................................................................................................................................................................................................

T

O

PR

Q

..........................................

..........................................

.........................

............................................................................................................

21. (D) Let # ABD = x and # BAC = y. Since the triangles

ABC and ABD are isosceles, # C = (180" " y)/2 and# D = (180" " x)/2. Then, noting that x + y = 90",we have

# C + # D = (360" " (x + y))/2 = 135".

•A

•B

•C

•D•E

y

x

.....................................................................................................................................................................................................................................................................................................................................................................................................................................................................................................................................................................................................................................

.................................

OR

Consider the interior angles of pentagon ADECB. Since triangles ABC andABD are isosceles, # C = # B and # D = # A. Since BD ' AC, the interiorangle at E measures 270". Since 540" is the sumof the interior angles of any pentagon,

# A + # B + # C + # D + # E

= 2# C + 2# D + 270" = 540",

from which it follows that # C + # D = 135".

•A

•B

•C

•D•E

..........................................................................................................................................................................................................................................................................................................................................................................................................................................................

......................

............

.........

.............................

22. (B) Because all quadruples are equally likely, we need only examine the sixclockwise orderings of the points: ACBD, ADBC, ABCD,ADCB, ABDC,and ACDB. Only the first two of these equally likely orderings satisfy theintersection condition, so the probability is 2/6 = 1/3.

Page 46: AMC

SOLUTIONS 1996 AHSME 7

23. (B) Let a, b, and c be the dimensions of the box. It is given that

140 = 4a + 4b + 4c and 21 =$

a2 + b2 + c2,

hence

35 = a + b + c (1) and 441 = a2 + b2 + c2 (2).

Square both sides of (1) and combine with (2) to obtain

1225 = (a + b + c)2

= a2 + b2 + c2 + 2ab + 2bc + 2ca

= 441 + 2ab + 2bc + 2ca.

Thus the surface area of the box is 2ab + 2bc + 2ca = 1225" 441 = 784.

24. (B) The kth 1 is at position

1 + 2 + 3 + · · · + k =k(k + 1)

2

and49(50)

2< 1234 <

50(51)

2, so there are 49 1’s among the first 1234 terms.

All the other terms are 2’s, so the sum is 1234(2)" 49 = 2419.

OR

The sum of all the terms through the occurrence of the kth 1 is

1 + (2 + 1) + (2 + 2 + 1) + · · · + (2 + 2 + · · · + 2# $% &k!1

+1)

= 1 + 3 + 5 + · · · + (2k " 1)

= k2.

The kth 1 is at position

1 + 2 + 3 + · · · + k =k(k + 1)

2.

It follows that the last 1 among the first 1234 terms of the sequence occurs atposition 1225 for k = 49. Thus, the sum of the first 1225 terms is 492 = 2401,and the sum of the next nine terms, all of which are 2’s, is 18, for a total of2401 + 18 = 2419.

Page 47: AMC

SOLUTIONS 1996 AHSME 8

25. (B) The equation x2 + y2 = 14x + 6y + 6 can be written

(x" 7)2 + (y " 3)2 = 82,

which defines a circle of radius 8 centered at (7, 3). If k is a possible value of3x + 4y for (x, y) on the circle, then the line 3x + 4y = k must intersect thecircle in at least one point. The largest value of k occurswhen the line is tangent to the circle, and is thereforeperpendicular to the radius at the point of tangency. Be-cause the slope of the tangent line is "3/4, the slope ofthe radius is 4/3. It follows that the point on the circlethat yields the maximum value of 3x + 4y is one of thetwo points of tangency,

x = 7 +3 · 8

5=

59

5, y = 3 +

4 · 8

5=

47

5,

or

x = 7" 3 · 8

5=

11

5, y = 3" 4 · 8

5= "17

5.

........

.........

..........................................................

............................

.................................................................................................................................................................................................................................................................................................................................................................................................

...............................................................................................................

...........................................................................................................................................................................................................................................................................

...........................................................................................................................................................................................................................................................

...........................................................................................................................................................................................................................................

•(7, 3)

The first point of tangency gives

3x + 4y = 3 · 59

5+ 4 · 47

5=

177

5+

188

5= 73,

and the second one gives 3x + 4y = 335 "

685 = "7. Thus 73 is the desired

maximum, while "7 is the minimum.

OR

Suppose that k = 3x + 4y is a possible value. Substituting y = (k " 3x)/4into x2 +y2 = 14x+6y+6, we get 16x2 +(k"3x)2 = 224x+24(k"3x)+96,which simplifies to

25x2 " 2(3k + 76)x + (k2 " 24k " 96) = 0. (1)

If the line 3x + 4y = k intersects the given circle, the discriminant of (1)must be nonnegative. Thus we get (3k + 76)2" 25(k2" 24k" 96) ( 0, whichsimplifies to

(k " 73)(k + 7) ! 0.

Hence "7 ! k ! 73.

Page 48: AMC

SOLUTIONS 1996 AHSME 9

26. (B) The hypothesis of equally likely events can be expressed as

'r

4

(

'n

4

( =

'r

3

('w

1

(

'n

4

( =

'r

2

('w

1

('b

1

(

'n

4

( =

'r

1

('w

1

('b

1

('g

1

(

'n

4

(

where r, w, b, and g denote the number of red, white, blue, and green marbles,respectively, and n = r + w + b + g. Eliminating common terms and solvingfor r in terms of w, b, and g, we get

r " 3 = 4w, r " 2 = 3b, and r " 1 = 2g.

The smallest r for which w, b, and g are all positive integers is r = 11, withcorresponding values w = 2, b = 3, and g = 5. So the smallest total numberof marbles is 11 + 2 + 3 + 5 = 21.

27. (D) From the description of the first ball we find that z ( 9/2, and fromthat of the second, z ! 11/2. Because z must be an integer, the only possiblelattice points in the intersection are of the form (x, y, 5). Substitute z = 5into the inequalities defining the balls:

x2 + y2 +)z " 21

2

*2

! 62 and x2 + y2 + (z " 1)2 !)

9

2

*2

.

These yield

x2 + y2 +)"11

2

*2

! 62 and x2 + y2 + (4)2 !)

9

2

*2

,

which reduce to

x2 + y2 ! 23

4and x2 + y2 ! 17

4.

If (x, y, 5) satisfies the second inequality, then it must satisfy the first one.The only remaining task is to count the lattice points that satisfy the secondinequality. There are 13:

("2, 0, 5), (2, 0, 5), (0,"2, 5), (0, 2, 5), ("1,"1, 5),(1,"1, 5), ("1, 1, 5), (1, 1, 5), ("1, 0, 5), (1, 0, 5),(0,"1, 5), (0, 1, 5), and (0, 0, 5).

Page 49: AMC

SOLUTIONS 1996 AHSME 10

28. (C) Let h be the required distance. Find the volume of pyramid ABCD asa third of the area of a triangular base times the altitude to that base in twodi!erent ways, and equate these volumes. Use the altitude AD to &BCD tofind that the volume is 8. Next, note that h is thelength of the altitude of the pyramid from D to&ABC. Since the sides of &ABC are 5, 5, and4$

2, by the Pythagorean Theorem the altitude tothe side of length 4

$2 is a =

$17. Thus, the area

of&ABC is 2$

34, and the volume of the pyramidis 2$

34h/3. Equating the volumes yields

2$

34h/3 = 8, and thus h = 12/$

34 % 2.1.

5a

5

2$

2 2$

2

A

B C

..............................................................................................................................................................................................................................................................................................................................................................................................................................................................................................................................................................

OR

Imagine the parallelepiped embedded in a coordinate system as shown in thediagram. The equation for the plane (in intercept form) is x

4 + y4 + z

3 = 1.Thus, it can be expressed as 3x + 3y + 4z " 12 =0. The formula for the distance d from a point(a, b, c) to the plane Rx + Sy + Tz + U = 0 isgiven by

d =|Ra + Sb + Tc + U |$

R2 + S2 + T 2,

which in this case is

|"12|$32 + 32 + 42

=12$34% 2.1. .................

..................................

..................................

..................................

..................................

.......................

.......................................................

..........................y

.....................................................................................................................................x

........

........

........

............................

..........................

z

•A(0, 0, 3)

•B(4, 0, 0)

•C(0, 4, 0)

•D(0, 0, 0)

.....................................................................................................................................................................................................................................................

29. (C) Let 2e13e25e3 · · · be the prime factorization of n. Then the number ofpositive divisors of n is (e1 + 1)(e2 + 1)(e3 + 1) · · · . In view of the giveninformation, we have

28 = (e1 + 2)(e2 + 1)P

and30 = (e1 + 1)(e2 + 2)P,

where P = (e3+1)(e4+1) · · · . Subtracting the first equation from the second,we obtain 2 = (e1 " e2)P, so either e1 " e2 = 1 and P = 2, or e1 " e2 = 2and P = 1. The first case yields 14 = (e1 + 2)e1 and (e1 + 1)2 = 15; since e1

is a nonnegative integer, this is impossible. In the second case, e2 = e1 " 2and 30 = (e1 + 1)e1, from which we find e1 = 5 and e2 = 3. Thus n = 2533,so 6n = 2634 has (6 + 1)(4 + 1) = 35 positive divisors.

Page 50: AMC

SOLUTIONS 1996 AHSME 11

30. (E) In hexagon ABCDEF , let AB = BC = CD = 3 and let DE = EF =FA = 5. Since arc BAF is one third of the circumference of the circle, itfollows that # BCF = # BEF = 60". Similarly, # CBE = # CFE = 60". Let Pbe the intersection of BE and CF , Q that of BE and AD, and R that of CFand AD. Triangles EFP and BCP are equilateral, and by symmetry, trianglePQR is isosceles and thus also equilateral.Furthermore, # BAD and # BED subtendthe same arc, as do # ABE and # ADE.Hence triangles ABQ and EDQ are simi-lar. Therefore,

AQ

EQ=

BQ

DQ=

AB

ED=

3

5.

It follows that

AD!PQ2

PQ + 5=

3

5and

3" PQAD+PQ

2

=3

5.

•A

•B •C

•D

•E

•F

5 5

5

333

............................................................................................................. ...................................................................................................................................................................................................................................................................

....................................................................................................................................

..........................................................................................................................................................................................................................................................

.......................................................................................................................................................................................................................................................... ..........................................................................................................................................................................................................................................................

........

.........

............................................................................

.........................

...................................

....................................................................................................................................................................................................................................................................................................................................................................................................................................................................................................................

.....................................................................................................................................

Solving the two equations simultaneously yields AD = 360/49, so m+n = 409.

OR

In hexagon ABCDEF, let AB = BC = CD = a and let DE = EF = FA =b. Let O denote the center of the circle, and let r denote the radius. Since thearc BAF is one-third of the circle, it follows that # BAF = # FOB = 120". Byusing the Law of Cosines to compute BF two ways, we have a2+ab+b2 = 3r2.Let # AOB = 2". Then a = 2r sin ", and

AD = 2r sin(3")

= 2r sin " · (3" 4 sin2 ")

= a

'

3" a2

r2

(

= 3a

'

1" a2

a2 + ab + b2

(

=3ab(a + b)

a2 + ab + b2.

Substituting a = 3 and b = 5, we get AD = 360/49, so m + n = 409.

Page 51: AMC

SOLUTIONS 1996 AHSME 12

OR

In hexagon ABCDEF , let AB = BC = CD = 3 and minor arcs AB, BC,and CD each be x", let DE = EF = FA = 5 and minor arcs DE, EF , andFA each be y". Then

3x" + 3y" = 360", so x" + y" = 120".

Therefore, # BAF = 120", so BF 2 = 32 + 52 " 2 · 3 · 5 cos 120" = 49 by theLaw of Cosines, so BF = 7. Similarly, CE = 7. Using Ptolemy’s Theoremin quadrilateral BCEF , we have

BE · CF = CF 2 = 15 + 49 = 64 so CF = 8.

Using Ptolemy’s Theorem in quadrilateral ABCF , we find AC = 39/7. Fi-nally, using Ptolemy’s Theorem in quadrilateral ABCD, we have AC2 =3(AD) + 9 and, since AC = 39/7, we have AD = 360/49 and m + n = 409.

Note. Ptolemy’s Theorem: If a quadrilateral is inscribed in a circle, theproduct of the diagonals equals the sum of the products of the opposite sides.

Page 52: AMC

49th AHSME 1998 2

1. EDCBA

70

92

48

75

52

38

36

10

69

41

Each of the sides of five congruent rectangles is labeled with an integer, asshown above. These five rectangles are placed, without rotating or reflecting,in positions I through V so that the labels on coincident sides are equal.

VIV

IIIIII

Which of the rectangles is in position I?

(A) A (B) B (C) C (D) D (E) E

2. Letters A, B, C, and D represent four di!erent digits selected from 0, 1, 2, . . . , 9.If (A + B)/(C + D) is an integer that is as large as possible, what is the valueof A + B?

(A) 13 (B) 14 (C) 15 (D) 16 (E) 17

3. If a,b, and c are digits for which

7 a 2! 4 8 b

c 7 3

then a + b + c =

(A) 14 (B) 15 (C) 16 (D) 17 (E) 18

4. Define [a, b, c] to meana + b

c, where c "= 0. What is the value of

[[60, 30, 90], [2, 1, 3], [10, 5, 15]]?

(A) 0 (B) 0.5 (C) 1 (D) 1.5 (E) 2

5. If 21998 ! 21997 ! 21996 + 21995 = k · 21995, what is the value of k?

(A) 1 (B) 2 (C) 3 (D) 4 (E) 5

Page 53: AMC

49th AHSME 1998 3

6. If 1998 is written as a product of two positive integers whose di!erence is assmall as possible, then the di!erence is

(A) 8 (B) 15 (C) 17 (D) 47 (E) 93

7. If N > 1, then3

!

N3"

N 3#

N =

(A) N127 (B) N

19 (C) N

13 (D) N

1327 (E) N

8. A square with sides of length 1 is divided into twocongruent trapezoids and a pentagon, which haveequal areas, by joining the center of the square withpoints on three of the sides, as shown. Find x, thelength of the longer parallel side of each trapezoid.

..............................................................................................................................................................................• •

••

• x

(A)3

5(B)

2

3(C)

3

4(D)

5

6(E)

7

8

9. A speaker talked for sixty minutes to a full auditorium. Twenty percent of theaudience heard the entire talk and ten percent slept through the entire talk.Half of the remainder heard one third of the talk and the other half heard twothirds of the talk. What was the average number of minutes of the talk heardby members of the audience?

(A) 24 (B) 27 (C) 30 (D) 33 (E) 36

10. A large square is divided into a small square sur-rounded by four congruent rectangles as shown.The perimeter of each of the congruent rectanglesis 14. What is the area of the large square?

(A) 49 (B) 64 (C) 100 (D) 121 (E) 196

11. Let R be a rectangle. How many circles in the plane of R have a diameterboth of whose endpoints are vertices of R?

(A) 1 (B) 2 (C) 4 (D) 5 (E) 6

12. How many di!erent prime numbers are factors of N if

log2(log3(log5(log7 N))) = 11?

(A) 1 (B) 2 (C) 3 (D) 4 (E) 7

Page 54: AMC

49th AHSME 1998 4

13. Walter rolls four standard six-sided dice and finds that the product of thenumbers on the upper faces is 144. Which of the following could not be thesum of the upper four faces?

(A) 14 (B) 15 (C) 16 (D) 17 (E) 18

14. A parabola has vertex at (4,!5) and has two x-intercepts, one positive andone negative. If this parabola is the graph of y = ax2 + bx + c, which of a, b,and c must be positive?

(A) only a (B) only b (C) only c (D) a and b only (E) none

15. A regular hexagon and an equilateral triangle have equal areas. What is theratio of the length of a side of the triangle to the length of a side of thehexagon?

(A)#

3 (B) 2 (C)#

6 (D) 3 (E) 6

16. The figure shown is the union of a circle and twosemicircles of diameters a and b, all of whose cen-ters are collinear. The ratio of the area of theshaded region to that of the unshaded region is

........

.........................................................

...........................

...................................................................................................................................................................................................................................................................................................................................................................

......................................................................................................................................................................

....................... .........

.......................................

......................................................................................................................................a b!$%!$%...

.

.

.

.

.

.

.

.

.

.

.

.

.

.

.

.

.

.

.

.

.

.

.

.

.

.

.

.

.

.

.

.

.

.

.

.

.

.

.

.

.

.

.

.

.

.

.

.

.

.

.

.

.

.

.

.

.

.

.

.

.

.

.

.

.

.

.

.

.

.

.

.

.

.

.

.

.

.

.

.

.

.

.

.

.

.

.

.

.

.

.

.

.

.

.

.

.

.

.

.

.

.

.

.

.

.

.

.

.

.

.

.

.

.

.

.

.

.

.

.

.

.

.

.

.

.

.

.

.

.

.

.

.

.

.

.

.

.

.

.

.

.

.

.

.

.

.

.

.

.

.

.

.

.

.

.

.

.

.

.

.

.

.

.

.

.

.

.

.

.

.

.

.

.

.

.

.

.

.

.

.

.

.

.

.

.

.

.

.

.

.

.

.

.

.

.

.

.

.

.

.

.

.

.

.

.

.

.

.

.

.

.

.

.

.

.

.

.

.

.

.

.

.

.

.

.

.

.

.

.

.

.

.

.

.

.

.

.

.

.

.

.

.

.

.

.

.

.

.

.

....

.

.

.....

.

.

.

.

.

....

.

.

....

.

.

............

.

..............

..

(A)!

a

b(B)

a

b(C)

a2

b2(D)

a + b

2b(E)

a2 + 2ab

b2 + 2ab

17. Let f(x) be a function with the two properties:

(a) for any two real numbers x and y, f(x + y) = x + f(y), and

(b) f(0) = 2.

What is the value of f(1998)?

(A) 0 (B) 2 (C) 1996 (D) 1998 (E) 2000

18. A right circular cone of volume A, a right circular cylinder of volume M , anda sphere of volume C all have the same radius, and the common height of thecone and the cylinder is equal to the diameter of the sphere. Then

(A) A!M + C = 0 (B) A + M = C (C) 2A = M + C

(D) A2 !M2 + C2 = 0 (E) 2A + 2M = 3C

19. How many triangles have area 10 and vertices at (!5, 0), (5, 0), and (5 cos !, 5 sin !)for some angle !?

(A) 0 (B) 2 (C) 4 (D) 6 (E) 8

Page 55: AMC

49th AHSME 1998 5

20. Three cards, each with a positive integer written on it, are lying face-down ona table. Casey, Stacy, and Tracy are told that

(a) the numbers are all di!erent,

(b) they sum to 13, and

(c) they are in increasing order, left to right.

First, Casey looks at the number on the leftmost card and says, “I don’t haveenough information to determine the other two numbers.” Then Tracy looks atthe number on the rightmost card and says, “I don’t have enough informationto determine the other two numbers.” Finally, Stacy looks at the numberon the middle card and says, “I don’t have enough information to determinethe other two numbers.” Assume that each person knows that the other tworeason perfectly and hears their comments. What number is on the middlecard?

(A) 2 (B) 3 (C) 4 (D) 5

(E) There is not enough information to determine the number.

21. In an h-meter race, Sunny is exactly d meters ahead of Windy when Sunnyfinishes the race. The next time they race, Sunny sportingly starts d metersbehind Windy, who is at the starting line. Both runners run at the sameconstant speed as they did in the first race. How many meters ahead is Sunnywhen Sunny finishes the second race?

(A)d

h(B) 0 (C)

d2

h(D)

h2

d(E)

d2

h! d

22. What is the value of the expression

1

log2 100!+

1

log3 100!+

1

log4 100!+ · · · +

1

log100 100!?

(A) 0.01 (B) 0.1 (C) 1 (D) 2 (E) 10

23. The graphs of x2 + y2 = 4 + 12x + 6y and x2 + y2 = k + 4x + 12y intersectwhen k satisfies a & k & b, and for no other values of k. Find b! a.

(A) 5 (B) 68 (C) 104 (D) 140 (E) 144

24. Call a 7-digit telephone number d1d2d3-d4d5d6d7 memorable if the prefix se-quence d1d2d3 is exactly the same as either of the sequences d4d5d6 or d5d6d7

(possibly both). Assuming that each di can be any of the ten decimal digits0, 1, 2, . . . 9, the number of di!erent memorable telephone numbers is

(A) 19,810 (B) 19,910 (C) 19,990 (D) 20,000 (E) 20,100

Page 56: AMC

49th AHSME 1998 6

25. A piece of graph paper is folded once so that (0,2) is matched with (4,0), and(7,3) is matched with (m,n). Find m + n.

(A) 6.7 (B) 6.8 (C) 6.9 (D) 7.0 (E) 8.0

26. In quadrilateral ABCD, it is given that ! A = 120", angles B and D are rightangles, AB = 13, and AD = 46. Then AC =

(A) 60 (B) 62 (C) 64 (D) 65 (E) 72

27. A 9' 9' 9 cube is composed of twenty-seven 3' 3' 3 cubes. The big cube is‘tunneled’ as follows: First, the six 3'3'3 cubeswhich make up the center of each face as well asthe center 3 ' 3 ' 3 cube are removed as shown.Second, each of the twenty remaining 3 ' 3 ' 3cubes is diminished in the same way. That is, thecenter facial unit cubes as well as each center cubeare removed.The surface area of the final figure is

............................................

..

............................................

..............................................

........

........

........

........

........

........

...................................................

........

........

.........................

.............................................................................................................................................

.......................

.................

.....................................

............................

...................................................................................

................................................................................

...

...................................................

..................................................

...........

..........................................

..

.........................................

..........

........

........

........

........

........

.....................................................................................................................................................

....

....................................

..

.....................................

........

........

.....

..........................................................................................................................

............................................................................................

........

........

........

........

........

........

.....................................................................

........................................................................................................

........

........

........

........

........

........

.. ................................................................................

........

........

........

........

........

........

.... ...................................................................................

.....................................................

.........................................................................

...........................................

...........................................

...............................................................................

.....................................

........

........

........

........

........

........

..

..

.............................................

..

.................................................................................................

..................................

...

.............................................................................................

..................................

...................................................

..............................................................................

.....

..................................................

.........................................

..........

........

........

........

........

.......

.............................................................................

..

..

..

..

..

..

..

..

..

..

..

.

..

..

..

..

..

..

..

..

..

..

..

..

..

..

..

..

..

..

..

..

...........................................

..

..

..

..

..

..

..

..

.

..

..

..

..

..

..

..

..

..

..

..

..

.

..

..

..

..

..

..

..

..

..

..

..

..

.

..

..

..

..

..

..

..

..

..

..

..

..

..

..

..

..

..

..

..

..

(A) 384 (B) 729 (C) 864 (D) 1024 (E) 1056

28. In triangle ABC, angle C is a right angle and CB > CA. Point D is locatedon BC so that angle CAD is twice angle DAB. If AC/AD = 2/3, thenCD/BD = m/n, where m and n are relatively prime positive integers. Findm + n.

(A) 10 (B) 14 (C) 18 (D) 22 (E) 26

29. A point (x, y) in the plane is called a lattice point if both x and y are integers.The area of the largest square that contains exactly three lattice points in itsinterior is closest to

(A) 4.0 (B) 4.2 (C) 4.5 (D) 5.0 (E) 5.6

30. For each positive integer n, let

an =(n + 9)!

(n! 1)!

Let k denote the smallest positive integer for which the rightmost nonzerodigit of ak is odd. The rightmost nonzero digit of ak is

(A) 1 (B) 3 (C) 5 (D) 7 (E) 9

Page 57: AMC

2 SOLUTIONS 1999 AHSME

1. (E) Pairing the first two terms, the next two terms, etc. yields

1! 2 + 3! 4 + · · ·! 98 + 99 =

(1! 2) + (3! 4) + · · · + (97! 98) + 99 =

!1! 1! 1! · · ·! 1 + 99 = 50,

since there are 49 of the !1’s.

OR

1! 2 + 3! 4 + · · ·! 98 + 99 =

1 + [(!2 + 3) + (!4 + 5) + · · · + (!98 + 99)] =

1 + [1 + 1 + · · · + 1] = 1 + 49 = 50.

2. (A) Triangles with side lengths of 1, 1, 1 and 2, 2, 2 are equilateral and notcongruent, so (A) is false. Statement (B) is true since all triangles are convex.Statements (C) and (E) are true since each interior angle of an equilateraltriangle measures 60!. Furthermore, all three sides of an equilateral trianglehave the same length, so (D) is also true.

3. (E) The desired number is the arithmetic average or mean:

1

2

!1

8+

1

10

"=

1

2· 18

80=

9

80.

4. (A) A number one less than a multiple of 5 is has a units digit of 4 or 9.A number whose units digit is 4 cannot be one greater than a multiple of 4.Thus, it is su!cient to examine the numbers of the form 10d + 9 where d isone of the ten digits. Of these, only 9, 29, 49, 69 and 89 are one greater than amultiple of 4. Among these, only 29 and 89 are prime and their sum is 118.

5. (C) If the suggested retail price was P , then the marked price was 0.7P . Halfof this is 0.35P , so Alice paid 35% of the suggested retail price.

6. (D) Note that

21999 · 52001 = 21999 · 51999 · 52 = 101999 · 25 = 25

1999 zeros# $% &0 . . . 0 .

Hence the sum of the digits is 7.

Page 58: AMC

SOLUTIONS 1999 AHSME 3

7. (B) The sum of the angles in a convex hexagonis 720! and each angle must be less than 180!.If four of the angles are acute, then their sumwould be less than 360!, and therefore at leastone of the two remaining angles would be greaterthan 180!, a contradiction. Thus there can be atmost three acute angles. The hexagon shown hasthree acute angles, A, C, and E.

......................................................................................................................................................

.......................................................................................

..................

..................

..................

.....................................................................................................................................................................•

A•C

•E

•B

•D•F

OR

The result holds for any convex n-gon. The sum of the exterior angles of aconvex n-gon is 360!. Hence at most three of these angles can be obtuse, forotherwise the sum would exceed 360!. Thus the largest number of acute anglesin any convex n-gon is three.

8. (D) Let w and 2w denote the ages of Walter and his grandmother, respectively,at the end of 1994. Then their respective years of birth are 1994 ! w and1994!2w. Hence (1994!w)+(1994!2w) = 3838, and it follows that w = 50and Walter’s age at the end of 1999 will be 55.

9. (D) The next palindromes after 29792 are 29892, 29992, 30003, and 30103.The di"erence 30103! 29792 = 311 is too far to drive in three hours withoutexceeding the speed limit of 75 miles per hour. Ashley could have driven30003! 29792 = 211 miles during the three hours for an average speed of 701

3miles per hour.

10. (C) Since both I and III cannot be false, the digit must be 1 or 3. So either I orIII is the false statement. Thus II and IV must be true and (C) is necessarilycorrect. For the same reason, (E) must be incorrect. If the digit is 1, (B) and(D) are incorrect, and if the digit is 3, (A) is incorrect.

11. (A) The locker labeling requires 137.94/0.02 = 6897 digits. Lockers 1 through9 require 9 digits, lockers 10 through 99 require 2 ·90 = 180 digits, and lockers100 through 999 require 3 · 900 = 2700 digits. Hence the remaining lockersrequire 6897! 2700! 180! 9 = 4008 digits, so there must be 4008/4 = 1002more lockers, each using four digits. In all, there are 1002+999 = 2001 studentlockers.

Page 59: AMC

4 SOLUTIONS 1999 AHSME

12. (C) The x-coordinates of the intersection points are precisely the zeros ofthe polynomial p(x) ! q(x). This polynomial has degree at most three, so ithas at most three zeros. Hence, the graphs of the fourth degree polynomialfunctions intersect at most three times. Finding an example to show that threeintersection points can be achieved is left to the reader.

13. (C) Since an+1 = 3"

99 · an for all n # 1, it follows that a1, a2, a3, . . . is ageometric sequence whose first term is 1 and whose common ratio is r = 3

"99.

Thusa100 = a1 · r100"1 =

'3"

99(99

= 9933.

14. (A) Tina and Alina each sang either 5 or 6 times. If N denotes the number ofsongs sung by trios, then 3N = 4 + 5 + 5 + 7 = 21 or 3N = 4 + 5 + 6 + 7 = 22or 3N = 4 + 6 + 6 + 7 = 23. Since the girls sang as trios, the total must bea multiple of 3. Only 21 qualifies. Therefore, N = 21/3 = 7 is the number ofsongs the trios sang.

Challenge. Devise a schedule for the four girls so that each one sings therequired number of songs.

15. (E) From the identity 1 + tan2 x = sec2 x it follows that 1 = sec2 x! tan2 x =(sec x! tan x)(sec x + tanx) = 2(secx + tanx), so sec x + tanx = 0.5.

OR

The given relation can be written as1! sin x

cos x= 2. Squaring both sides yields

(1! sin x)2

1! sin2 x= 4, hence

1! sin x

1 + sin x= 4. It follows that sinx = !3

5and that

cos x =1! sin x

2=

1! (!3/5)

2=

4

5.

Thus sec x + tanx = 54 !

34 = 0.5.

Page 60: AMC

SOLUTIONS 1999 AHSME 5

16. (C) Let E be the intersection of the di-agonals of a rhombus ABCD satisfyingthe conditions of the problem. Becausethese diagonals are perpendicular and bi-sect each other, $ABE is a right trian-gle with sides 5, 12, and 13 and area 30.Therefore the altitude drawn to side AB is60/13, which is the radius of the inscribedcircle centered at E.

...............................................................................................................................................................................................................................................................................................................................

..................................

..................................

..................................

..............................................................................

.............................................................................................

...................................................................................

.......................

........................................................................................................................................

•A

•B

•E

•C•D

17. (C) From the hypothesis, P (19) = 99 and P (99) = 19. Let

P (x) = (x! 19)(x! 99)Q(x) + ax + b,

where a and b are constants and Q(x) is a polynomial. Then

99 = P (19) = 19a + b and 19 = P (99) = 99a + b.

It follows that 99a ! 19a = 19 ! 99, hence a = !1 and b = 99 + 19 = 118.Thus the remainder is !x + 118.

18. (E) Note that the range of log x on the interval (0, 1) is the set of all negativenumbers, infinitely many of which are zeros of the cosine function. In fact,since cos(x) = 0 for all x of the form !

2 ± n!,

f(10!2"n!) = cos(log(10

!2"n!))

= cos!

!

2! n!

"

= 0

for all positive integers n.

19. (C) Let DC = m and AD = n. By the Pythagorean Theorem, AB2 =AD2 + DB2. Hence (m + n)2 = n2 + 57, which yields m(m + 2n) = 57.Since m and n are positive integers, the only possibilities are m = 1, n = 28and m = 3, n = 8. The second of these gives the least possible value ofAC = m + n, namely 11.

Page 61: AMC

6 SOLUTIONS 1999 AHSME

20. (E) For n # 3,

an =a1 + a2 + · · · + an"1

n! 1.

Thus (n! 1)an = a1 + a2 + · · · + an"1. It follows that

an+1 =a1 + a2 + · · · + an"1 + an

n=

(n! 1) · an + an

n= an,

for n # 3. Since a9 = 99 and a1 = 19, it follows that

99 = a3 =19 + a2

2,

and hence that a2 = 179. (The sequence is 19, 179, 99, 99, . . ..)

21. (B) Since 202 + 212 = 292, the converse of thePythagorean Theorem applies, so the triangle hasa right angle. Thus its hypotenuse is a diameterof the circle, so the region with area C is a semi-circle and is congruent to the semicircle formed bythe other three regions. The area of the triangle is210, hence A + B + 210 = C. To see that the otheroptions are incorrect, note that

......................

......................

......................

......................

......................

......................

......................

............................................................................................................................................................................................................................................

.............................................................................................................................................

.....................

..............................................

............................................................................

A

B

C

(A) A + B < A + B + 210 = C;

(C) A2 + B2 < (A + B)2 < (A + B + 210)2 = C2;

(D) 20A + 21B < 29A + 29B < 29(A + B + 210) = 29C; and

(E)1

A2+

1

B2>

1

A2>

1

C2.

Page 62: AMC

SOLUTIONS 1999 AHSME 7

22. (C) The first graph is an inverted ‘V-shaped’right angle with vertex at (a, b) and the sec-ond is a V-shaped right angle with vertex at(c, d). Thus (a, b), (2, 5), (c, d), and (8, 3) areconsecutive vertices of a rectangle. The diag-onals of this rectangle meet at their commonmidpoint, so the x-coordinate of this mid-point is (2+8)/2 = (a+c)/2. Thus a+c = 10.

................................................................................................................................................................................................................................................................................................................................................................................................

................................................................................................................................................................................................................................................................................................................................................................................................

•(c, d)

(a, b)

•(2, 5)

(8, 3)

OR

Use the given information to obtain the equations 5 = !|2 ! a| + b, 5 =|2 ! c| + d, 3 = !|8 ! a| + b, and 3 = |8 ! c| + d. Subtract the third fromthe first to eliminate b and subtract the fourth from the second to eliminated. The two resulting equations |8! a|!| 2! a| = 2 and |2! c|!| 8! c| = 2can be solved for a and c. To solve the former, first consider all a % 2, forwhich the equation reduces to 8 ! a ! (2 ! a) = 2, which has no solutions.Then consider all a in the interval 2 % a % 8, for which the equation reducesto 8 ! a ! (a ! 2) = 2, which yields a = 4. Finally, consider all a # 8, forwhich the equation reduces to a!8! (a!2) = 2, which has no solutions. Theother equation can be solved similarly to show that c = 6. Thus a + c = 10.

23. (E) Extend FA and CB to meet at X, BC andED to meet at Y , and DE and AF to meet atZ. The interior angles of the hexagon are 120!.Thus the triangles XY Z, ABX, CDY, and EFZare equilateral. Since AB = 1, BX = 1. SinceCD = 2, CY = 2. Thus XY = 7 and Y Z = 7.Since Y D = 2 and DE = 4,EZ = 1. The areaof the hexagon can be found by subtracting theareas of the three small triangles from the areaof the large triangle:

.................................................................................................................................................................................................................................................................................................................................................................................................................................................. ..........

........................................................................................................................................ ..................................................................................................................................................•

Z•X

•F

•A

•Y

•E •B

•D •C

72

)"3

4

*

! 12

)"3

4

*

! 22

)"3

4

*

! 12

)"3

4

*

=43"

3

4.

Page 63: AMC

8 SOLUTIONS 1999 AHSME

24. (B) Any four of the six given points determine a unique convex quadrilateral,

so there are exactly'

64

(= 15 favorable outcomes when the chords are selected

randomly. Since there are'

62

(= 15 chords, there are

'154

(= 1365 ways to pick

the four chords. So the desired probability is 15/1365 = 1/91.

25. (B) Multiply both sides of the equation by 7! to obtain

3600 = 2520a2 + 840a3 + 210a4 + 42a5 + 7a6 + a7.

It follows that 3600! a7 is a multiple of 7, which implies that a7 = 2. Thus,

3598

7= 514 = 360a2 + 120a3 + 30a4 + 6a5 + a6.

Reason as above to show that 514! a6 is a multiple of 6, which implies thata6 = 4. Thus, 510/6 = 85 = 60a2 + 20a3 + 5a4 + a5. Then it follows that85 ! a5 is a multiple of 5, whence a5 = 0. Continue in this fashion to obtaina4 = 1, a3 = 1, and a2 = 1. Thus the desired sum is 1 + 1 + 1 + 0 + 4 + 2 = 9.

26. (D) The interior angle of a regular n-gon is 180(1!2/n). Let a be the numberof sides of the congruent polygons and let b be the number of sides of the thirdpolygon (which could be congruent to the first two polygons). Then

2 · 180!1! 2

a

"+ 180

!1! 2

b

"= 360.

Clearing denominators and factoring yields the equation

(a! 4)(b! 2) = 8,

whose four positive integral solutions are (a, b) = (5, 10), (6, 6), (8, 4), and(12, 3). These four solutions give rise to polygons with perimeters of 14, 12, 14and 21, respectively, so the largest possible perimeter is 21.

A

P = 14

............................

............................................................................................................................................................. ...........................................................................................................................................................................................................................................................................................................................

.........................

....................................

•••

•••

• •••

••

••

A

P = 12

.......................................................................... ...............................................................................................................

..................................... ..........

...........................

.....................................

.....................................

..................................... ..........

...........................

.....................................

•••• •

•••• •

•••

•• •

• A

P = 14

.....................................

........................

............. .....................................

.....................................

........................

............. .....................................

........................

.............

......................................................................

.....................................

•••

•• •

••••

•• •

A

P = 21

.....................................................................

.....

.....................................

.....................................

................................

..... .....................................

.....................................

.....................................

.....................................................................

.....

.....................................

.....................................

................................

..... .....................................

.....................................

.....................................•

•••

••

• •••

••

•••

••

• •••

Page 64: AMC

SOLUTIONS 1999 AHSME 9

27. (A) Square both sides of the equations and add the results to obtain

9(sin2 A + cos2 A) + 16(sin2 B + cos2 B) + 24(sinA cos B + sin B cos A) = 37.

Hence, 24 sin(A+B) = 12. Thus sinC = sin(180!!A!B) = sin(A+B) = 12 ,

so # C = 30! or # C = 150!. The latter is impossible because it would implythat A < 30! and consequently that 3 sinA + 4 cosB < 3 · 1

2 + 4 < 6, acontradiction. Therefore # C = 30!.

Challenge. Prove that there is a unique such triangle (up to similarity), the

one for which cosA = 5"12$

337 and cos B = 66"3

$3

74 .

28. (E) Let a, b, and c denote the number of !1’s, 1’s, and 2’s in the sequence,respectively. We need not consider the zeros. Then a, b, c are nonnegativeintegers satisfying !a + b + 2c = 19 and a + b + 4c = 99. It follows thata = 40! c and b = 59! 3c, where 0 % c % 19 (since b # 0), so

x31 + x3

2 + · · · + x3n = !a + b + 8c = 19 + 6c.

The lower bound is achieved when c = 0 (a = 40, b = 59). The upper boundis achieved when c = 19 (a = 21, b = 2). Thus m = 19 and M = 133, soM/m = 7.

Page 65: AMC

10 SOLUTIONS 1999 AHSME

29. (C) Let A, B, C, and D be the vertices of the tetrahedron. Let O be thecenter of both the inscribed and circumscribed spheres. Let the inscribedsphere be tangent to the face ABC at the point E, and let its volume beV . Note that the radius of the inscribed sphere is OE and the radius ofthe circumscribed sphere is OD. Draw OA, OB, OC, and OD to obtainfour congruent tetrahedra ABCO, ABDO, ACDO, and BCDO, each withvolume 1/4 that of the original tetrahedron. Because the two tetrahedraABCD and ABCO share the same base, $ABC, the ratio of the distancefrom O to face ABC to the distance from D to face ABC is 1/4; that is,OD = 3 · OE. Thus the volume of the circumscribed sphere is 27V . ExtendDE to meet the circumscribed sphere at F . Then DF = 2 · DO = 6 · OE.Thus EF = 2 · OE, so the sphere with diameter EF is congruent to theinscribed sphere, and thus has volume V . Similarly each of the other threespheres between the tetrahedron and the circumscribed sphere have volumeV . The five congruent small spheres have no volume in common and lieentirely inside the circumscribed sphere, so the ratio 5V/27V is the prob-ability that a point in the circumscribed sphere also lies in one of the smallspheres. The fraction 5/27 is closer to 0.2 than it is to any of the other choices.

.......................................................................................................................................................................................................................................................................................................................................................................................................................................................................................................................................................................................................................................................................................................................................................................

...................................

...........................

..............................................................................................................................................................................................................................................................................................................................................................................................................................

..................

...................

.....................

.......................

...........................

...................................

..................................................................................................................................................................................................

...................................................................................................................................................................................................................................................................................................................................................................................................................

................................................................................................................................................................................................................................................................................................................................................................................................................

........................................................................................................................................................................................................................................................................................................................................................................................................................

..............

..............

..............

..............

..............

..............

..............

..............

..............

..............

..............

..............

.............................................................................................................................................................................................................................

......................................................................................................................................................

..................

..................

..................

..................

..................

..................

..................

..................

..................

..................

..................

..................

..................

..................

..................

..................

..................

..................

..................

.

.......

....................................................................................

..................................

.....

..........................

..................................

......

............................................................... • B

•C

•D

•O

•A

• E

•F

Page 66: AMC

SOLUTIONS 1999 AHSME 11

30. (D) Let m + n = s. Then m3 + n3 + 3mn(m + n) = s3. Subtracting the givenequation from the latter yields

s3 ! 333 = 3mns! 99mn.

It follows that (s ! 33)(s2 + 33s + 332 ! 3mn) = 0, hence either s = 33 or(m + n)2 + 33(m + n) + 332 ! 3mn = 0. The second equation is equivalentto (m ! n)2 + (m + 33)2 + (n + 33)2 = 0, whose only solution, (!33,!33),qualifies. On the other hand, the solutions to m+n = 33 satisfying the requiredconditions are (0, 33), (1, 32), (2, 31), . . . , (33, 0), of which there are 34. Thusthere are 35 solutions altogether.

Page 67: AMC

First AMC..........................10 2000 2

1. In the year 2001, the United States will host the International MathematicalOlympiad. Let I, M, and O be distinct positive integers such that the productI · M · O = 2001. What is the largest possible value of the sum I + M + O?

(A) 23 (B) 55 (C) 99 (D) 111 (E) 671

2. 2000(20002000) =

(A) 20002001 (B) 40002000 (C) 20004000

(D) 4, 000, 0002000 (E) 20004,000,000

3. Each day, Jenny ate 20% of the jellybeans that were in her jar at the beginningof that day. At the end of the second day, 32 remained. How many jellybeanswere in the jar originally?

(A) 40 (B) 50 (C) 55 (D) 60 (E) 75

4. Chandra pays an on-line service provider a fixed monthly fee plus an hourlycharge for connect time. Her December bill was $12.48, but in January herbill was $17.54 because she used twice as much connect time as in December.What is the fixed monthly fee?

(A) $2.53 (B) $5.06 (C) $6.24 (D) $7.42 (E) $8.77

5. Points M and N are the midpoints of sides PA and PB of !PAB. As Pmoves along a line that is parallel to side AB, how many of the four quantitieslisted below change?

(a) the length of the segment MN

(b) the perimeter of !PAB

(c) the area of !PAB

(d) the area of trapezoid ABNM............................................................................................................................................................................................

"#

A B

P

M N

(A) 0 (B) 1 (C) 2 (D) 3 (E) 4

6. The Fibonacci sequence 1, 1, 2, 3, 5, 8, 13, 21, . . . starts with two 1s, and eachterm afterwards is the sum of its two predecessors. Which one of the tendigits is the last to appear in the units position of a number in the Fibonaccisequence?

(A) 0 (B) 4 (C) 6 (D) 7 (E) 9

Page 68: AMC

First AMC..........................10 2000 3

7. In rectangle ABCD, AD = 1, P is on AB,and DB and DP trisect ! ADC. What isthe perimeter of !BDP?

................................................................................................................................................................................................................................................................................................................................

•D

•C

•B

•A

•P

(A) 3 +

$3

3(B) 2 +

4$

3

3(C) 2 + 2

$2 (D)

3 + 3$

5

2(E) 2 +

5$

3

3

8. At Olympic High School, 2/5 of the freshmen and 4/5 of the sophomorestook the AMC

.......................... 10. Given that the number of freshmen and sophomorecontestants was the same, which of the following must be true?

(A) There are five times as many sophomores as freshmen.

(B) There are twice as many sophomores as freshmen.

(C) There are as many freshmen as sophomores.

(D) There are twice as many freshmen as sophomores.

(E) There are five times as many freshmen as sophomores.

9. If |x% 2| = p, where x < 2, then x% p =

(A) %2 (B) 2 (C) 2% 2p (D) 2p% 2 (E) |2p% 2|

10. The sides of a triangle with positive area have lengths 4, 6, and x. The sidesof a second triangle with positive area have lengths 4, 6, and y. What is thesmallest positive number that is not a possible value of |x% y|?

(A) 2 (B) 4 (C) 6 (D) 8 (E) 10

11. Two di!erent prime numbers between 4 and 18 are chosen. When their sumis subtracted from their product, which of the following numbers could beobtained?

(A) 21 (B) 60 (C) 119 (D) 180 (E) 231

Page 69: AMC

First AMC..........................10 2000 4

12. Figures 0, 1, 2, and 3 consist of 1, 5, 13, and 25 nonoverlapping unit squares,respectively. If the pattern were continued, how many nonoverlapping unitsquares would there be in figure 100?

(A) 10401 (B) 19801 (C) 20201 (D) 39801 (E) 40801

figure 0 figure 1 figure 2 figure 3

13. There are 5 yellow pegs, 4 red pegs, 3 greenpegs, 2 blue pegs, and 1 orange peg to beplaced on a triangular peg board. In howmany ways can the pegs be placed so thatno (horizontal) row or (vertical) columncontains two pegs of the same color?

(A) 0 (B) 1 (C) 5! · 4! · 3! · 2! · 1!

(D) 15!/(5! · 4! · 3! · 2! · 1!) (E) 15!

.............................................................................................................................................................................................................................................................................................

............................................................

.......

.....................................................

........................................

&& & & & &

& & & &

& & &

& &

&

14. Mrs. Walter gave an exam in a mathematics class of five students. She enteredthe scores in random order into a spreadsheet, which recalculated the classaverage after each score was entered. Mrs. Walter noticed that after eachscore was entered, the average was always an integer. The scores (listed inascending order) were 71, 76, 80, 82, and 91. What was the last score Mrs.Walter entered?

(A) 71 (B) 76 (C) 80 (D) 82 (E) 91

15. Two non-zero real numbers, a and b, satisfy ab = a%b. Which of the following

is a possible value ofa

b+

b

a% ab?

(A) %2 (B) %1

2(C)

1

3(D)

1

2(E) 2

Page 70: AMC

First AMC..........................10 2000 5

16. The diagram shows 28 lattice points, each one unit from its nearest neighbors.Segment AB meets segment CD at E. Find the length of segment AE.

(A) 4$

5/3 (B) 5$

5/3 (C) 12$

5/7 (D) 2$

5 (E) 5$

65/9

...................................................................................................................................................................................

.........................................................................................................................................................................................................................................................................................................................................................................................................................................• • • • • • •

•••••••

• • • • • • •

•••••••

A

B

C

D

E

17. Boris has an incredible coin changing machine. When he puts in a quarter, itreturns five nickels; when he puts in a nickel, it returns five pennies; and whenhe puts in a penny, it returns five quarters. Boris starts with just one penny.Which of the following amounts could Boris have after using the machinerepeatedly?

(A) $3.63 (B) $5.13 (C) $6.30 (D) $7.45 (E) $9.07

18. Charlyn walks completely around the boundary of a square whose sides areeach 5 km long. From any point on her path she can see exactly 1 km horizon-tally in all directions. What is the area of the region consisting of all pointsCharlyn can see during her walk, expressed in square kilometers and roundedto the nearest whole number?

(A) 24 (B) 27 (C) 39 (D) 40 (E) 42

19. Through a point on the hypotenuse of a right triangle, lines are drawn parallelto the legs of the triangle so that the triangle is divided into a square and twosmaller right triangles. The area of one of the two small right triangles is mtimes the area of the square. The ratio of the area of the other small righttriangle to the area of the square is

(A)1

2m + 1(B) m (C) 1%m (D)

1

4m(E)

1

8m2

20. Let A, M, and C be nonnegative integers such that A + M + C = 10. Whatis the maximum value of A · M · C + A · M + M · C + C · A?

(A) 49 (B) 59 (C) 69 (D) 79 (E) 89

Page 71: AMC

First AMC..........................10 2000 6

21. If all alligators are ferocious creatures and some creepy crawlers are alligators,which statement(s) must be true?

I. All alligators are creepy crawlers.

II. Some ferocious creatures are creepy crawlers.

III. Some alligators are not creepy crawlers.

(A) I only (B) II only (C) III only

(D) II and III only (E) None must be true

22. One morning each member of Angela’s family drank an 8-ounce mixture ofco!ee with milk. The amounts of co!ee and milk varied from cup to cup, butwere never zero. Angela drank a quarter of the total amount of milk and asixth of the total amount of co!ee. How many people are in the family?

(A) 3 (B) 4 (C) 5 (D) 6 (E) 7

23. When the mean, median, and mode of the list

10, 2, 5, 2, 4, 2, x

are arranged in increasing order, they form a non-constant arithmetic progres-sion. What is the sum of all possible real values of x?

(A) 3 (B) 6 (C) 9 (D) 17 (E) 20

24. Let f be a function for which f(x/3) = x2 + x + 1. Find the sum of all valuesof z for which f(3z) = 7.

(A) %1/3 (B) %1/9 (C) 0 (D) 5/9 (E) 5/3

25. In year N , the 300th day of the year is a Tuesday. In year N +1, the 200th dayis also a Tuesday. On what day of the week did the 100th day of year N % 1occur?

(A) Thursday (B) Friday (C) Saturday (D) Sunday (E) Monday

Page 72: AMC

Solutions 2000 AMC..........................10 2

1. (E) Factor 2001 into primes to get 2001 = 3 · 23 · 29. The largest possible sumof three distinct factors whose product is 2001 is the one which combines thetwo largest prime factors, namely I = 23 · 29 = 667,M = 3, and O = 1, sothe largest possible sum is 1 + 3 + 667 = 671.

2. (A) 2000(20002000) = (20001)(20002000) = 2000(1+2000) = 20002001. All theother options are greater than 20002001.

3. (B) Since Jenny ate 20% of the jellybeans remaining each day, 80% of thejellybeans are left at the end of each day. If x is the number of jellybeans inthe jar originally, then (0.8)2x = 32. Thus x = 50.

4. (D) Since Chandra paid an extra $5.06 in January, her December connect timemust have cost her $5.06. Therefore, her monthly fee is $12.48!$5.06 = $7.42.

5. (B) Since "ABP is similar to "MNP andPM = 1

2 ·AP , it follows that MN = 12 ·AB. Since

the base AB and the altitude to AB of "ABPdo not change, the area does not change. Thealtitude of the trapezoid is half that of the trian-gle, and the bases do not change as P changes, sothe area of the trapezoid does not change. Onlythe perimeter changes (reaching a minimum when"ABP is isosceles).

............................................................................................................................................................................................

#$

A B

P

M N

6. (C) The sequence of units digits is

1, 1, 2, 3, 5, 8, 3, 1, 4, 5, 9, 4, 3, 7, 0, 7, 7, 4, 1, 5, 6, . . . .

The digit 6 is the last of the ten digits to appear.

7. (B) Both triangles APD and CBD are

30!!60!!90! triangles. Thus DP = 2"

33

and DB = 2. Since # BDP = # PBD, itfollows that PB = PD = 2

"3

3 . Hence the

perimeter of "BDP is 2"

33 + 2

"3

3 + 2 =

2 + 4"

33 .

.......................................................................................................................................................................................................................................................................................................................................................

•D

•C

•B

•A

•P

Page 73: AMC

Solutions 2000 AMC..........................10 3

8. (D) Let f and s represent the numbers of freshmen and sophomores at theschool, respectively. According to the given condition, (2/5)f = (4/5)s. Thus,f = 2s. That is, there are twice as many freshmen as sophomores.

9. (C) Since x < 2, it follows that |x! 2| = 2! x. If 2! x = p, then x = 2! p.Thus x! p = 2! 2p.

10. (D) By the Triangle Inequality, each of x and y can be any number strictlybetween 2 and 10, so 0 % |x! y| < 8. Therefore, the smallest positive numberthat is not a possible value of |x! y| is 10! 2 = 8.

11. (C) There are five prime numbers between 4 and 18: 5, 7, 11, 13, and 17.Hence the product of any two of these is odd and the sum is even. Becausexy ! (x + y) = (x ! 1)(y ! 1) ! 1 increases as either x or y increases (sinceboth x and y are bigger than 1), the answer must be an odd number that is nosmaller than 23 = 5·7!(5+7) and no larger than 191 = 13·17!(13+17). Theonly possibility among the options is 119, and indeed 119 = 11 ·13! (11+13).

12. (C) Calculating the number of squares in the first few figures uncovers apattern. Figure 0 has 2(0) + 1 = 2(02) + 1 squares, figure 1 has 2(1) + 3 =2(12)+3 squares, figure 2 has 2(1+3)+5 = 2(22)+5 squares, and figure 3 has2(1 + 3 + 5) + 7 = 2(32) + 7 squares. In general, the number of unit squaresin figure n is

2(1 + 3 + 5 + · · · + (2n! 1)) + 2n + 1 = 2(n2) + 2n + 1.

Therefore, the figure 100 has 2(1002) + 2 · 100 + 1 = 20201.

OR

Each figure can be considered to be a large square with identical small piecesdeleted from each of the four corners. Figure 1 has 32 ! 4(1) unit squares,figure 2 has 52! 4(1+2) unit squares, and figure 3 has 72! 4 · (1+2+3) unitsquares. In general, figure n has

(2n + 1)2 ! 4(1 + 2 + . . . + n) = (2n + 1)2 ! 2n(n + 1) unit squares.

Thus figure 100 has 2012 ! 200(101) = 20201 unit squares.

OR

The number of unit squares in figure n is the sum of the first n positive oddintegers plus the sum of the first n + 1 positive odd integers. Since the sum ofthe first k positive odd integers is k2, figure n has n2 + (n + 1)2 unit squares.So figure 100 has 1002 + 1012 = 20201 unit squares.

Page 74: AMC

Solutions 2000 AMC..........................10 4

13. (B or C) To avoid having two yellow pegs in the same row or column, theremust be exactly one yellow peg in each row and in each column. Hence, start-ing at the top of the array, the peg in the first row must be yellow, the secondpeg of the second row must be yellow, the thirdpeg of the third row must be yellow, etc. Toavoid having two red pegs in some row, theremust be a red peg in each of rows 2, 3, 4, and5. The red pegs must be in the first positionof the second row, the second position of thethird row, etc. Continuation yields exactlyone ordering that meets the requirements,as shown. On the other hand, the questioncould be interpreted as asking the number ofways the pegs could be placed into the array(distinguishing the pegs of the same color).In this case, the desired count is 5! · 4! · 3! ·2! · 1!. The decision was made to give creditfor both options B and C.

............................................................................................................................................................................................................................................................................................................................................................

...........................................

...........................................

.............................o b g r y

b g r y

g r y

r y

y

14. (C) Note that the integer average condition means that the sum of the scoresof the first n students is a multiple of n. The scores of the first two studentsmust be both even or both odd, and the sum of the scores of the first threestudents must be divisible by 3. The remainders when 71, 76, 80, 82, and 91are divided by 3 are 2, 1, 2, 1, and 1, respectively. Thus the only sum of threescores divisible by 3 is 76 + 82 + 91 = 249, so the first two scores entered are76 and 82 (in some order), and the third score is 91. Since 249 is 1 larger thana multiple of 4, the fourth score must be 3 larger than a multiple of 4, and theonly possibility is 71, leaving 80 as the score of the fifth student.

Page 75: AMC

Solutions 2000 AMC..........................10 5

15. (E) Combine the three terms over a common denominator and replace ab inthe numerator with a! b to get

a

b+

b

a! ab =

a2 + b2 ! (ab)2

ab

=a2 + b2 ! (a! b)2

ab

=a2 + b2 ! (a2 ! 2ab + b2)

ab

=2ab

ab= 2.

OR

Note that a = a/b ! 1 and b = 1 ! b/a. It follows thata

b+

b

a! ab =

a

b+

b

a!

!a

b! 1

" #

1! b

a

$

=a

b+

b

a!

#a

b+

b

a! 2

$

= 2.

16. (B) Extend DC to F . Triangles FAE and DBE are similar with ratio 5 : 4.Thus AE = 5 · AB/9, AB =

&32 + 62 =

&45 = 3

&5, and AE = 5(3

&5)/9 =

5&

5/3.

......................................................................................................................................................................................................................................................................................................................................................................................................................................................................................................................................................................................................................................................................................................................• • • • • • •

•••••••

• • • • • • •

•••••••

• •

A

B

C

D

F

E

OR

Coordinatize the points so that A = (0, 3), B = (6, 0), C = (4, 2), and D =(2, 0). Then the line through A and B is given by x + 2y = 6, and the linethrough C and D is given by x ! y = 2. Solve these simultaneously to get

E =%

103 , 4

3

&. Hence AE =

'%103 ! 0

&2+

%43 ! 3

&2=

(1259 = 5

"5

3 .

Page 76: AMC

Solutions 2000 AMC..........................10 6

17. (D) Neither of the exchanges quarter# five nickels nor nickel# five pennieschanges the total value of Boris’s coins. The exchange penny # five quartersincreases the total value of Boris’s coins by $1.24. Hence, Boris must have$.01 + $1.24n after n uses of the last exchange. Only option D is of this form:745 = 1 + 124 · 6. In cents, option A is 115 more than a multiple of 124, B is17 more than a multiple of 124, C is 10 more than a multiple of 124, and E is39 more than a multiple of 124.

18. (C) At any point on Charlyn’s walk, she can see all the points inside a circleof radius 1 km. The portion of the viewable region inside the square consistsof the interior of the square except for a smaller square with side length 3 km.This portion of the viewable region has area (25! 9) km2. The portion of theviewable region outside the square consists of four rectangles, each 5 km by1 km, and four quarter-circles, each with a radius of 1 km. This portion ofthe viewable region has area 4(5 + !

4 ) = (20 + !) km2. The area of the entireviewable region is 36 + ! ' 39 km2.

........

........................................

.....................................................................................................................................................................................................................................................................................................

.........

........................................

.....................................

......................................................................................

......................................................................................

......................................................................................

Page 77: AMC

Solutions 2000 AMC..........................10 7

19. (D) Without loss of generality, let the sideof the square have length 1 unit and let thearea of triangle ADF be m. Let AD = r andEC = s. Because triangles ADF and FECare similar, s/1 = 1/r. Since 1

2r = m, thearea of triangle FEC is 1

2s = 12r = 1

4m ............

......................

......................

......................

......................

......................

......................

......................

....................•

A

r

s

1•

B•C

•D

•E

•F

OR

Let B = (0, 0), E = (1, 0), F = (1, 1), andD = (0, 1) be the vertices of the square.Let C = (1 + 2m, 0), and notice that thearea of BEFD is 1 and the area of triangleFEC is m. The slope of the line through

C and F is ! 1

2m; thus, it intersects the

y-axis at A =!0, 1 +

1

2m

". The area of

triangle ADF is therefore1

4m.

......................

......................

......................

......................

......................

......................

......................

......................

......................

......................

......................

......................

.............•A

•(0, 0)

•(1 + 2m, 0)

•(0, 1)

•(1, 0)

•(1, 1)

20. (C) Note that

AMC+AM +MC+CA = (A+1)(M +1)(C+1)!(A+M +C)!1 = pqr!11,

where p, q, and r are positive integers whose sum is 13. A case-by-case analysisshows that pqr is largest when two of the numbers p, q, r are 4 and the thirdis 5. Thus the answer is 4 · 4 · 5! 11 = 69.

Page 78: AMC

Solutions 2000 AMC..........................10 8

21. (B) From the conditions we can conclude that some creepy crawlers are fe-rocious (since some are alligators). Hence, there are some ferocious creaturesthat are creepy crawlers, and thus II must be true. The diagram below showsthat the only conclusion that can be drawn is existence of an animal in theregion with the dot. Thus, neither I nor III follows from the given conditions.

.......

.......

.......

.........................................................................................

..................

....................

.......................

...............................

..........................................................................................................................................................................................................................................................................................................................................................................................................................................................................................................................................................................................................................................

...................................

.........................

........................................................................................................................................................ .......

.......

.......

.........................................................................................

..................

....................

.......................

...............................

..........................................................................................................................................................................................................................................................................................................................................................................................................................................................................................................................................................................................................................................

...................................

.........................

........................................................................................................................................................

ferociouscreatures

creepycrawlersalligators

•.................................

..............................................................................................

......................................................................................................................................................................................................................................................................................................................................................................................................................................

.............

22. (C) Suppose that the whole family drank x cups of milk and y cups of co!ee.Let n denote the number of people in the family. The information given impliesthat x/4 + y/6 = (x + y)/n. This leads to

3x(n! 4) = 2y(6! n).

Since x and y are positive, the only positive integer n for which both sideshave the same sign is n = 5.

OR

If Angela drank c cups of co!ee and m cups of milk, then 0 < c < 1 andm + c = 1. The number of people in the family is 6c + 4m = 4 + 2c, which isan integer if and only if c = 1

2 . Thus, there are 5 people in the family.

23. (E) If x were less than or equal to 2, then 2 would be both the median and themode of the list. Thus x > 2. Consider the two cases 2 < x < 4, and x ( 4.

Case 1: If 2 < x < 4, then 2 is the mode, x is the median, and 25+x7 is the

mean, which must equal 2 ! (x! 2) , x+22 , or x + (x! 2), depending on the

size of the mean relative to 2 and x . These give x = 38 , x = 36

5 , and x = 3, ofwhich x = 3 is the only value between 2 and 4.

Case 2: If x ( 4, then 4 is the median, 2 is the mode, and 25+x7 is the mean,

which must be 0, 3, or 6. Thus x = !25,!4, or 17, of which 17 is the only oneof these values greater than or equal to 4.

Thus the x-values sum to 3 + 17 = 20.

Page 79: AMC

Solutions 2000 AMC..........................10 9

24. (B) Let x = 9z. Then f(3z) = f(9z/3) = f(3z) = (9z)2 + 9z + 1 = 7.Simplifying and solving the equation for z yields 81z2 + 9z ! 6 = 0, so 3(3z +1)(9z ! 2) = 0. Thus z = !1/3 or z = 2/9. The sum of these values is !1/9.

Note. The answer can also be obtained by using the sum-of-roots formula on81z2 + 9z ! 6 = 0. The sum of the roots is !9/81 = !1/9.

25. (A) Note that, if a Tuesday is d days after a Tuesday, then d is a multiple of7. Next, we need to consider whether any of the years N!1, N,N +1 is a leapyear. If N is not a leap year, the 200th day of year N+1 is 365!300+200 = 265days after a Tuesday, and thus is a Monday, since 265 is 6 larger than a multipleof 7. Thus, year N is a leap year and the 200th day of year N + 1 is anotherTuesday (as given), being 266 days after a Tuesday. It follows that year N ! 1is not a leap year. Therefore, the 100th day of year N ! 1 precedes the givenTuesday in year N by 365!100+300 = 565 days, and therefore is a Thursday,since 565 = 7 · 80 + 5 is 5 larger than a multiple of 7.

Page 80: AMC

2nd AMC 10 2001

1. The median of the list

n, n + 3, n + 4, n + 5, n + 6, n + 8, n + 10, n + 12, n + 15

is 10. What is the mean?

(A) 4 (B) 6 (C) 7 (D) 10 (E) 11

2. A number x is 2 more than the product of its reciprocal and its additive inverse.In which interval does the number lie?

(A) !4 " x " !2 (B) !2 < x " 0 (C) 0 < x " 2

(D) 2 < x " 4 (E) 4 < x " 6

3. The sum of two numbers is S. Suppose 3 is added to each number and theneach of the resulting numbers is doubled. What is the sum of the final twonumbers?

(A) 2S + 3 (B) 3S + 2 (C) 3S + 6 (D) 2S + 6 (E) 2S + 12

4. What is the maximum number for the possible points of intersection of a circleand a triangle?

(A) 2 (B) 3 (C) 4 (D) 5 (E) 6

1

Page 81: AMC

2nd AMC 10 2001

5. How many of the twelve pentominoes pictured below have at least one line ofsymmetry?

(A) 3 (B) 4 (C) 5 (D) 6 (E) 7

6. Let P (n) and S(n) denote the product and the sum, respectively, of the digitsof the integer n. For example, P (23) = 6 and S(23) = 5. Suppose N is atwo-digit number such that N = P (N) + S(N). What is the units digit of N?

(A) 2 (B) 3 (C) 6 (D) 8 (E) 9

2

Page 82: AMC

2nd AMC 10 2001

7. When the decimal point of a certain positive decimal number is moved fourplaces to the right, the new number is four times the reciprocal of the originalnumber. What is the original number?

(A) 0.0002 (B) 0.002 (C) 0.02 (D) 0.2 (E) 2

8. Wanda, Darren, Beatrice, and Chi are tutors in the school math lab. Theirschedule is as follows: Darren works every third school day, Wanda worksevery fourth school day, Beatrice works every sixth school day, and Chi worksevery seventh school day. Today they are all working in the math lab. In howmany school days from today will they next be together tutoring in the lab?

(A) 42 (B) 84 (C) 126 (D) 178 (E) 252

9. The state income tax where Kristin lives is levied at the rate of p% of the first$28000 of annual income plus (p + 2)% of any amount above $28000. Kristinnoticed that the state income tax she paid amounted to (p + 0.25)% of herannual income. What was her annual income?

(A) $28000 (B) $32000 (C) $35000 (D) $42000 (E) $56000

10. If x, y, and z are positive with xy = 24, xz = 48, and yz = 72, then x + y + zis

(A) 18 (B) 19 (C) 20 (D) 22 (E) 24

3

Page 83: AMC

2nd AMC 10 2001

11. Consider the dark square in an array of unit squares, part of which is shown.The first ring of squares around this center square contains 8 unit squares. Thesecond ring contains 16 unit squares. If we continue this process, the number

of unit squares in the 100th ring is

(A) 396 (B) 404 (C) 800 (D) 10,000 (E) 10,404

................

................

................

................

................

................

.

................

................

................

................

................

................

.

................

................

................

................

................

................

.

................

................

................

................

................

................

.

................

................

................

................

................

................

.

................

................

................

................

................

................

.

................

................

................

................

................

................

.

................

................

................

................

................

................

.

................

................

................

................

................

................

.

................

................

................

................

................

................

.

................

................

................

................

................

................

.

................

................

................

................

................

................

.

................

................

................

................

................

................

.

................

................

................

................

................

................

.

................

................

................

................

................

................

.

................

................

................

................

................

................

.

................

................

................

................

................

................

.

................

................

................

................

................

................

.

................

................

................

................

................

................

.

................

................

................

................

................

................

.

................

................

................

................

................

................

.

................

................

................

................

................

................

.

................

................

................

................

................

................

.

................

................

................

................

................

................

.

................

................

................

................

................

................

.

................

................

................

................

................

................

.

................

................

................

................

................

................

.

................

................

................

................

................

................

.

................

................

................

................

................

................

.

................

................

................

................

................

................

.

................

................

................

................

................

................

.

................

................

................

................

................

................

.

................

................

................

................

................

................

.

................

................

................

................

................

................

.

................

................

................

................

................

................

.

................

................

................

................

................

................

.

................

................

................

................

................

................

.

................

................

................

................

................

................

.

................

................

................

................

................

................

.

................

................

................

................

................

................

.

................

................

................

................

................

................

.

................

................

................

12. Suppose that n is the product of three consecutive integers and that n isdivisible by 7. Which of the following is not necessarily a divisor of n?

(A) 6 (B) 14 (C) 21 (D) 28 (E) 42

13. A telephone number has the form ABC-DEF-GHIJ, where each letter repre-sents a di!erent digit. The digits in each part of the number are in decreasingorder; that is, A > B > C, D > E > F, and G > H > I > J. Furthermore,D, E, and F are consecutive even digits; G, H, I, and J are consecutive odddigits; and A + B + C = 9. Find A.

(A) 4 (B) 5 (C) 6 (D) 7 (E) 8

14. A charity sells 140 benefit tickets for a total of $2001. Some tickets sell forfull price (a whole dollar amount), and the rest sell for half price. How muchmoney is raised by the full-price tickets?

(A) $782 (B) $986 (C) $1158 (D) $1219 (E) $1449

4

Page 84: AMC

2nd AMC 10 2001

15. A street has parallel curbs 40 feet apart. A crosswalk bounded by two parallelstripes crosses the street at an angle. The length of the curb between thestripes is 15 feet and each stripe is 50 feet long. Find the distance, in feet,between the stripes.

(A) 9 (B) 10 (C) 12 (D) 15 (E) 25

16. The mean of three numbers is 10 more than the least of the numbers and 15less than the greatest. The median of the three numbers is 5. What is theirsum?

(A) 5 (B) 20 (C) 25 (D) 30 (E) 36

17. Which of the cones below can be formed from a 252! sector of a circle of radius10 by aligning the two straight sides?

.......

.......

.......................................................................

.....................................................................................................................................................................................................................................................

.................................

252!...........................................................................

•10

(A)

.........................................................................................................................................................

.........................................................................................................................................................

...................................................................

........................

.............................................................................................................................................................

......................................................................

.......

.......

.......

.......

.......

.......

.......

.......

.......

.......

.......

.......

.......

.......

.......

.......

.......

.......

......

10

6•

(B)

.........................................................................................................................................................

.........................................................................................................................................................

...................................................................

........................

.............................................................................................................................................................

.......

.......

.......

.......

.......

.......

.......

.......

.......

.......

.......

.......

.......

.......

.......

.......

.......

.......

.......

......

...............................................................

10

6•

(C)

.........................................................................................................................................................

.........................................................................................................................................................

...................................................................

........................

.............................................................................................................................................................

.......

.......

.......

.......

.......

.......

.......

.......

.......

.......

.......

.......

.......

.......

.......

.......

.......

.......

.......

......

...............................................................

10

7•

(D)

.........................................................................................................................................................

.........................................................................................................................................................

...................................................................

........................

.............................................................................................................................................................

.......

.......

.......

.......

.......

.......

.......

.......

.......

.......

.......

.......

.......

.......

.......

.......

.......

.......

.......

......

...............................................................

10

7•

(E)

.........................................................................................................................................................

.........................................................................................................................................................

...................................................................

........................

.............................................................................................................................................................

.......

.......

.......

.......

.......

.......

.......

.......

.......

.......

.......

.......

.......

.......

.......

.......

.......

.......

.......

......

...............................................................

10

8•

5

Page 85: AMC

2nd AMC 10 2001

18. The plane is tiled by congruent squares and congruent pentagons as indicated.The percent of the plane that is enclosed by the pentagons is closest to

(A) 50 (B) 52 (C) 54 (D) 56 (E) 58

....................................

........................

............

....................................

........................

............

....................................

........................

............

....................................

........................

............

....................................

........................

............

....................................

........................

............

....................................

........................

............

....................................

........................

............

....................................

........................

............

00 ···

...

1

1 ···

...

2

2 ···

...

3

3 ···

...

4

4 ···

...

5

5 ···

...

6

6 ···

...

7

7 ···

...

8

8 ···

...

9

9 ···

...

19. Pat wants to buy four donuts from an ample supply of three types of donuts:glazed, chocolate, and powdered. How many di!erent selections are possible?

(A) 6 (B) 9 (C) 12 (D) 15 (E) 18

20. A regular octagon is formed by cutting an isosceles right triangle from each ofthe corners of a square with sides of length 2000. What is the length of eachside of the octagon?

(A)1

3(2000) (B) 2000

!#2! 1

"(C) 2000

!2!

#2"

(D) 1000 (E) 1000#

2

21. A right circular cylinder with its diameter equal to its height is inscribed in aright circular cone. The cone has diameter 10 and altitude 12, and the axesof the cylinder and cone coincide. Find the radius of the cylinder.

(A)8

3(B)

30

11(C) 3 (D)

25

8(E)

7

2

6

Page 86: AMC

2nd AMC 10 2001

22. In the magic square shown, the sums of the numbers in each row, column, anddiagonal are the same. Five of these numbers are represented by v, w, x, y, andz. Find y + z.

25 z 21

18 x y

v 24 w

(A) 43 (B) 44 (C) 45 (D) 46 (E) 47

23. A box contains exactly five chips, three red and two white. Chips are randomlyremoved one at a time without replacement until all the red chips are drawnor all the white chips are drawn. What is the probability that the last chipdrawn is white?

(A)3

10(B)

2

5(C)

1

2(D)

3

5(E)

7

10

24. In trapezoid ABCD, AB and CD are perpendicular to AD, with AB +CD =BC, AB < CD, and AD = 7. What is AB · CD?

(A) 12 (B) 12.25 (C) 12.5 (D) 12.75 (E) 13

25. How many positive integers not exceeding 2001 are multiples of 3 or 4 but not5?

(A) 768 (B) 801 (C) 934 (D) 1067 (E) 1167

7

Page 87: AMC

2nd AMC 10 2001

1. (E) The middle number in the 9-number list is n + 6, which is given as 10.Thus n = 4. Add the terms together to get 9n + 63 = 9 · 4 + 63 = 99. Thusthe mean is 99/9 = 11.

2. (C) The reciprocal of x is 1/x, and the additive inverse of x is !x. Theproduct of these is (1/x) · (!x) = !1. So x = !1 + 2 = 1, which is in theinterval 0 < x " 2.

3. (E) Suppose the two numbers are a and b. Then the desired sum is

2(a + 3) + 2(b + 3) = 2(a + b) + 12 = 2S + 12.

4. (E) The circle can intersect at most two points of each side of the triangle,so the number can be no greater than six. The figure shows that the numbercan indeed be six.

...............................................................................................................................................................................................................................................................................................................................................................................................................................................................................................................................................................

......................................................................

......................................................................

......................................................................

....................................

..........................................................................................................................................................................................................................................................................................................................................

..............................................................................................................................................................................................................................................

.......................

............................

..................................................

.........................................................................................................................................................................................

1

Page 88: AMC

2nd AMC 10 2001

5. (D) Exactly six have at least one line of symmetry. They are:

6. (E) Suppose N = 10a+b. Then 10a+b = ab+(a+b). It follows that 9a = ab,which implies that b = 9, since a #= 0.

7. (C) If x is the number, then moving the decimal point four places to the rightis the same as multiplying x by 10000. That is, 10000x = 4 · (1/x), which isequivalent to x2 = 4/10000. Since x is positive, it follows that x = 2/100 =0.02.

8. (B) The number of school days until they will next be together is the leastcommon multiple of 3, 4, 6, and 7, which is 84.

9. (B) If Kristin’s annual income is x $ 28000 dollars, then

p

100· 28000 +

p + 2

100· (x! 28000) =

p + 0.25

100· x.

Multiplying by 100 and expanding yields

28000p + px + 2x! 28000p! 56000 = px + 0.25x.

So, 1.75x = 74x = 56000 and x = 32000.

2

Page 89: AMC

2nd AMC 10 2001

10. (D) Since

x =24

y=

48

z,

we have z = 2y. So 72 = 2y2, which implies that y = 6, x = 4, and z = 12.Hence x + y + z = 22.

OR

Take the product of the equations to get xy · xz · yz = 24 · 48 · 72. Thus

(xyz)2 = 23 · 3 · 24 · 3 · 23 · 32 = 210 · 34.

So (xyz)2 = (25 · 32)2, and we have xyz = 25 · 32. Therefore,

x =xyz

yz=

25 · 32

23 · 32= 4.

From this it follows that y = 6 and z = 12, so the sum is 4 + 6 + 12 = 22.

11. (C) The nth ring can be partitioned into four rectangles: two containing2n + 1 unit squares and two containing 2n! 1 unit squares. So there are

2(2n + 1) + 2(2n! 1) = 8n

unit squares in the nth ring. Thus, the 100th ring has 8 · 100 = 800 unitsquares.

OR

The nth ring can be obtained by removing a square of side 2n ! 1 from asquare of side 2n + 1. So it contains

(2n + 1)2 ! (2n! 1)2 = (4n2 + 4n + 1)! (4n2 ! 4n + 1) = 8n

unit squares.

12. (D) In any triple of consecutive integers, at least one is even and one is amultiple of 3. Therefore, the product of the three integers is both even and amultiple of 3. Since 7 is a divisor of the product, the numbers 6, 14, 21, and42 must also be divisors of the product. However, 28 contains two factors of2, and n need not. For example, 5 · 6 · 7 is divisible by 7, but not by 28.

3

Page 90: AMC

2nd AMC 10 2001

13. (E) The last four digits (GHIJ) are either 9753 or 7531, and the remainingodd digit (either 1 or 9) is A, B, or C. Since A + B + C = 9, the odd digitamong A, B, and C must be 1. Thus the sum of the two even digits in ABCis 8. The three digits in DEF are 864, 642, or 420, leaving the pairs 2 and 0, 8and 0, or 8 and 6, respectively, as the two even digits in ABC. Of those, onlythe pair 8 and 0 has sum 8, so ABC is 810, and the required first digit is 8.The only such telephone number is 810-642-9753.

14. (A) Let n be the number of full-price tickets and p be the price of each indollars. Then

np + (140! n) · p

2= 2001, so p(n + 140) = 4002.

Thus n + 140 must be a factor of 4002 = 2 · 3 · 23 · 29. Since 0 " n " 140, wehave 140 " n + 140 " 280, and the only factor of 4002 that is in the requiredrange for n + 140 is 174 = 2 · 3 · 29. Therefore, n + 140 = 174, so n = 34 andp = 23. The money raised by the full-price tickets is 34 · 23 = 782 dollars.

15. (C) The crosswalk is in the shape of a parallelogram with base 15 feet andaltitude 40 feet, so its area is 15% 40 = 600 ft2. But viewed another way, theparallelogram has base 50 feet and altitude equal to the distance between thestripes, so this distance must be 600/50 = 12 feet.

............................................................................................................................................................................

............................................................................................................................................................................

40 50 50

15

15

4

Page 91: AMC

2nd AMC 10 2001

16. (D) Since the median is 5, we can write the three numbers as x, 5, and y,where

1

3(x + 5 + y) = x + 10 and

1

3(x + 5 + y) + 15 = y.

If we add these equations, we get

2

3(x + 5 + y) + 15 = x + y + 10,

and solving for x + y gives x + y = 25. Hence, the sum of the numbers isx + 5 + y = 30.

OR

Let m be the mean of the three numbers. Then the least of the numbers ism ! 10 and the greatest is m + 15. The middle of the three numbers is themedian, 5. So

1

3((m! 10) + 5 + (m + 15)) = m

and m = 10. Hence, the sum of the three numbers is 3(10) = 30.

17. (C) The slant height of the cone is 10, the radius of the sector. The circum-ference of the base of the cone is the same as the length of the sector’s arc.This is 252/360 = 7/10 of the circumference, 20!, of the circle from which thesector is cut. The base circumference of the cone is 14!, so its radius is 7.

18. (D) The pattern shown below is repeated in the plane. In fact, nine repetitionsof it are shown in the statement of the problem. Note that four of the ninesquares in the three-by-three square are not in the four pentagons that makeup the three-by-three square. Therefore, the percentage of the plane that isenclosed by pentagons is

1! 4

9=

5

9= 55

5

9%.

..........................................................................................................................................................

5

Page 92: AMC

2nd AMC 10 2001

19. (D) The number of possible selections is the number of solutions of theequation

g + c + p = 4

where g, c, and p represent, respectively, the number of glazed, chocolate,and powdered donuts. The 15 possible solutions to this equation are (4, 0, 0),(0, 4, 0), (0, 0, 4), (3, 0, 1), (3, 1, 0), (1, 3, 0), (0, 3, 1), (1, 0, 3), (0, 1, 3), (2, 2, 0),(2, 0, 2), (0, 2, 2), (2, 1, 1), (1, 2, 1), and (1, 1, 2).

ORCode each selection as a sequence of four &’s and two |’s, where each & repre-sents a donut and each | denotes a “separator” between types of donuts. Forexample, & & | & | & represents two glazed donuts, one chocolate donut, andone powdered donut. From the six slots that can be occupied by a | or a &, wemust choose two places for the |’s to determine a selection. Thus, there are!

62

"' C6

2 = 15 selections.

20. (B) Let x represent the length of each side of the octagon, which is also thelength of the hypotenuse of each of the right triangles. Each leg of the righttriangles has length x

(2/2, so

2 · x(

2

2+ x = 2000, and x =

2000(2 + 1

= 2000!(

2! 1".

6

Page 93: AMC

2nd AMC 10 2001

21. (B) Let the cylinder have radius r and height 2r. Since )APQ is similar to)AOB, we have

12! 2r

r=

12

5, so r =

30

11.

............................................................................................................................................................................................................................................................................................................................................................................................................................

............................................................................................

O

A

B

P Qr

12! 2r

2r

22. (D) Since v appears in the first row, first column, and one diagonal, the sumof the remaining two numbers in each of these lines must be the same. Thus,

25 + 18 = 24 + w = 21 + x,

so w = 19 and x = 22. Now 25, 22, and 19 form a diagonal with a sum of 66,so we can find v = 23, y = 26, and z = 20. Hence y + z = 46.

7

Page 94: AMC

2nd AMC 10 2001

23. (D) Think of continuing the drawing until all five chips are removed from thebox. There are ten possible orderings of the colors: RRRWW, RRWRW,RWRRW, WRRRW, RRWWR, RWRWR, WRRWR, RWWRR, WRWRR,and WWRRR. The six orderings that end in R represent drawings that wouldhave ended when the second white chip was drawn.

OR

Imagine drawing until only one chip remains. If the remaining chip is red,then that draw would have ended when the second white chip was removed.The last chip will be red with probability 3/5.

24. (B) Let E be the foot of the perpendicular from B to CD. Then AB = DEand BE = AD = 7. By the Pythagorean Theorem,

AD2 = BE2 = BC2 ! CE2

= (CD + AB)2 ! (CD ! AB)2

= (CD + AB + CD ! AB)(CD + AB ! CD + AB)

= 4 · CD · AB.

Hence, AB · CD = AD2/4 = 72/4 = 49/4 = 12.25.......................................................................................................................................................................................................

•A •D

•E

•C

•B

25. (B) For integers not exceeding 2001, there are *2001/3+ = 667 multiples of 3and *2001/4+ = 500 multiples of 4. The total, 1167, counts the *2001/12+ =166 multiples of 12 twice, so there are 1167 ! 166 = 1001 multiples of 3or 4. From these we exclude the *2001/15+ = 133 multiples of 15 and the*2001/20+ = 100 multiples of 20, since these are multiples of 5. However, thisexcludes the *2001/60+ = 33 multiples of 60 twice, so we must re-include these.The number of integers satisfying the conditions is 1001!133!100+33 = 801.

8

Page 95: AMC

51st AMC..........................12 2000 2

1. In the year 2001, the United States will host the International MathematicalOlympiad. Let I, M, and O be distinct positive integers such that the productI · M · O = 2001. What is the largest possible value of the sum I + M + O?

(A) 23 (B) 55 (C) 99 (D) 111 (E) 671

2. 2000(20002000) =

(A) 20002001 (B) 40002000 (C) 20004000

(D) 4, 000, 0002000 (E) 20004,000,000

3. Each day, Jenny ate 20% of the jellybeans that were in her jar at the beginningof that day. At the end of the second day, 32 remained. How many jellybeanswere in the jar originally?

(A) 40 (B) 50 (C) 55 (D) 60 (E) 75

4. The Fibonacci sequence 1, 1, 2, 3, 5, 8, 13, 21, . . . starts with two 1s, and eachterm afterwards is the sum of its two predecessors. Which one of the tendigits is the last to appear in the units position of a number in the Fibonaccisequence?

(A) 0 (B) 4 (C) 6 (D) 7 (E) 9

5. If |x! 2| = p, where x < 2, then x! p =

(A) !2 (B) 2 (C) 2! 2p (D) 2p! 2 (E) |2p! 2|

6. Two di!erent prime numbers between 4 and 18 are chosen. When their sumis subtracted from their product, which of the following numbers could beobtained?

(A) 21 (B) 60 (C) 119 (D) 180 (E) 231

7. How many positive integers b have the property that logb 729 is a positiveinteger?

(A) 0 (B) 1 (C) 2 (D) 3 (E) 4

Page 96: AMC

51st AMC..........................12 2000 3

8. Figures 0, 1, 2, and 3 consist of 1, 5, 13, and 25 nonoverlapping unit squares,respectively. If the pattern were continued, how many nonoverlapping unitsquares would there be in figure 100?

(A) 10401 (B) 19801 (C) 20201 (D) 39801 (E) 40801

figure 0 figure 1 figure 2 figure 3

9. Mrs. Walter gave an exam in a mathematics class of five students. She enteredthe scores in random order into a spreadsheet, which recalculated the classaverage after each score was entered. Mrs. Walter noticed that after eachscore was entered, the average was always an integer. The scores (listed inascending order) were 71, 76, 80, 82, and 91. What was the last score Mrs.Walter entered?

(A) 71 (B) 76 (C) 80 (D) 82 (E) 91

10. The point P = (1, 2, 3) is reflected in the xy-plane, then its image Q is rotatedby 180! about the x-axis to produce R, and finally, R is translated by 5 unitsin the positive-y direction to produce S. What are the coordinates of S?

(A) (1, 7,!3) (B) (!1, 7,!3) (C) (!1,!2, 8)

(D) (!1, 3, 3) (E) (1, 3, 3)

11. Two non-zero real numbers, a and b, satisfy ab = a!b. Which of the following

is a possible value ofa

b+

b

a! ab?

(A) !2 (B) !1

2(C)

1

3(D)

1

2(E) 2

12. Let A, M, and C be nonnegative integers such that A + M + C = 12. Whatis the maximum value of A · M · C + A · M + M · C + C · A?

(A) 62 (B) 72 (C) 92 (D) 102 (E) 112

13. One morning each member of Angela’s family drank an 8-ounce mixture ofco!ee with milk. The amounts of co!ee and milk varied from cup to cup, butwere never zero. Angela drank a quarter of the total amount of milk and asixth of the total amount of co!ee. How many people are in the family?

(A) 3 (B) 4 (C) 5 (D) 6 (E) 7

Page 97: AMC

51st AMC..........................12 2000 4

14. When the mean, median, and mode of the list

10, 2, 5, 2, 4, 2, x

are arranged in increasing order, they form a non-constant arithmetic progres-sion. What is the sum of all possible real values of x?

(A) 3 (B) 6 (C) 9 (D) 17 (E) 20

15. Let f be a function for which f(x/3) = x2 + x + 1. Find the sum of all valuesof z for which f(3z) = 7.

(A) !1/3 (B) !1/9 (C) 0 (D) 5/9 (E) 5/3

16. A checkerboard of 13 rows and 17 columns has a number written in eachsquare, beginning in the upper left corner, so that the first row is numbered1, 2, . . . , 17, the second row 18, 19, . . . , 34, and so on down the board. If theboard is renumbered so that the left column, top to bottom, is 1, 2, . . . , 13, thesecond column 14, 15, . . . , 26 and so on across the board, some squares havethe same numbers in both numbering systems. Find the sum of the numbersin these squares (under either system).

(A) 222 (B) 333 (C) 444 (D) 555 (E) 666

17. A circle centered at O has radius 1 and containsthe point A. Segment AB is tangent to the circleat A and " AOB = !. If point C lies on OA andBC bisects " ABO, then OC =

(A) sec2 ! ! tan ! (B)1

2(C)

cos2 !

1 + sin !

(D)1

1 + sin !(E)

sin !

cos2 !

...........................................................................................................................................................................

.......................................................................................................................................................

..................................................................................................

...............................................................................................................................................................

...........................

.................................................................................................................................................................................................................................................................................................................

•O

!

•B

•C

•A

18. In year N , the 300th day of the year is a Tuesday. In year N +1, the 200th dayis also a Tuesday. On what day of the week did the 100th day of year N ! 1occur?

(A) Thursday (B) Friday (C) Saturday (D) Sunday (E) Monday

Page 98: AMC

51st AMC..........................12 2000 5

19. In triangle ABC, AB = 13, BC = 14, and AC = 15. Let D denote themidpoint of BC and let E denote the intersection of BC with the bisector ofangle BAC. Which of the following is closest to the area of the triangle ADE?

(A) 2 (B) 2.5 (C) 3 (D) 3.5 (E) 4

20. If x, y, and z are positive numbers satisfying

x + 1/y = 4, y + 1/z = 1, and z + 1/x = 7/3,

then xyz =

(A) 2/3 (B) 1 (C) 4/3 (D) 2 (E) 7/3

21. Through a point on the hypotenuse of a right triangle, lines are drawn parallelto the legs of the triangle so that the triangle is divided into a square and twosmaller right triangles. The area of one of the two small right triangles is mtimes the area of the square. The ratio of the area of the other small righttriangle to the area of the square is

(A)1

2m + 1(B) m (C) 1!m (D)

1

4m(E)

1

8m2

22. The graph below shows a portion of the curve defined by the quartic polyno-mial P (x) = x4 + ax3 + bx2 + cx + d. Which of the following is the smallest?

(A) P (!1)

(B) The product of the zeros of P

(C) The product of the non-real zeros of P

(D) The sum of the coe"cients of P

(E) The sum of the real zeros of P

10

-10

-3 3

..............................................................................................................................................................................................................................................................

...........................................................................................................................................................................................................................................................................................................................................................................................................................................................................................................................................................................................................................................................................................................................................................................

Page 99: AMC

51st AMC..........................12 2000 6

23. Professor Gamble buys a lottery ticket, which requires that he pick six di!erentintegers from 1 through 46, inclusive. He chooses his numbers so that the sumof the base-ten logarithms of his six numbers is an integer. It so happensthat the integers on the winning ticket have the same property— the sum ofthe base-ten logarithms is an integer. What is the probability that ProfessorGamble holds the winning ticket?

(A) 1/5 (B) 1/4 (C) 1/3 (D) 1/2 (E) 1

24. If circular arcs AC and BC have centers at B andA, respectively, then there exists a circle tangent

to both"AC and

"BC, and to AB. If the length of

"BC is 12, then the circumference of the circle is

(A) 24 (B) 25 (C) 26 (D) 27 (E) 28..........................................................................................................

...

.......

.......

.............................................................

...................

...............

..............................................................................................

......................................................................................................................................................•

A•B

•C

25. Eight congruent equilateral triangles, each of a di!er-ent color, are used to construct a regular octahedron.How many distinguishable ways are there to constructthe octahedron? (Two colored octahedrons are distin-guishable if neither can be rotated to look just like theother.)

(A) 210 (B) 560 (C) 840

(D) 1260 (E) 1680

..........................................................................................................................................................................................................................................................................................................

. . . . . . . . . . . . . . . . . . . . . . . .............................

............................

............................

............................

............

..................................................................................................................................................................................................................................................................................

...................................................................................................................................................................................

..............................................................................................................................

..............................

..............................

..............................

....

........................

........................

Page 100: AMC

2 Solutions 2000 AMC..........................12

1. (E) Factor 2001 into primes to get 2001 = 3 · 23 · 29. The largest possible sumof three distinct factors whose product is 2001 is the one which combines thetwo largest prime factors, namely I = 23 · 29 = 667, M = 3, and O = 1, sothe largest possible sum is 1 + 3 + 667 = 671.

2. (A) 2000(20002000) = (20001)(20002000) = 2000(1+2000) = 20002001. All theother options are greater than 20002001.

3. (B) Since Jenny ate 20% of the jellybeans remaining each day, 80% of thejellybeans are left at the end of each day. If x is the number of jellybeans inthe jar originally, then (0.8)2x = 32. Thus x = 50.

4. (C) The sequence of units digits is

1, 1, 2, 3, 5, 8, 3, 1, 4, 5, 9, 4, 3, 7, 0, 7, 7, 4, 1, 5, 6, . . . .

The digit 6 is the last of the ten digits to appear.

5. (C) Since x < 2, it follows that |x! 2| = 2! x. If 2! x = p, then x = 2! p.Thus x! p = 2! 2p.

6. (C) There are five prime numbers between 4 and 18: 5, 7, 11, 13, and 17.Hence the product of any two of these is odd and the sum is even. Becausexy!(x+y) = (x!1)(y!1)!1 increases as either x or y increases (since both xand y are bigger than 1), the answer must be an odd number that is no smallerthan 23 = 5 · 7! (5+7) and no larger than 191 = 13 · 17! (13+17). The onlypossibility among the options is 119, and indeed 119 = 11 · 13! (11 + 13).

7. (E) If logb 729 = n, then bn = 729 = 36, so n must be an integer factor of 6;that is, n = 1, 2, 3, or 6. Since 729 = 7291 = 272 = 93 = 36, the correspondingvalues of b are 36, 33, 32, and 3.

Page 101: AMC

Solutions 2000 AMC..........................12 3

8. (C) Calculating the number of squares in the first few figures uncovers apattern. Figure 0 has 2(0) + 1 = 2(02) + 1 squares, figure 1 has 2(1) + 3 =2(12)+3 squares, figure 2 has 2(1+3)+5 = 2(22)+5 squares, and figure 3 has2(1 + 3 + 5) + 7 = 2(32) + 7 squares. In general, the number of unit squaresin figure n is

2(1 + 3 + 5 + · · · + (2n! 1)) + 2n + 1 = 2(n2) + 2n + 1.

Therefore, the figure 100 has 2(1002) + 2 · 100 + 1 = 20201.

OR

Each figure can be considered to be a large square with identical small piecesdeleted from each of the four corners. Figure 1 has 32 ! 4(1) unit squares,figure 2 has 52! 4(1+2) unit squares, and figure 3 has 72! 4 · (1+2+3) unitsquares. In general, figure n has

(2n + 1)2 ! 4(1 + 2 + . . . + n) = (2n + 1)2 ! 2n(n + 1) unit squares.

Thus figure 100 has 2012 ! 200(101) = 20201 unit squares.

OR

The number of unit squares in figure n is the sum of the first n positive oddintegers plus the sum of the first n + 1 positive odd integers. Since the sum ofthe first k positive odd integers is k2, figure n has n2 + (n + 1)2 unit squares.So figure 100 has 1002 + 1012 = 20201 unit squares.

9. (C) Note that the integer average condition means that the sum of the scoresof the first n students is a multiple of n. The scores of the first two studentsmust be both even or both odd, and the sum of the scores of the first threestudents must be divisible by 3. The remainders when 71, 76, 80, 82, and 91are divided by 3 are 2, 1, 2, 1, and 1, respectively. Thus the only sum of threescores divisible by 3 is 76 + 82 + 91 = 249, so the first two scores entered are76 and 82 (in some order), and the third score is 91. Since 249 is 1 larger thana multiple of 4, the fourth score must be 3 larger than a multiple of 4, and theonly possibility is 71, leaving 80 as the score of the fifth student.

10. (E) Reflecting the point (1, 2, 3) in the xy-plane produces (1, 2,!3). A half-turn about the x-axis yields (1,!2, 3). Finally, the translation gives (1, 3, 3).

Page 102: AMC

4 Solutions 2000 AMC..........................12

11. (E) Combine the three terms over a common denominator and replace ab inthe numerator with a! b to get

a

b+

b

a! ab =

a2 + b2 ! (ab)2

ab

=a2 + b2 ! (a! b)2

ab

=a2 + b2 ! (a2 ! 2ab + b2)

ab

=2ab

ab= 2.

OR

Note that a = a/b ! 1 and b = 1 ! b/a. It follows thata

b+

b

a! ab =

a

b+

b

a!

!a

b! 1

" #

1! b

a

$

=a

b+

b

a!

#a

b+

b

a! 2

$

= 2.

12. (E) Note that

AMC+AM +MC+CA = (A+1)(M +1)(C+1)!(A+M +C)!1 = pqr!13,

where p, q, and r are positive integers whose sum is 15. A case-by-case analysisshows that pqr is largest when p = 5, q = 5, and r = 5. Thus the answer is5 · 5 · 5! 13 = 112.

13. (C) Suppose that the whole family drank x cups of milk and y cups of co!ee.Let n denote the number of people in the family. The information given impliesthat x/4 + y/6 = (x + y)/n. This leads to

3x(n! 4) = 2y(6! n).

Since x and y are positive, the only positive integer n for which both sideshave the same sign is n = 5.

OR

If Angela drank c cups of co!ee and m cups of milk, then 0 < c < 1 andm + c = 1. The number of people in the family is 6c + 4m = 4 + 2c, which isan integer if and only if c = 1

2 . Thus, there are 5 people in the family.

Page 103: AMC

Solutions 2000 AMC..........................12 5

14. (E) If x were less than or equal to 2, then 2 would be both the median and themode of the list. Thus x > 2. Consider the two cases 2 < x < 4, and x " 4.

Case 1: If 2 < x < 4, then 2 is the mode, x is the median, and 25+x7 is the

mean, which must equal 2 ! (x! 2) , x+22 , or x + (x! 2), depending on the

size of the mean relative to 2 and x . These give x = 38 , x = 36

5 , and x = 3, ofwhich x = 3 is the only value between 2 and 4.

Case 2: If x " 4, then 4 is the median, 2 is the mode, and 25+x7 is the mean,

which must be 0, 3, or 6. Thus x = !25,!4, or 17, of which 17 is the only oneof these values greater than or equal to 4.

Thus the x-values sum to 3 + 17 = 20.

15. (B) Let x = 9z. Then f(3z) = f(9z/3) = f(3z) = (9z)2 + 9z + 1 = 7.Simplifying and solving the equation for z yields 81z2 + 9z ! 6 = 0, so 3(3z +1)(9z ! 2) = 0. Thus z = !1/3 or z = 2/9. The sum of these values is !1/9.

Note. The answer can also be obtained by using the sum-of-roots formula on81z2 + 9z ! 6 = 0. The sum of the roots is !9/81 = !1/9.

16. (D) Suppose each square is identified by an ordered pair (m,n), where m isthe row and n is the column in which it lies. In the original system, each square(m,n) has the number 17(m ! 1) + n assigned; in the renumbered system, ithas the number 13(n ! 1) + m assigned to it. Equating the two expressionsyields 4m!3n = 1, whose acceptable solutions are (1, 1), (4, 5), (7, 9), (10, 13),and (13, 17). These squares are numbered 1, 56, 111, 166, and 221, respectively,and the sum is 555.

Page 104: AMC

6 Solutions 2000 AMC..........................12

17. (D) The fact that OA = 1 implies that BA = tan !and BO = sec !. Since BC bisects ! ABO, itfollows that OB

BA = OCCA , which implies OB

OB+BA =OC

OC+CA = OC. Substituting yields

OC =sec !

sec ! + tan !=

1

1 + sin !.

...........................................................................................................................................................................

.......................................................................................................................................................

..................................................................................................

...............................................................................................................................................................

...........................

.................................................................................................................................................................................................................................................................................................................

•O

!

•B

•C

•A

OR

Let " = ! CBO = ! ABC. Using the Law of Sines on triangle BCO yieldssin !

BC=

sin "

OC, so OC =

BC sin "

sin !. In right triangle ABC, sin " =

1!OC

BC.

Hence OC =1!OC

sin !. Solving this for OC yields OC =

1

1 + sin !.

18. (A) Note that, if a Tuesday is d days after a Tuesday, then d is a multiple of7. Next, we need to consider whether any of the years N!1, N, N +1 is a leapyear. If N is not a leap year, the 200th day of year N+1 is 365!300+200 = 265days after a Tuesday, and thus is a Monday, since 265 is 6 larger than a multipleof 7. Thus, year N is a leap year and the 200th day of year N + 1 is anotherTuesday (as given), being 266 days after a Tuesday. It follows that year N ! 1is not a leap year. Therefore, the 100th day of year N ! 1 precedes the givenTuesday in year N by 365!100+300 = 565 days, and therefore is a Thursday,since 565 = 7 · 80 + 5 is 5 larger than a multiple of 7.

19. (C) By Heron’s Formula the area of triangle ABC is%

(21)(8)(7)(6), which is84, so the altitude from vertex A is2(84)/14 = 12. The midpoint D dividesBC into two segments of length 7, andthe bisector of angle BAC divides BC intosegments of length 14(13/28) = 6.5 and14(15/28) = 7.5 (since the angle bisector di-vides the opposite side into lengths propor-tional to the remaining two sides). Thus thetriangle ADE has base DE = 7! 6.5 = 0.5and altitude 12, so its area is 3.

................................................................................................................................................................................................................................................................................................................................................................................................................... .......

.......

.......

.......

.......

.......

.......

.......

.......

.......

.......

.......

.......

.......

.......

.......

.......

.......

.......

.......

.......

.......

.......

.......

....................................................................................................................................................................................•C

•B

•D

•E

•A

Page 105: AMC

Solutions 2000 AMC..........................12 7

20. (B) Note that (x+1/y)+ (y +1/z)+ (z +1/x) = 4+1+7/3 = 22/3 and that

28/3 = 4 · 1 · 7/3 = (x + 1/y)(y + 1/z)(z + 1/x)

= xyz + x + y + z + 1/x + 1/y + 1/z + 1/(xyz)

= xyz + 22/3 + 1/(xyz).

It follows that xyz + 1/(xyz) = 2 and (xyz ! 1)2 = 0. Hence xyz = 1.

OR

By substitution,

4 = x +1

y= x +

1

1! 1/z= x +

1

1! 3x/(7x! 3)= x +

7x! 3

4x! 3.

Thus 4(4x ! 3) = x(4x ! 3) + 7x ! 3, which simplifies to (2x ! 3)2 = 0.Accordingly, x = 3/2, z = 7/3 ! 2/3 = 5/3, and y = 1 ! 3/5 = 2/5, soxyz = (3/2)(2/5)(5/3) = 1.

21. (D) Without loss of generality, let the sideof the square have length 1 unit and let thearea of triangle ADF be m. Let AD = r andEC = s. Because triangles ADF and FECare similar, s/1 = 1/r. Since 1

2r = m, thearea of triangle FEC is 1

2s = 12r = 1

4m ............

......................

......................

......................

......................

......................

......................

......................

....................•

Ar

s

1•

B•C

•D

•E

•F

OR

Let B = (0, 0), E = (1, 0), F = (1, 1), andD = (0, 1) be the vertices of the square.Let C = (1 + 2m, 0), and notice that thearea of BEFD is 1 and the area of triangleFEC is m. The slope of the line through

C and F is ! 1

2m; thus, it intersects the

y-axis at A =!0, 1 +

1

2m

". The area of

triangle ADF is therefore1

4m.

......................

......................

......................

......................

......................

......................

......................

......................

......................

......................

......................

......................

.............•A

•(0, 0)

•(1 + 2m, 0)

•(0, 1)

•(1, 0)

•(1, 1)

Page 106: AMC

8 Solutions 2000 AMC..........................12

22. (C) First note that the quartic polynomial can have no more real zeros thanthe two shown. (If it did, the quartic P (x) ! 5 would have more than fourzeros.) The sum of the coe"cients of P is P (1), which is greater than 3.The product of all the zeros of P is the constant term of the polynomial,which is the y-intercept, which is greater than 5. The sum of the real zerosof P (the sum of the x-intercepts) is greater than 4.5, and P (!1) is greaterthan 4. However, since the product of the real zeros of P is greater than4.5 and the product of all the zeros is less than 6, it follows that the productof the non-real zeros of P is less than 2, making it the smallest of the numbers.

10

-10

-3 3

..............................................................................................................................................................................................................................................................

...........................................................................................................................................................................................................................................................................................................................................................................................................................................................................................................................................................................................................................................................................................................................................................................

Page 107: AMC

Solutions 2000 AMC..........................12 9

23. (B) In order for the sum of the logarithms of six numbers to be an integer k, theproduct of the numbers must be 10k. The only prime factors of 10 are 2 and 5,so the six integers must be chosen from the list 1, 2, 4, 5, 8, 10, 16, 20, 25, 32, 40.For each of these, subtract the number of times that 5 occurs as a fac-tor from the number of times 2 occurs as a factor. This yields the list0, 1, 2,!1, 3, 0, 4, 1,!2, 5, 2. Because 10k has just as many factors of 2 as ithas of 5, the six chosen integers must correspond to six integers in the latterlist that sum to 0. Two of the numbers must be !1 and !2, because thereare only two zeros in the list, and no number greater than 2 can appear in thesum, which must therefore be(!2) + (!1) + 0 + 0 + 1 + 2 = 0. It follows thatProfessor Gamble chose 25, 5, 1, 10, one number from {2, 20}, and one num-ber from {4, 40}. There are four possible tickets Professor Gamble could havebought and only one is a winner, so the probability that Professor Gamblewins the lottery is 1/4.

OR

As before, the six integers must be chosen from the set S = {1, 2, 4, 5, 8, 10, 16,20, 25, 32, 40}. The product of the smallest six numbers in S is 3,200 > 103,so the product of the numbers on the ticket must be 10k for some k " 4. Onthe other hand, there are only six factors of 5 available among the numbers inS, so the product p can only be 104, 105, or 106.Case 1, p = 106. There is only one way to produce 106, since all six factors of 5must be used and their product is already 106, leaving 1 as the other number:1, 5, 10, 20, 25, 40.Case 2, p = 105. To produce a product of 105 we must use six numbers thatinclude five factors of 5 and five factors of 2 among them. We cannot use both20 and 40, because these numbers combine to give five factors of 2 among themand the other four numbers would have to be odd (whereas there are only threeodd numbers in S). If we omit 40, we must include the other multiples of 5(5, 10, 20, 25) plus two numbers whose product is 4 (necessarily 1 and 4). If weomit 20, we must include 5, 10, 25, and 40, plus two numbers with a productof 2 (necessarily 1 and 2).Case 3, p = 104. To produce a product of 104 we must use six numbers thatinclude four factors of 5 and four factors of 2 among them. So that there areonly four factors of 2, we must include 1, 5, 25, 2, and 10. These include twofactors of 2 and four factors of 5, so the sixth number must contain two factorsof 2 and no 5’s, so must be 4.Thus there are four lottery tickets whose numbers have base-ten logarithmswith an integer sum: {1, 5, 10, 20, 25, 40}, {1, 2, 5, 10, 25, 40}, {1, 2, 4, 5, 10, 25},and {1, 4, 5, 10, 20, 25}. Professor Gamble has a 1/4 probability of being awinner.

Page 108: AMC

10 Solutions 2000 AMC..........................12

24. (D) Construct the circle with center A and radius AB. Let F be the point oftangency of the two circles. Draw AF , and let E be the point of intersection ofAF and the given circle. By the Power of a Point Theorem, AD2 = AF · AE(see Note below). Let r be the radius of the smaller circle. Since AF and ABare radii of the larger circle, AF = AB and AE = AF ! EF = AB ! 2r.Because AD = AB/2, substitution into the first equation yields

(AB/2)2 = AB · (AB ! 2r),

or, equivalently,r

AB=

3

8. Points A, B, and C

are equidistant from each other, so#BC = 60"

and thus the circumference of the larger circle is 6·(length of

#BC) = 6·12. Let c be the circumference

of the smaller circle. Since the circumferences ofthe two circles are in the same ratio as their radii,c

72=

r

AB=

3

8. Therefore c = 3

8(72) = 27.

.......

.......

............................................................................................

....

.......

.......

.............................................................

...................

..........................

..........................................................................................................................................................................................................................................................................................................................................................................................................................

............................

..........................................................................................................................

.......................................................................

....................................................................................................................................................................

..............................................................................................

•E

•A

•B

•C

• r

•D

•F

Note. From any exterior point P , a secant PAB and a tangent PT are drawn.Consider triangles PAT and PTB. They have a common angle P . Since an-

gles ATP and PBT intercept the same arc#AT , they are congruent. Therefore

triangles PAT and PTB are similar, and it follows that PA/PT = PT/PBand PA · PB = PT 2. The number PT 2 is called the power of the point Pwith respect to the circle. Intersecting secants, tangents, and chords, pairedin any manner create various cases of this theorem, which is sometimes calledCrossed Chords.

................................................................................................................................................................................................................................................................................................................

..............................................................................................................................................................................................................

.............................................................................................................................................................................................................................................................................................................................................................................................

..............................................................................................................................................................................

...................

...........................

.................

• B

•P •T

•A

Page 109: AMC

Solutions 2000 AMC..........................12 11

25. (E) The octahedron has 8 congruent equilateral triangular faces that form 4pairs of parallel faces. Choose one color for the bottom face. There are 7choices for the color of the top face. Three of the remaining faces have anedge in common with the bottom face. There are

&63

'= 20 ways of choosing

the colors for these faces and two ways to arrange these on the three faces(clockwise and counterclockwise). Finally, there are 3! = 6 ways to fix the lastthree colors. Thus the total number of distinguishable octahedrons that canbe constructed is 7 · 20 · 2 · 6 = 1680.

OR

Place a cube inside the octahedron so thateach of its vertices touches a face of theoctahedron. Then assigning colors to thefaces of the octahedron is equivalent to as-signing colors to the vertices of the cube.Pick one vertex and assign it a color. Thenthe remaining colors can be assigned in 7!ways. Since three vertices are joined byedges to the first vertex, they are inter-changeable by a rotation of the cube, hencethe answer is 7!/3 = 1680.

...................................................................................................................................................................................................................................................................................................................................................................................................................................

. . . . . . . . . . . . . . . . . . . . . . . . . . . . . . . . . .............................

............................

............................

............................

............................

............................

.......

..................................................................................................................................................................................................................................................................................................................................................................................................

............................................................................................................................................................................................................................

.......................................................................................................................................................................................

..............................

..............................

..............................

..............................

..............................

..........

..................................

..................................

Page 110: AMC

52th AMC 12 2001

1. The sum of two numbers is S. Suppose 3 is added to each number and theneach of the resulting numbers is doubled. What is the sum of the final twonumbers?

(A) 2S + 3 (B) 3S + 2 (C) 3S + 6 (D) 2S + 6 (E) 2S + 12

2. Let P (n) and S(n) denote the product and the sum, respectively, of the digitsof the integer n. For example, P (23) = 6 and S(23) = 5. Suppose N is atwo-digit number such that N = P (N) + S(N). What is the units digit of N?

(A) 2 (B) 3 (C) 6 (D) 8 (E) 9

3. The state income tax where Kristin lives is levied at the rate of p% of the first$28000 of annual income plus (p + 2)% of any amount above $28000. Kristinnoticed that the state income tax she paid amounted to (p + 0.25)% of herannual income. What was her annual income?

(A) $28000 (B) $32000 (C) $35000 (D) $42000 (E) $56000

4. The mean of three numbers is 10 more than the least of the numbers and 15less than the greatest. The median of the three numbers is 5. What is theirsum?

(A) 5 (B) 20 (C) 25 (D) 30 (E) 36

5. What is the product of all positive odd integers less than 10000?

(A)10000!

(5000!)2(B)

10000!

25000(C)

9999!

25000

(D)10000!

25000 · 5000!(E)

5000!

25000

6. A telephone number has the form ABC-DEF-GHIJ, where each letter repre-sents a di!erent digit. The digits in each part of the number are in decreasingorder; that is, A > B > C, D > E > F, and G > H > I > J. Furthermore,D, E, and F are consecutive even digits; G, H, I, and J are consecutive odddigits; and A + B + C = 9. Find A.

(A) 4 (B) 5 (C) 6 (D) 7 (E) 8

1

Page 111: AMC

52th AMC 12 2001

7. A charity sells 140 benefit tickets for a total of $2001. Some tickets sell forfull price (a whole dollar amount), and the rest sell for half price. How muchmoney is raised by the full-price tickets?

(A) $782 (B) $986 (C) $1158 (D) $1219 (E) $1449

8. Which of the cones below can be formed from a 252! sector of a circle of radius10 by aligning the two straight sides?

.......

.......

.......

......................................................................................................................

...................................

..........................................................................................................................................................................................................................................................................................................................................................................................................................

...................

........................................

252!.....................................................................................................................................

•10

(A)

.........................................................................................................................................................

.........................................................................................................................................................

...................................................................

........................

.............................................................................................................................................................

......................................................................

.......

.......

.......

.......

.......

.......

.......

.......

.......

.......

.......

.......

.......

.......

.......

.......

.......

.......

......

10

6•

(B)

.........................................................................................................................................................

.........................................................................................................................................................

...................................................................

........................

.............................................................................................................................................................

.......

.......

.......

.......

.......

.......

.......

.......

.......

.......

.......

.......

.......

.......

.......

.......

.......

.......

.......

......

...............................................................

10

6•

(C)

.........................................................................................................................................................

.........................................................................................................................................................

...................................................................

........................

.............................................................................................................................................................

.......

.......

.......

.......

.......

.......

.......

.......

.......

.......

.......

.......

.......

.......

.......

.......

.......

.......

.......

......

...............................................................

10

7•

(D)

.........................................................................................................................................................

.........................................................................................................................................................

...................................................................

........................

.............................................................................................................................................................

.......

.......

.......

.......

.......

.......

.......

.......

.......

.......

.......

.......

.......

.......

.......

.......

.......

.......

.......

......

...............................................................

10

7•

(E)

.........................................................................................................................................................

.........................................................................................................................................................

...................................................................

........................

.............................................................................................................................................................

.......

.......

.......

.......

.......

.......

.......

.......

.......

.......

.......

.......

.......

.......

.......

.......

.......

.......

.......

......

...............................................................

10

8•

2

Page 112: AMC

52th AMC 12 2001

9. Let f be a function satisfying f(xy) = f(x)/y for all positive real numbers xand y. If f(500) = 3, what is the value of f(600)?

(A) 1 (B) 2 (C)5

2(D) 3 (E)

18

5

10. The plane is tiled by congruent squares and congruent pentagons as indicated.The percent of the plane that is enclosed by the pentagons is closest to

(A) 50 (B) 52 (C) 54 (D) 56 (E) 58

....................................

........................

............

....................................

........................

............

....................................

........................

............

....................................

........................

............

....................................

........................

............

....................................

........................

............

....................................

........................

............

....................................

........................

............

....................................

........................

............

00 ···

...

1

1 ···

...

2

2 ···

...

3

3 ···

...

4

4 ···

...

5

5 ···

...

6

6 ···

...

7

7 ···

...

8

8 ···

...

9

9 ···

...

11. A box contains exactly five chips, three red and two white. Chips are randomlyremoved one at a time without replacement until all the red chips are drawnor all the white chips are drawn. What is the probability that the last chipdrawn is white?

(A)3

10(B)

2

5(C)

1

2(D)

3

5(E)

7

10

12. How many positive integers not exceeding 2001 are multiples of 3 or 4 but not5?

(A) 768 (B) 801 (C) 934 (D) 1067 (E) 1167

13. The parabola with equation y = ax2+bx+c and vertex (h, k) is reflected aboutthe line y = k. This results in the parabola with equation y = dx2 + ex + f .Which of the following equals a + b + c + d + e + f?

(A) 2b (B) 2c (C) 2a + 2b (D) 2h (E) 2k

14. Given the nine-sided regular polygon A1A2A3A4A5A6A7A8A9, how many dis-tinct equilateral triangles in the plane of the polygon have at least two verticesin the set {A1, A2, . . . A9}?

(A) 30 (B) 36 (C) 63 (D) 66 (E) 72

3

Page 113: AMC

52th AMC 12 2001

15. An insect lives on the surface of a regular tetrahedron with edges of length 1.It wishes to travel on the surface of the tetrahedron from the midpoint of oneedge to the midpoint of the opposite edge. What is the length of the shortestsuch trip? (Note: Two edges of a tetrahedron are opposite if they have nocommon endpoint.)

...................................................................................................................................................................................................................................................................................................................................................................................................

............................

............................

............................

............................

................

.......................................................................................................................................................................................

. . . . . . . . . . . . . . .

(A)1

2

!3 (B) 1 (C)

!2 (D)

3

2(E) 2

16. A spider has one sock and one shoe for each of its eight legs. In how manydi!erent orders can the spider put on its socks and shoes, assuming that, oneach leg, the sock must be put on before the shoe?

(A) 8! (B) 28 · 8! (C) (8!)2 (D)16!

28(E) 16!

4

Page 114: AMC

52th AMC 12 2001

17. A point P is selected at random from the interior of the pentagon with verticesA = (0, 2), B = (4, 0), C = (2! + 1, 0), D = (2! + 1, 4), and E = (0, 4). Whatis the probability that " APB is obtuse?

(A)1

5(B)

1

4(C)

5

16(D)

3

8(E)

1

2

........................................................................................................................................................................................................................................................................................................

.....................

................................

..............................................................................................................................

•B

•C

•D•E

•A

18. A circle centered at A with a radius of 1 and a circle centered at B with aradius of 4 are externally tangent. A third circle is tangent to the first twoand to one of their common external tangents as shown. The radius of thethird circle is

(A)1

3(B)

2

5(C)

5

12(D)

4

9(E)

1

2..........

.......................

...................................

........................................................................................................................................................................................................................................................................................................................................................................................................................................................................................................

..........................

................................................................................................................................................................................................................

.......

.....................................................................................................................................................................

...................................

.................................................

........................................

•A

•B

19. The polynomial P (x) = x3 + ax2 + bx + c has the property that the mean ofits zeros, the product of its zeros, and the sum of its coe"cients are all equal.If the y-intercept of the graph of y = P (x) is 2, what is b?

(A) "11 (B) "10 (C) "9 (D) 1 (E) 5

5

Page 115: AMC

52th AMC 12 2001

20. Points A = (3, 9), B = (1, 1), C = (5, 3), and D = (a, b) lie in the first quadrantand are the vertices of quadrilateral ABCD. The quadrilateral formed byjoining the midpoints of AB, BC,CD, and DA is a square. What is the sumof the coordinates of point D?

(A) 7 (B) 9 (C) 10 (D) 12 (E) 16

21. Four positive integers a, b, c, and d have a product of 8! and satisfy

ab + a + b = 524,

bc + b + c = 146, and

cd + c + d = 104.

What is a" d?

(A) 4 (B) 6 (C) 8 (D) 10 (E) 12

22. In rectangle ABCD, points F and G lie on AB so that AF = FG = GB andE is the midpoint of DC. Also, AC intersects EF at H and EG at J . Thearea of rectangle ABCD is 70. Find the area of triangle EHJ .

(A)5

2(B)

35

12(C) 3 (D)

7

2(E)

35

8

........................

........................

........................

........................

........................

........................

.............

....................................................................................................................................................................................•

A•B

•C•D •E

•F

•G

•H •J

23. A polynomial of degree four with leading coe"cient 1 and integer coe"cientshas two real zeros, both of which are integers. Which of the following can alsobe a zero of the polynomial?

(A)1 + i

!11

2(B)

1 + i

2(C)

1

2+ i (D) 1 +

i

2(E)

1 + i!

13

2

6

Page 116: AMC

52th AMC 12 2001

24. In triangle ABC, " ABC = 45!. Point D is on BC so that 2 · BD = CD and" DAB = 15!. Find " ACB.

(A) 54! (B) 60! (C) 72! (D) 75! (E) 90!

.........................................................................................................................................................................................................................................................................................................................................................................................................................................

..............................................

..............................................

..............................................

........

•A

•B

•C

•D

25. Consider sequences of positive real numbers of the form x, 2000, y, . . . , inwhich every term after the first is 1 less than the product of its two immediateneighbors. For how many di!erent values of x does the term 2001 appearsomewhere in the sequence?

(A) 1 (B) 2 (C) 3 (D) 4 (E) more than 4

7

Page 117: AMC

Solutions 2001 52nd AMC 12

1. (E) Suppose the two numbers are a and b. Then the desired sum is

2(a + 3) + 2(b + 3) = 2(a + b) + 12 = 2S + 12.

2. (E) Suppose N = 10a+b. Then 10a+b = ab+(a+b). It follows that 9a = ab,which implies that b = 9, since a != 0.

3. (B) If Kristin’s annual income is x " 28000 dollars, then

p

100· 28000 +

p + 2

100· (x# 28000) =

p + 0.25

100· x.

Multiplying by 100 and expanding yields

28000p + px + 2x# 28000p# 56000 = px + 0.25x.

So, 1.75x = 74x = 56000 and x = 32000.

4. (D) Since the median is 5, we can write the three numbers as x, 5, and y,where

1

3(x + 5 + y) = x + 10 and

1

3(x + 5 + y) + 15 = y.

If we add these equations, we get

2

3(x + 5 + y) + 15 = x + y + 10,

and solving for x + y gives x + y = 25. Hence, the sum of the numbers isx + 5 + y = 30.

OR

Let m be the mean of the three numbers. Then the least of the numbers ism # 10 and the greatest is m + 15. The middle of the three numbers is themedian, 5. So

1

3((m# 10) + 5 + (m + 15)) = m

and m = 10. Hence, the sum of the three numbers is 3(10) = 30.

1

Page 118: AMC

Solutions 2001 52nd AMC 12

5. (D) Note that

1 · 3 · · · 9999 =1 · 2 · 3 · · · 9999 · 10000

2 · 4 · · · 10000=

10000!

25000 · 1 · 2 · · · 5000=

10000!

25000 · 5000!.

6. (E) The last four digits (GHIJ) are either 9753 or 7531, and the remainingodd digit (either 1 or 9) is A, B, or C. Since A + B + C = 9, the odd digitamong A, B, and C must be 1. Thus the sum of the two even digits in ABCis 8. The three digits in DEF are 864, 642, or 420, leaving the pairs 2 and 0, 8and 0, or 8 and 6, respectively, as the two even digits in ABC. Of those, onlythe pair 8 and 0 has sum 8, so ABC is 810, and the required first digit is 8.The only such telephone number is 810-642-9753.

7. (A) Let n be the number of full-price tickets and p be the price of each indollars. Then

np + (140# n) · p

2= 2001, so p(n + 140) = 4002.

Thus n + 140 must be a factor of 4002 = 2 · 3 · 23 · 29. Since 0 $ n $ 140, wehave 140 $ n + 140 $ 280, and the only factor of 4002 that is in the requiredrange for n + 140 is 174 = 2 · 3 · 29. Therefore, n + 140 = 174, so n = 34 andp = 23. The money raised by the full-price tickets is 34 · 23 = 782 dollars.

8. (C) The slant height of the cone is 10, the radius of the sector. The circum-ference of the base of the cone is the same as the length of the sector’s arc.This is 252/360 = 7/10 of the circumference, 20!, of the circle from which thesector is cut. The base circumference of the cone is 14!, so its radius is 7.

2

Page 119: AMC

Solutions 2001 52nd AMC 12

9. (C) Note that

f(600) = f!500 · 6

5

"=

f(500)

6/5=

3

6/5=

5

2.

OR

For all positive x,

f(x) = f(1 · x) =f(1)

x,

so xf(x) is the constant f(1). Therefore,

600f(600) = 500f(500) = 500(3) = 1500,

so f(600) = 1500600 = 5

2 .

Note. f(x) = 1500x is the unique function satisfying the given conditions.

10. (D) The pattern shown below is repeated in the plane. In fact, nine repetitionsof it are shown in the statement of the problem. Note that four of the ninesquares in the three-by-three square are not in the four pentagons that makeup the three-by-three square. Therefore, the percentage of the plane that isenclosed by pentagons is

1# 4

9=

5

9= 55

5

9%.

..........................................................................................................................................................

3

Page 120: AMC

Solutions 2001 52nd AMC 12

11. (D) Think of continuing the drawing until all five chips are removed from thebox. There are ten possible orderings of the colors: RRRWW, RRWRW,RWRRW, WRRRW, RRWWR, RWRWR, WRRWR, RWWRR, WRWRR,and WWRRR. The six orderings that end in R represent drawings that wouldhave ended when the second white chip was drawn.

OR

Imagine drawing until only one chip remains. If the remaining chip is red,then that draw would have ended when the second white chip was removed.The last chip will be red with probability 3/5.

12. (B) For integers not exceeding 2001, there are %2001/3& = 667 multiples of 3and %2001/4& = 500 multiples of 4. The total, 1167, counts the %2001/12& =166 multiples of 12 twice, so there are 1167 # 166 = 1001 multiples of 3or 4. From these we exclude the %2001/15& = 133 multiples of 15 and the%2001/20& = 100 multiples of 20, since these are multiples of 5. However, thisexcludes the %2001/60& = 33 multiples of 60 twice, so we must re-include these.The number of integers satisfying the conditions is 1001#133#100+33 = 801.

13. (E) The equation of the first parabola can be written in the form

y = a(x# h)2 + k = ax2 # 2axh + ah2 + k,

and the equation for the second (having the same shape and vertex, but open-ing in the opposite direction) can be written in the form

y = #a(x# h)2 + k = #ax2 + 2axh# ah2 + k.

Hence,

a+b+c+d+e+f = a+(#2ah)+(ah2 +k)+(#a)+(2ah)+(#ah2 +k) = 2k.

OR

The reflection of a point (x, y) about the line y = k is (x, 2k # y). Thus, theequation of the reflected parabola is

2k # y = ax2 + bx + c, or equivalently, y = 2k # (ax2 + bx + c).

Hence a + b + c + d + e + f = 2k.

4

Page 121: AMC

Solutions 2001 52nd AMC 12

14. (D) Each of the#

92

$= 36 pairs of vertices determines two equilateral triangles,

for a total of 72 triangles. However, the three triangles A1A4A7, A2A5A8, andA3A6A9 are each counted 3 times, resulting in an overcount of 6. Thus, thereare 66 distinct equilateral triangles.

15. (B) Unfold the tetrahedron onto a plane. The two opposite-edge midpointsbecome the midpoints of opposite sides of a rhombus with sides of length 1,so are now 1 unit apart. Folding back to a tetrahedron does not change thedistance and it remains minimal.

......................................................................................................................................................................................................................................................................................................................................

..........................

..............................

................

.............

.......

.............................................

16. (D) Number the spider’s legs from 1 through 8, and let ak and bk denote thesock and shoe that will go on leg k. A possible arrangement of the socks andshoes is a permutation of the sixteen symbols a1, b1, . . . , a8, b8, in which ak

precedes bk for 1 $ k $ 8. There are 16! permutations of the sixteen symbols,and a1 precedes b1 in exactly half of these, or 16!/2 permutations. Similarly,a2 precedes b2 in exactly half of those, or 16!/22 permutations. Continuing, wecan conclude that ak precedes bk for 1 $ k $ 8 in exactly 16!/28 permutations.

5

Page 122: AMC

Solutions 2001 52nd AMC 12

17. (C) Since ! APB = 90" if and only if P lies on the semicircle with center(2, 1) and radius

'5, the angle is obtuse if and only if the point P lies inside

this semicircle. The semicircle lies entirely inside the pentagon, since thedistance, 3, from (2, 1) to DE is greater than the radius of the circle. Thus,the probability that the angle is obtuse is the ratio of the area of the semicircleto the area of the pentagon.

Let O = (0, 0), A = (0, 2), B = (4, 0), C = (2! + 1, 0), D = (2! + 1, 4), andE = (0, 4). Then the area of the pentagon is

[ABCDE] = [OCDE]# [OAB] = 4 · (2! + 1)# 1

2(2 · 4) = 8!,

and the area of the semicircle is

1

2!

#'5$2

=5

2!.

The probability is52!

8!=

5

16.

........................................................................................................................................................................................................................................................................................................

.....................

................................

..............................................................................................................................

•B

•C

•D•E

•A

6

Page 123: AMC

Solutions 2001 52nd AMC 12

18. (D) Let C be the intersection of the horizontal line through A and the verticalline through B. In right triangle ABC, we have BC = 3 and AB = 5, soAC = 4. Let x be the radius of the third circle, and D be the center. Let Eand F be the points of intersection of the horizontal line through D with thevertical lines through B and A, respectively, as shown.

In (BED we have BD = 4 + x and BE = 4# x, so

DE2 = (4 + x)2 # (4# x)2 = 16x,

and DE = 4'

x. In (ADF we have AD = 1 + x and AF = 1# x, so

FD2 = (1 + x)2 # (1# x)2 = 4x,

and FD = 2'

x. Hence,

4 = AC = FD + DE = 2'

x + 4'

x = 6'

x

and'

x = 23 , which implies x = 4

9 .

.......................................................................................................................................................................................................................................

.................................................................................................................................................................................................................................................................................

....................

.......................

............................

.................................................

...................................................................................................................................................................................................................................................................................................................................................................................................................................................................................................................................................................................................................................................................................................

..............................

..........................................................................................................................................................................................................................................................................................................................

..................

....

.....................................................................................................................................................................................................................................................

..............................................

..............................................................................

...................................................

•A

•B

G

C

F E

7

Page 124: AMC

Solutions 2001 52nd AMC 12

19. (A) The sum and product of the zeros of P (x) are #a and #c, respectively.Therefore,

#a

3= #c = 1 + a + b + c.

Since c = P (0) is the y-intercept of y = P (x), it follows that c = 2. Thusa = 6 and b = #11.

20. (C) Let the midpoints of sides AB, BC,CD and DA be denoted M, N, P,and Q, respectively. Then M = (2, 5) and N = (3, 2). Since MN has slope#3, the slope of MQ must be 1/3, and MQ = MN =

'10. An equation for

the line containing MQ is thus y# 5 = 13(x# 2), or y = (x + 13)/3. So Q has

coordinates of the form#a, 1

3(a + 13)$. Since MQ =

'10, we have

(a# 2)2 +!

a + 13

3# 5

"2

= 10

(a# 2)2 +!

a# 2

3

"2

= 10

10

9(a# 2)2 = 10

(a# 2)2 = 9

a# 2 = ±3

Since Q is in the first quadrant, a = 5 and Q = (5, 6). Since Q is the midpointof AD and A = (3, 9), we have D = (7, 3), and 7 + 3 = 10.

OR

Use translation by vectors. As before, M = (2, 5) and N = (3, 2). So##)NM = *#1, 3+. The vector

##)MQ must have the same length as

##)MN and be

perpendicular to it, so##)MQ = *3, 1+. Thus, Q = (5, 6). As before, D = (7, 3),

and the answer is 10.

OR

Each pair of opposite sides of the square are parallel to a diagonal of ABCD,so the diagonals of ABCD are perpendicular. Similarly, each pair of oppositesides of the square has length half that of a diagonal, so the diagonals ofABCD are congruent. Since the slope of AC is #3 and AC is perpendicularto BD, we have

b# 1

a# 1=

1

3, so a# 1 = 3(b# 1).

8

Page 125: AMC

Solutions 2001 52nd AMC 12

Since AC = BD,

40 = (a# 1)2 + (b# 1)2 = 9(b# 1)2 + (b# 1)2 = 10(b# 1)2,

and since b is positive, b = 3 and a = 1 + 3(b# 1) = 7. So the answer is 10.

21. (D) Note that

(a + 1)(b + 1) = ab + a + b + 1 = 524 + 1 = 525 = 3 · 52 · 7,

and(b + 1)(c + 1) = bc + b + c + 1 = 146 + 1 = 147 = 3 · 72.

Since (a + 1)(b + 1) is a multiple of 25 and (b + 1)(c + 1) is not a multiple of5, it follows that a + 1 must be a multiple of 25. Since a + 1 divides 525, a isone of 24, 74, 174, or 524. Among these only 24 is a divisor of 8!, so a = 24.This implies that b + 1 = 21, and b = 20. From this it follows that c + 1 = 7and c = 6. Finally, (c + 1)(d + 1) = 105 = 3 · 5 · 7, so d + 1 = 15 and d = 14.Therefore, a# d = 24# 14 = 10.

22. (C) The area of triangle EFG is (1/6)(70) = 35/3. Triangles AFH and CEHare similar, so 3/2 = EC/AF = EH/HF and EH/EF = 3/5. Triangles AGJand CEJ are similar, so 3/4 = EC/AG = EJ/JG and EJ/EG = 3/7.

Since the areas of the triangles that have a common altitude are proportionalto their bases, the ratio of the area of (EHJ to the area of (EHG is 3/7,and the ratio of the area of (EHG to that of (EFG is 3/5. Therefore, theratio of the area of (EHJ to the area of (EFG is (3/5)(3/7) = 9/35. Thus,the area of (EHJ is (9/35)(35/3) = 3.

........................

........................

........................

........................

........................

........................

.............

....................................................................................................................................................................................

..........................

............•A

•B

•C•D •E

•F

•G

•H •J

9

Page 126: AMC

Solutions 2001 52nd AMC 12

23. (A) If r and s are the integer zeros, the polynomial can be written in the form

P (x) = (x# r)(x# s)(x2 + "x + #).

The coe!cient of x3, "#(r+s), is an integer, so " is an integer. The coe!cientof x2, # # "(r + s) + rs, is an integer, so # is also an integer. Applying thequadratic formula gives the remaining zeros as

1

2(#" ±

%"2 # 4#) = #"

2± i

'4# # "2

2.

Answer choices (A), (B), (C), and (E) require that " = #1, which impliesthat the imaginary parts of the remaining zeros have the form ±

'4# # 1/2.

This is true only for choice (A).Note that choice (D) is not possible since this choice requires " = #2, whichproduces an imaginary part of the form

'# # 1, which cannot be 1

2 .

24. (D) Let E be a point on AD such that CE is perpendicular to AD, and drawBE. Since ! ADC is an exterior angle of (ADB, it follows that

! ADC = ! DAB + ! ABD = 15" + 45" = 60".

Thus, (CDE is a 30"– 60"– 90" triangle and DE = 12CD = BD. Hence,

(BDE is isosceles and ! EBD = ! BED = 30". But ! ECB is also equal to30" and therefore (BEC is isosceles with BE = EC. On the other hand,

! ABE = ! ABD # ! EBD = 45" # 30" = 15" = ! EAB.

Thus, (ABE is isosceles with AE = BE. Hence AE = BE = EC.The right triangle AEC is also isosceles with ! EAC = ! ECA = 45". Hence,

! ACB = ! ECA + ! ECD = 45" + 30" = 75".

.........................................................................................................................................................................................................................................................................................................................................................................................................................................

..............................................

..............................................

..............................................

........

..........................

..........................

.......................

..........................

..........................

.........

•A•B

E

C

•D

10

Page 127: AMC

Solutions 2001 52nd AMC 12

25. (D) If a, b, and c are three consecutive terms of such a sequence, thenac# 1 = b, which can be rewritten as c = (1 + b)/a. Applying this rule recur-sively and simplifying yields

. . . , a , b ,1 + b

a,

1 + a + b

ab,

1 + a

b, a , b , . . .

This shows that at most five di"erent terms can appear in such a sequence.Moreover, the value of a is determined once the value 2000 is assigned to band the value 2001 is assigned to another of the first five terms. Thus, thereare four such sequences that contain 2001 as a term, namely

2001, 2000, 1, 11000 ,

10011000 , 2001, . . . ,

1, 2000, 2001, 10011000 ,

11000 , 1, . . . ,

20014001999 , 2000, 4001999, 2001, 2002

4001999 ,2001

4001999 , . . . , and

4001999, 2000, 20014001999 ,

20024001999 , 2001, 4001999, . . . ,

respectively. The four values of x are 2001, 1, 20014001999 , and 4001999.

11

Page 128: AMC

Exercises of Geometry 1

Geometry

1. A chord which is the perpendicular bisector of a radius of length 12 in a circle, has length

(A) 3!

3 (B) 27 (C) 6!

3 (D) 12!

3 (E) none of these

2. In "ABC with right angle at C, altitude CH and median CM trisect the right angle. If the area of"CHM is K, then the area de"ABC is

(A) 6K (B) 4!

3K (C) 3!

3K (D) 3K (E) 4K

3. If the side of one square is the diagonal of a second square, what is the radio of the area of the firstsquare to the area of the second ?

(A) 2 (B)!

2 (C) 12 (D) 2

!2 (E) 4

4. If the sum of all the angles except one of a convex polygon is 2190o, then the numbers of sides of thepolygon must be

(A) 13 (B) 15 (C) 17 (D) 19 (E) 21

5. The volume of a pyramid whose base is an equilateral triangle of side length 6 and whose other edgesare each of length

!15 is

(A) 9 (B) 92 (C) 27

2 (D) 9!

32 (E) none of these

6. In the adjoining figure ABCD is a aquare and CMN is an equilateral triangle. If the area of ABCD isone square inch, then the area of CMN in square inches is

(A) 2!

3# 3

(B) 1#!

33

(C)!

34

(D)!

23

(E) 4# 2!

3

7. In the adjoining figure triangle ABC is such that AB=4 and AC=8. If M is the midpoint of BC andAM=3, what is the length of BC ?

(A) 2!

26

(B) 2!

31

(C) 9

(D) 4 + 2!

13

(E) not enough information given to solve theproblem

www.matebrunca.com Teacher Waldo Marquez Gonzalez

Page 129: AMC

Exercises of Geometry 2

8. Which one of the following statements is false? All equilateral triangles are

(A) equiangular(B) isosceles

(C) regular polygons(D) congruent to each other

(E) similar to each other

9. In the adjoining figure " E = 40o and arc AB, arc BC, and arc CD all have equal length. Find themeasure of " ACD.

(A) 10o

(B) 15o

(C) 20o

(D) 30o

(E) 22.5o

10. Each of the three circles in the adjoining figure is externally tangent to the other two, and each sideof the triangle is tangent to two of the circles. If each circle has radius three, then the perimeter of thetriangle is

(A) 36 + 9!

2

(B) 36 + 6!

3

(C) 36 + 9!

3

(D) 18 + 18!

3

(E) 45

11. Opposite sides of a regular hexagon are 12 inches apart. The length os each side, in inches, is

(A) 7, 5 (B) 6!

2 (C) 5!

2 (D) 92

!3 (E) 4

!3

12. If rectangle ABCD has area 72 square meters and E and G are the midpoints of sides AD and CD,respectively, then the are of rectangle DEFG in square meters is

(A) 8

(B) 9

(C) 12

(D) 18

(E) 34

www.matebrunca.com Teacher Waldo Marquez Gonzalez

Page 130: AMC

Exercises of Geometry 3

13. In the adjoining figura, ABCD is a square, ABE is an equilateral triangle and point E is outside squareABCD. What is the measure of " AED in degrees?

(A) 10

(B) 12.5

(C) 15

(D) 20

(E) 25

14. In the adjoining figure, CD is the diameter of a semi-circle with center O. Point A lies on the extensionof DC past C; point E lies on the semi-circle, and B is the point of intersection (distinct from E) ofline segment AE with the semi-circle. If length AB equals AD and the measure of " EOD is 45o, thenthe measure of " BAO is

(A) 10o

(B) 15o

(C) 20o

(D) 25o

(E) 30o

15. In the adjoining figure, CDE is an equilateral triangle and ABCD and DEFG are squares. The measureof of " GDA is

(A) 90o

(B) 105o

(C) 120o

(D) 135o

(E) 150o

16. If AB and CD are perpendicular diameters of circle Q, and " QPC = 60o, then the length of PQdivided by the length of AQ is

(A)!

32

(B)!

33

(C)!

22

(D) 12

(E) 23

www.matebrunca.com Teacher Waldo Marquez Gonzalez

Page 131: AMC

Exercises of Geometry 4

17. Point E is on side AB of square ABCD. If has length one and EC has length two, then the area of thesquare is

(A)!

3

(B)!

5

(C) 3(D) 2

!3

(E) 5

18. If the adjoining figure, PQ is a diagonal of the cube. If PQ has length a, then the surface area of thecube is

(A) 2a2

(B) 2!

2a2

(C) 2!

3a2

(D) 3!

3a2

(E) 6a2

19. The perimeter of a semicircular region, measure in centimeters, is numerically equal to its area, mea-sured in square centimeters. The radius of the semicircle, measure in centimeters, is

(A) ! (B) 2! (C) 1 (D) 1

2 (E) 4! + 2

20. Two congruent 30o-60o-90o triangles are placed so that they overlap partly and their hypotenusescoincide. If the hypotenuse of each triangle is 12, the area common to both triangles is

(A) 6!

3

(B) 8!

3

(C) 9!

3

(D) 12!

3

(E) 24

21. A circle of radius r is inscribed in a right isosceles triangle, and a circle of radius R is circumscribedabout the triangle (see adjacent figure). Then R

r equals

(A) 1 +!

2

(B) 2+!

22

(C)!

2#12

(D) 1+!

22

(E) 2(2#!

2)

www.matebrunca.com Teacher Waldo Marquez Gonzalez

Page 132: AMC

Exercises of Geometry 5

22. Triangle ABC has a right angle at C. If sin A = 23 , then tan A is

(A) 35 (B)

!5

3 (C) 2!5

(D)!

52 (E) 5

3

23. In the adjoining plane figure, sides AF and CD are parallel, as are sides AB and FE, and sides BC andED. Each side has length 1. Also, " FAB = " BCD = 60o. The area of the figure is

(A)!

32

(B) 1

(C) 32

(D)!

3

(E) 2

24. Figure ABCD is a trapezoid with AB $ DC, AB = 5, BC = 3!

2, " BCD = 45o and " CDA =60o. The length of DC is

(A) 7 + 23

!3

(B) 8

(C) 912

(D) 8 +!

3

(E) 8 + 3!

3

25. In an arcade game, the !!monster"" is the shaded sector of a circle of radius 1 cm, as shown in thefigure. The missing piece (the mouth) has central angle 60o. What is the perimeter of the monster incm?

(A) ! + 2

(B) 2!

(C) 53!

(D) 56! + 2

(E) 53! + 2

26. Pegs are put in a board 1 unit apart both horizontally and vertically. A rubber band is stretched over 4pegs as shown in the figure, forming a quadrilateral. Its area in square units is

(A) 4

(B) 4,5

(C) 5

(D) 5,5

(E) 6

www.matebrunca.com Teacher Waldo Marquez Gonzalez

Page 133: AMC

Exercises of Geometry 6

27. In a circle with center O, AD is a diameter, ABC is a chord, BO=5 and " ABO ="

CD= 60o. Then thelength of BC is

(A) 3

(B) 3 +!

3

(C) 5#!

32

(D) 5

(E) none of the above

28. In the figure, " ABC has a right angle at C and " A = 20o. If BD is the bisector of " ABC, then" BDC

(A) 40o

(B) 45o

(C) 50o

(D) 55o

(E) 60o

29. As shown in the figure, a triangular corner with side lengths DB = EB = 1 is cut from equilateraltriangle ABC of side length 3. The perimeter of the remaining quadrilateral ADEC is

(A) 6

(B) 612

(C) 7

(D) 712

(E) 8

30. In the figure the sum of the distances AD and BD is

(A) between 10 and 11

(B) 12

(C) between 15 and 16

(D) between 16 and 17

(E) 17

www.matebrunca.com Teacher Waldo Marquez Gonzalez

Page 134: AMC

Exercises of Geometry 7

31. Four rectangular paper strips of length 10 and width 1 are put flat on a table and overlap perpendicu-larly as shown. How much area of the table is covered?

(A) 36

(B) 40

(C) 44

(D) 96

(E) 100

32. Segment AB is both a diameter of a circle of radius 2 and a side of an equilateral triangle ABC. Thecirccle also intersects AC and BC at points D and E, respectively (see adjacent figure). The length ofAE is

(A) 32

(B) 53

(C)!

32

(D)!!

3

(E) 2+!

32

33. In the adjoining figure the five circles are tangent to one another consecutively and to the lines L1

and L2. If the radius of the largest circle is 18 and that of the smallest one is 8, then the radius of themiddle circle is

(A) 12

(B) 12.5

(C) 13

(D) 13.5

(E) 14

34. Distinct points A and B are on a semicircle with diameter MN and center C. The point P is on CNand " CAP = " CBP = 10o.

If"

MA= 40o, then"

BN equals

(A) 10o

(B) 15o

(C) 20o

(D) 25o

(E) 30o

www.matebrunca.com Teacher Waldo Marquez Gonzalez

Page 135: AMC

Exercises of Geometry 8

35. A rectangle intersects a circle as shown: AB=4, BC=5 and DE=3. Then EF equals

(A) 6

(B) 7

(C) 203

(D) 8

(E) 9

36. A right triangle ABC with hypotenuse AB has side AC=15. Altitude CH divides AB into segmentsAH and HB, with HB = 16. The area of"ABC is

(A) 120

(B) 144

(C) 150

(D) 216

(E) 144!

5

www.matebrunca.com Teacher Waldo Marquez Gonzalez

Page 136: AMC

Exercises of Geometry 9

Bibliography

1. Artino, Ralph A., Anthony M. Gaglione and Niel Shell. The Contest Problem Book IV. annual highSchool Examination: 1973-1982. Ralph A. Artino of The City College of New york, Anthony M.Gaglione of The U.S. Naval Academy and Niel Shell of The City College of New York.

2. Berzsenyi, George, and Stephen B. Maurer. The Contest Problem Book V. American High SchoolMathematics Examinations and American Invitational Mathematics Examinations: 1983-1988. Prob-lems and solutions compilad and augmented by George Berzsenyi of Rose-Hulman Institute of Tech-nology and Stephen B. Maurer of Swartmore College.

www.matebrunca.com Teacher Waldo Marquez Gonzalez